AeroElectric-Archive.digest.vol-po

August 25, 2020 - September 20, 2020



      > performance (except perhaps for charging time)
      > is unaffected by bus voltage. Note: the red plot
      > is what the battery delivered 'off the shelf' after
      > having been used/stored/maintained at intervals
      > over the past 5 or so years.
      >
      > I not that the battery's demonstrated capacity is
      > on the order of 10.3 AH . . . down slightly from
      > demonstrated 11.6 AH when received as new.
      >
      > These data are consistent with an article I
      > found on powerstream.com
      >
      > https://tinyurl.com/yakthgt6
      >
      > Where the author demonstrates the profound difference
      > in performance between Lithium Iron versus the
      > Lithium Polymer products. The article shows how the popular
      > and quite common Lithium Polymer cells fully charge at
      > 4.2 volts with stored energy profoundly affected by
      > the top-off voltage. Same goes for Lithium Ion cells
      > widely offered as rechargeable power for a constellation
      > of consumer products.
      >
      > Touchstone of note:  No matter what claims are made
      > for the capacity of these cells, the real capability
      > of an 18650 Li-Ion cell is on the order of 2800 mAH. This
      > is easily confirmed by studies of published engineering data
      > sheets for name brand cells like Panasonic, A123,
      > Sanyo, etc.
      >
      > https://tinyurl.com/y32rh8ym
      >
      > Those ads on eBay touting 4000-10,000 mAH are blowing
      > smoke up your pant leg. Now, I not implying that
      > these cells are not serviceable products . . . price/
      > performance can be quite good but unless verified
      > by published performance data sheets, be skeptical
      > of stellar capacity claims.
      >
      > That leaves us with the lowly Lithium Iron Phosphate
      > cell. It has the poorest capacity for unit volume of
      > all the popular products . . . but they are relatively
      > immune to catastrophic failure. Hence, the chemistry
      > of choice for the 'big guys' in TSO batteries for
      > CT aircraft. Typical energy ratings for the 18650
      > LiFePO4 are on the order of 1000-1500 mAH.
      >
      > A noteworthy fallout of my studies suggests there
      > is no value in running the bus at any higher
      > than 14.0 volts for the LiFePO4 products.
      >
      > Just thought I'd share some of the the bucket
      > of data point's I've gathered so far . . .
      > I'm still pondering the burned alternator
      > coils on the Revmaster engine . . . got some
      > ideas I want to massage on the test bench . . .
      >
      >
      >    Bob . . .
      
________________________________________________________________________________
From: Ron Springer <ron228rj(at)gmail.com>
Date: Aug 25, 2020
Subject: B&C SD-8 Alternator/Regulator
Due to some strange behavior on my EFIS shortly after takeoff, I theorized that I might have a noisy electrical system that was causing EFIS problems. The charging system (SD-8) doesn't kick in until I take off and achieve higher RPM. I could not reproduce the problem on the ground so I could not easily hook up an oscilloscope. Maybe because static runs don't achieve the same RPM at full throttle, and high speed taxi runs on a 3000 ft runway achieve higher RPM but don't last long enough? So, I did another flight and took off and when the problem happened again, I turned off the alternator so I was just running on the battery. The EFIS problems continued so I don't believe I have a noisy electrical system causing the problem. I also swapped out the regulator and replaced the battery just to be thorough! My real question is related to what happens when I disconnect a PM alternator from the electrical system between the regulator and the bus. Does it keep generating 8 amps that now has to be dissipated as heat by the voltage regulator? I did notice some noise, or a change in the noise, in my headset when I disconnected the alternator/regulator. Also, what would happen if I disconnected the two leads for the alternator so that they weren't even connected to a regulator? Thanks, Ron ________________________________________________________________________________
Date: Aug 25, 2020
From: "Robert L. Nuckolls, III" <nuckolls.bob(at)aeroelectric.com>
Subject: Re: B&C SD-8 Alternator/Regulator
At 02:35 PM 8/25/2020, you wrote: >Due to some strange behavior on my EFIS shortly >after takeoff, I theorized that I might have a >noisy electrical system that was causing EFIS problems. Is this a new condition or has always existed? >The charging system (SD-8) doesn't kick in until >I take off and achieve higher RPM. I could not >reproduce the problem on the ground so I could >not easily hook up an oscilloscope. Maybe >because static runs don't achieve=C2 the same RPM >at full throttle, and high speed taxi runs on a >3000 ft runway achieve higher RPM but don't last long enough? Fair hypothesis . . . >So, I did another flight and took off and when >the problem=C2 happened again, I turned off the >alternator so I was just running on the battery. >The EFIS problems continued so I don't believe I >have a noisy electrical system causing the >problem. I also swapped out the regulator and >replaced the battery just to be thorough! Good . . . >My real question is related to what happens when >I disconnect a PM alternator from the electrical >system between the regulator and the bus. Does >it keep generating 8 amps that now has to be >dissipated as heat by the voltage regulator? I >did notice some noise, or a change in the noise, >in my headset when I disconnected the alternator/regulator. Depends on the pedigree of the regulator. Some designs do indeed 'short' the alternator windings to regulate power at the battery. When ship's systems are using power, the need to short-out excess energy is minimized. But you're right, if this is a shunt style R/R, reducing output current to zero MAY cause the regulator to severely abuse the alternator mistakenly thinking that ship's loads are still there . . . just very small. >Also, what would happen if I disconnected the >two leads for the alternator so that they >weren't even connected to a regulator? Conduct the experiment. Opening the AC power BETWEEN alternator and rectifier/regulator is the preferred method for shutting down the system. All energies in the alternator system go to zero. See: https://tinyurl.com/7vp9g4e Unhook the alternator and do a battery only flight. Bob . . . ________________________________________________________________________________
Date: Aug 25, 2020
From: "Robert L. Nuckolls, III" <nuckolls.bob(at)aeroelectric.com>
Subject: Re: Some interesting observations
At 11:06 AM 8/25/2020, you wrote: >Bob I am often in awe of your energy in serving the OBAM >community..thank you for your missives, they inspire neophytes like >me to get "electron savvy" >Best Regards, >Bob Verwey >082 331 2727 To be inspiring is a teacher's prime directive . . . thank you sir. Bob . . . ________________________________________________________________________________
From: Ron Springer <ron228rj(at)gmail.com>
Date: Aug 25, 2020
Subject: Re: B&C SD-8 Alternator/Regulator
On Tue, Aug 25, 2020 at 5:34 PM Robert L. Nuckolls, III < nuckolls.bob(at)aeroelectric.com> wrote: > At 02:35 PM 8/25/2020, you wrote: > > Due to some strange behavior on my EFIS shortly after takeoff, I theorized > that I might have a noisy electrical system that was causing EFIS problems. > > > Is this a new condition or has always > existed? > I thought it was a new condition. My plane has been down for quite a while during an engine rebuild. I am trying to fly it and monitor temperatures on my new engine but all the engine data is fluctuating wildly. I looked back at some old data and I could see the same symptoms occasionally, but it was more mild and very infrequent. I just never noticed it. Now, it is happening nearly continuously and the variations are far more extreme. The EFIS has three other issues as well. It has lost the GPS signal, it always stops recording data to the USB stick once the temperature oscillations start, and it only sometimes displays the ADSB-in data that I am feeding it. One thing in common with all of these is that they are connected to serial ports on the EFIS. Either four things just simultaneously failed or there is a problem with the EFIS serial ports. That's my current theory. > Also, what would happen if I disconnected the two leads for the alternator > so that they weren't even connected to a regulator? > > > Conduct the experiment. Opening the AC power > BETWEEN alternator and rectifier/regulator is > the preferred method for shutting down the > system. All energies in the alternator > system go to zero. See: > > https://tinyurl.com/7vp9g4e > > Unhook the alternator and do a battery only > flight. > Yes, that is exactly what I am planning for the next flight. Additionally, the EIS is remotely mounted but it has a display. I intend to fly with a GoPro aimed at it to see if the CHT values that it displays match those on the EFIS. If there is simply a communication problem or some source of noise in the EFIS, then the EIS should be displaying the correct CHT values and not wildly fluctuating. Ron ________________________________________________________________________________
Date: Aug 26, 2020
From: "Robert L. Nuckolls, III" <nuckolls.bob(at)aeroelectric.com>
Subject: Re: B&C SD-8 Alternator/Regulator
>=C2 Is this a new condition or has always >=C2 existed? > >I thought it was a new condition. My plane has >been down for quite a while during an engine >rebuild. I am trying to fly it and monitor >temperatures on my new engine but all the engine >data is fluctuating=C2 wildly. I looked back at >some old data and I could see the same symptoms >occasionally, but it was more mild and very >infrequent. I just never noticed it. Now, it is >happening nearly continuously and the variations >are far more extreme. The EFIS has three other >issues as well. It has lost the GPS signal, it >always stops recording data to the USB stick >once the temperature oscillations start, and it >only sometimes displays the ADSB-in data that I >am feeding it. One thing in common with all of >these is that they are connected to serial ports >on the EFIS. Either four things just >simultaneously failed or there is a problem with >the EFIS serial ports. That's my current theory. Serial data are pretty robust with respect to noises . . . I'm pondering the possibility of a grounding issue for the analog inputs. Analog temperature data signals, whether thermocouple or RTC are small and the fact that the whole suite of measurements are flaky suggests something common to signal conditioning. >=C2 Unhook the alternator and do a battery only >=C2 flight. > >Yes, that is exactly what I am planning for the >next flight. Additionally, the EIS is remotely >mounted but it has a display. I intend to fly >with a GoPro aimed at it to see if the CHT >values that it displays match those on the EFIS. >If there is simply a communication problem or >some source of noise in the EFIS, then the EIS >should be displaying the correct CHT values and not wildly fluctuating. Is there a downloadable installation manual for your EFIS? Bob . . . ________________________________________________________________________________
From: Ron Springer <ron228rj(at)gmail.com>
Subject: Re: B&C SD-8 Alternator/Regulator
Date: Aug 26, 2020
> On Aug 26, 2020, at 10:31 AM, Robert L. Nuckolls, III <nuckolls.bob@aeroel ectric.com> wrote: > > Serial data are pretty robust with respect > to noises . . . I'm pondering the possibility > of a grounding issue for the analog inputs. > Analog temperature data signals, whether thermocouple > or RTC are small and the fact that the whole > suite of measurements are flaky suggests > something common to signal conditioning. The interesting thing is that the bad values apply across all channels, incl uding ones that aren=99t even hooked up, like cylinders 5 and 6. Anoth er one is coolant temperature. It always should read 59 deg for no signal si nce it is not hooked up. My data recording at 5 Hz captured a single inciden ce of -32000 degF and then back to 59 until recording stopped. Values in the -32000 range have shown up on CHT channels and others as well. The manufacturer is mystified. I am collecting more data today, checking all ground connections, and then will probably ship it to them for testing tomo rrow. Ron ________________________________________________________________________________
From: Charlie England <ceengland7(at)gmail.com>
Date: Aug 26, 2020
Subject: Re: B&C SD-8 Alternator/Regulator
On Wed, Aug 26, 2020 at 11:25 AM Ron Springer wrote: > > On Aug 26, 2020, at 10:31 AM, Robert L. Nuckolls, III < > nuckolls.bob(at)aeroelectric.com> wrote: > > Serial data are pretty robust with respect > to noises . . . I'm pondering the possibility > of a grounding issue for the analog inputs. > Analog temperature data signals, whether thermocouple > or RTC are small and the fact that the whole > suite of measurements are flaky suggests > something common to signal conditioning. > > > The interesting thing is that the bad values apply across all channels, > including ones that aren=99t even hooked up, like cylinders 5 and 6 . Another > one is coolant temperature. It always should read 59 deg for no signal > since it is not hooked up. My data recording at 5 Hz captured a single > incidence of -32000 degF and then back to 59 until recording stopped. > Values in the -32000 range have shown up on CHT channels and others as > well. > > The manufacturer is mystified. I am collecting more data today, checking > all ground connections, and then will probably ship it to them for testin g > tomorrow. > > Ron > I hate saying this out loud, but have you tried whacking the case to see if you can stimulate the glitches? IIRC, you said it only happens in flight, right? Vibration can generate all kinds of weird symptoms in electronics if there are intermittent connections involved. Charlie ________________________________________________________________________________
From: Ron Springer <ron228rj(at)gmail.com>
Subject: Re: B&C SD-8 Alternator/Regulator
Date: Aug 26, 2020
No problem saying it. That is worth trying! Sent from my iPhone > On Aug 26, 2020, at 1:00 PM, Charlie England wrote: > > =EF=BB > > >> On Wed, Aug 26, 2020 at 11:25 AM Ron Springer wrote: >> >>>> On Aug 26, 2020, at 10:31 AM, Robert L. Nuckolls, III <nuckolls.bob@aer oelectric.com> wrote: >>>> >>> Serial data are pretty robust with respect >>> to noises . . . I'm pondering the possibility >>> of a grounding issue for the analog inputs. >>> Analog temperature data signals, whether thermocouple >>> or RTC are small and the fact that the whole >>> suite of measurements are flaky suggests >>> something common to signal conditioning. >> >> The interesting thing is that the bad values apply across all channels, i ncluding ones that aren=99t even hooked up, like cylinders 5 and 6. An other one is coolant temperature. It always should read 59 deg for no signal since it is not hooked up. My data recording at 5 Hz captured a single inci dence of -32000 degF and then back to 59 until recording stopped. Values in t he -32000 range have shown up on CHT channels and others as well. >> >> The manufacturer is mystified. I am collecting more data today, checking a ll ground connections, and then will probably ship it to them for testing to morrow. >> >> Ron > > I hate saying this out loud, but have you tried whacking the case to see i f you can stimulate the glitches? IIRC, you said it only happens in flight, r ight? Vibration can generate all kinds of weird symptoms in electronics if t here are intermittent connections involved. > Charlie ________________________________________________________________________________
Subject: Use 1/4" hole on starter for engine ground?
From: "rparigoris" <rparigor(at)hotmail.com>
Date: Aug 26, 2020
Hi Group I have a Rotax 914 with heavy duty starter. There is a casting at the rear of the starter with an untapped 1/4" hole perhaps 1/2" thick. Is there any reason why I couldn't use this as my only engine ground? I can't use the engine lift point as ground because the wire will interfere with cooling air baffle air flow down and around the B&C SD20S alternator. There is a tapped 6mm hole on the 2 aft cylinders but the starter is closer to the firewall ground pass through. Ron P. Read this topic online here: http://forums.matronics.com/viewtopic.php?p=498003#498003 ________________________________________________________________________________
Date: Aug 26, 2020
From: "Robert L. Nuckolls, III" <nuckolls.bob(at)aeroelectric.com>
Subject: Re: B&C SD-8 Alternator/Regulator
> >I hate saying this out loud, but have you tried whacking the case to >see if you can stimulate the glitches? IIRC, you said it only >happens in flight, right? Vibration can generate all kinds of weird >symptoms in electronics if there are intermittent connections involved. >Charlie You're telling your age my friend . . . I'll bet you had real world experience with vacuum tube electronics! Bob . . . ________________________________________________________________________________
Date: Aug 26, 2020
From: "Robert L. Nuckolls, III" <nuckolls.bob(at)aeroelectric.com>
Subject: Re: Use 1/4" hole on starter for engine ground?
At 12:45 PM 8/26/2020, you wrote: > >Hi Group I have a Rotax 914 with heavy duty starter. There is a >casting at the rear of the starter with an untapped 1/4" hole >perhaps 1/2" thick. Is there any reason why I couldn't use this as >my only engine ground? Is this a thru hole . . . can you use a bolt and nut to effect the mash-up? Got a nice flat surface to mate up with your terminal? Get the terminal flat compressed to the crankcase with a stout (grade 8) bolt having a washer under the head of the bolt against the other side of the terminal. I'm presuming there's a nut on the other side of the casting. Torque it up until you're worried for the future of your 1/4" ratchet and call it good. Bob . . . ________________________________________________________________________________
From: Charlie England <ceengland7(at)gmail.com>
Date: Aug 26, 2020
Subject: Re: B&C SD-8 Alternator/Regulator
On Wed, Aug 26, 2020 at 4:50 PM Robert L. Nuckolls, III < nuckolls.bob(at)aeroelectric.com> wrote: > > I hate saying this out loud, but have you tried whacking the case to see > if you can stimulate the glitches? IIRC, you said it only happens in > flight, right? Vibration can generate all kinds of weird symptoms in > electronics if there are intermittent connections involved. > Charlie > > > You're telling your age my friend . . . I'll bet you > had real world experience with vacuum tube electronics! > > > Bob . . . > Uh...yep. Tube-top-thump was an essential troubleshooting tool; even the musicians knew that one. Still have a scar on my hand somewhere from touching the wrong (750V) terminal in the guts of a Marshall guitar amp I was repairing, some time in the mid 1970s. I've forgotten a lot of stuff; not that smell.... But I've even seen solid state components that were shock-sensitive when they started going south. Charlie ________________________________________________________________________________
Subject: Re: Use 1/4" hole on starter for engine ground?
From: "user9253" <fransew(at)gmail.com>
Date: Aug 26, 2020
Actually that ground lug on the starter is the ideal location for the engine ground cable. Current will not have to pass through the engine block to crank the engine. -------- Joe Gores Read this topic online here: http://forums.matronics.com/viewtopic.php?p=498010#498010 ________________________________________________________________________________
Date: Aug 26, 2020
From: "Robert L. Nuckolls, III" <nuckolls.bob(at)aeroelectric.com>
Subject: Re: Use 1/4" hole on starter for engine ground?(PS)
Torque it up until you're worried for the future of your 1/4" ratchet and call it good. P.S. as a point of reference, the terminals on a PC680 battery are 6 mm female threads in brass . . . Emacs! Your primary concern at this junction is integrity under engine vibration and temperature variations. Your hardware will be steel and torqued to a lot higher forces/pressures than the battery . . . Bob . . . ________________________________________________________________________________
Date: Aug 26, 2020
From: "Robert L. Nuckolls, III" <nuckolls.bob(at)aeroelectric.com>
Subject: Re: B&C SD-8 Alternator/Regulator
> >Uh...yep. Tube-top-thump was an essential >troubleshooting tool; even the musicians knew >that one. Still have a scar on my hand somewhere >from touching the wrong (750V) terminal in the >guts of a Marshall guitar amp I was repairing, >some time in the mid 1970s. I've forgotten a lot of stuff; not that smell....=C2 Speaking of smells . . . the youngster's look at you incredulously when you assert that sticking your nose into the cabinet after removing the back was a powerful diagnostic tool. Power transformers, electrolytic caps, paper caps, plastic caps, resistors, flyback transformers, resistors, etc. all had their own characteristic odors when 'smoked'. Staring with overt symptoms one could be halfway to a diagnosis by simply sniffing as you pulled the back off the set. Not so much these days . . . Bob . . . ________________________________________________________________________________
From: Ron Springer <ron228rj(at)gmail.com>
Subject: Re: B&C SD-8 Alternator/Regulator
Date: Aug 26, 2020
Just did a test flight and shot some video of the EIS to compare to the EFIS . I haven=99t even looked at it yet because I did something else for t he first time. I shut off the magneto and everything got perfectly quiet on t he EFIS, along with the noise that seems to be getting louder in my headset o n each flight! I never really focused on the magneto because it was brand new. But, let me t ell you about the old mag. I hand prop my VariEze on the magneto only and su ddenly I couldn=99t get it started any more. I suspected the mag was w eak. I opened it to adjust the egap and the slot wasn=99t big enough t o adjust it. Then I dug through the original builder=99s paperwork and learned it had 1100 hours on it and probably was never serviced. So, I just bought a new one. It would have been more obvious if I had flown the new ma g immediately but I installed it and then decided to tear down the engine an d do a bunch of other mods. I am guessing that if the wires are the problem, installing a new, much stro nger magneto can make things a lot worse? Ron Sent from my iPhone > On Aug 26, 2020, at 6:36 PM, Robert L. Nuckolls, III <nuckolls.bob@aeroele ctric.com> wrote: > > =EF=BB >> >> >> Uh...yep. Tube-top-thump was an essential troubleshooting tool; even the m usicians knew that one. Still have a scar on my hand somewhere from touching the wrong (750V) terminal in the guts of a Marshall guitar amp I was repair ing, some time in the mid 1970s. I've forgotten a lot of stuff; not that sme ll....=C3=82 > > > Speaking of smells . . . the youngster's look > at you incredulously when you assert that sticking > your nose into the cabinet after removing the back > was a powerful diagnostic tool. > > Power transformers, electrolytic caps, paper > caps, plastic caps, resistors, flyback transformers, > resistors, etc. all had their own characteristic > odors when 'smoked'. Staring with overt symptoms > one could be halfway to a diagnosis by simply sniffing > as you pulled the back off the set. > > Not so much these days . . . > > > > > Bob . . . ________________________________________________________________________________
From: Ron Springer <ron228rj(at)gmail.com>
Subject: Re: B&C SD-8 Alternator/Regulator
Date: Aug 26, 2020
I am still not sure why it only seems to occur in the air. More vibration? I t shakes pretty good at full throttle on the ground too. The ground runs hav e primarily been done the VariEze nose on the ground. But, the plane is nonc onductive. Could the proximity of the ground have an effect? Ron Sent from my iPhone > On Aug 26, 2020, at 7:28 PM, Ron Springer wrote: > > =EF=BBJust did a test flight and shot some video of the EIS to compare t o the EFIS. I haven=99t even looked at it yet because I did something e lse for the first time. I shut off the magneto and everything got perfectly q uiet on the EFIS, along with the noise that seems to be getting louder in my headset on each flight! > > I never really focused on the magneto because it was brand new. But, let m e tell you about the old mag. I hand prop my VariEze on the magneto only and suddenly I couldn=99t get it started any more. I suspected the mag wa s weak. I opened it to adjust the egap and the slot wasn=99t big enoug h to adjust it. Then I dug through the original builder=99s paperwork a nd learned it had 1100 hours on it and probably was never serviced. So, I ju st bought a new one. It would have been more obvious if I had flown the new m ag immediately but I installed it and then decided to tear down the engine a nd do a bunch of other mods. > > I am guessing that if the wires are the problem, installing a new, much st ronger magneto can make things a lot worse? > > Ron > > Sent from my iPhone > >>> On Aug 26, 2020, at 6:36 PM, Robert L. Nuckolls, III <nuckolls.bob@aeroe lectric.com> wrote: >>> >> =EF=BB >>> >>> >>> Uh...yep. Tube-top-thump was an essential troubleshooting tool; even the musicians knew that one. Still have a scar on my hand somewhere from touchi ng the wrong (750V) terminal in the guts of a Marshall guitar amp I was repa iring, some time in the mid 1970s. I've forgotten a lot of stuff; not that s mell....=C3=82 >> >> >> Speaking of smells . . . the youngster's look >> at you incredulously when you assert that sticking >> your nose into the cabinet after removing the back >> was a powerful diagnostic tool. >> >> Power transformers, electrolytic caps, paper >> caps, plastic caps, resistors, flyback transformers, >> resistors, etc. all had their own characteristic >> odors when 'smoked'. Staring with overt symptoms >> one could be halfway to a diagnosis by simply sniffing >> as you pulled the back off the set. >> >> Not so much these days . . . >> >> >> >> >> Bob . . . ________________________________________________________________________________
Date: Aug 26, 2020
From: "Robert L. Nuckolls, III" <nuckolls.bob(at)aeroelectric.com>
Subject: Re: B&C SD-8 Alternator/Regulator
At 06:28 PM 8/26/2020, you wrote: >Just did a test flight and shot some video of >the EIS to compare to the EFIS. I haven=99t even >looked at it yet because I did something else >for the first time. I shut off the magneto and >everything got perfectly quiet on the EFIS, >along with the noise that seems to be getting >louder in my headset on each flight! Aha! One more flight test: Leave the mag wire totally unhooked at the engine end. Of course this leaves you an always-hot mag but the p-lead effects are now out of the diagnostic loop. If the effect goes away, you're looking at p-lead; if it stays you're looking at plug wires. Bob . . . ________________________________________________________________________________
From: Ken Ryan <keninalaska(at)gmail.com>
Date: Aug 26, 2020
Subject: Re: B&C SD-8 Alternator/Regulator
I had a tv that would periodically just lose the picture and a very hard slap to the left side of the case would instantly bring it back. Sometimes it took two very hard slaps! On Wed, Aug 26, 2020 at 1:50 PM Robert L. Nuckolls, III < nuckolls.bob(at)aeroelectric.com> wrote: > > I hate saying this out loud, but have you tried whacking the case to see > if you can stimulate the glitches? IIRC, you said it only happens in > flight, right? Vibration can generate all kinds of weird symptoms in > electronics if there are intermittent connections involved. > Charlie > > > You're telling your age my friend . . . I'll bet you > had real world experience with vacuum tube electronics! > > > Bob . . . > ________________________________________________________________________________
Subject: Anti corrosion on connections?
From: "rparigoris" <rparigor(at)hotmail.com>
Date: Aug 26, 2020
Hi Group, thank you for answer on using starter for ground on my Rotax 914. The answer is yes I can put an An-4 nut on the other side of bolt. Question: Should I use any anti corrosion goop on my starter ground joint? Something like Al-Ox which is a green goop you use when "tipping" aluminium home wires with copper in wire nut? Or?? Next question is for Ring terminals, PIDG Fast-Ons and D-Sub connectors, should any goop be applied to these connections? CRC 2-56? Or? Thx. Ron P Read this topic online here: http://forums.matronics.com/viewtopic.php?p=498021#498021 ________________________________________________________________________________
Date: Aug 26, 2020
From: "Robert L. Nuckolls, III" <nuckolls.bob(at)aeroelectric.com>
Subject: Re: Anti corrosion on connections?
At 09:38 PM 8/26/2020, you wrote: > >Hi Group, thank you for answer on using starter for ground on my >Rotax 914. The answer is yes I can put an An-4 nut on the other side of bolt. > >Question: Should I use any anti corrosion goop on my starter ground >joint? Something like Al-Ox >which is a green goop you use when "tipping" aluminium home wires >with copper in wire nut? Or?? Nope. A properly torqued fastener provides a gas-tight connection where it matters. The visible surfaces may get pretty ugly with age but INSIDE, it will be just as bright and clean as it was when you made up the joint 10 years ago. >Next question is for Ring terminals, PIDG Fast-Ons and D-Sub connectors, >should any goop be applied to these connections? CRC 2-56? Or? Thx. Ron P Only as local extremes suggest . . . like tropical sea shores or ocean based seaplanes. Then consult your local mechanics for their favorite potions demonstrated to forestall . . . not halt but forestall environmental destruction of the airplane. But for the vast majority of our favorite pleasure machines no special treatments are necessary or useful. Bob . . . ________________________________________________________________________________
Subject: Re: RV10 primary elec sys wiring
From: "airknot" <airkbp(at)gmail.com>
Date: Aug 27, 2020
Dear Gents, Revised my ELA calculations with assumtions: -Comm system is working continiously during flight -To start and taxi phases power consumtion for backupbatteries charging (G5, 2 -Skyview dispalys and IBBS backup battery) was added -Pitot heating works during takeoff, approach and landing phases(icing conditions) -fuel pump works during start, taxi, t/o, landing please, critic me ) https://tinyurl.com/y6q2jrma p.s. feel myself extremely self-doubt... to check all real-life consumptions I must build primary elec system on airplane. and hookup all the devices. or i am not right ( Read this topic online here: http://forums.matronics.com/viewtopic.php?p=498025#498025 ________________________________________________________________________________
From: Rowland Carson <rowlandcarson(at)gmail.com>
Subject: Re: Use 1/4" hole on starter for engine ground?
Date: Aug 27, 2020
On 2020-08-26, at 18:45, rparigoris wrote: > Hi Group I have a Rotax 914 with heavy duty starter. There is a casting at the rear of the starter with an untapped 1/4" hole perhaps 1/2" thick. Is there any reason why I couldn't use this as my only engine ground? Ron - thats what I have done on the Rotax 912ULS in my Europa - see <http://www.rowlandcarson.org.uk/aviation/europa_435/starter_wires_a.php>. It seemed the obvious and sensible choice. in friendship Rowland | Rowland Carson ... that's Rowland with a 'w' ... | http://www.rowlandcarson.org.uk | Skype, Twitter: rowland_carson Facebook: Rowland Carson ________________________________________________________________________________
From: Charlie England <ceengland7(at)gmail.com>
Date: Aug 27, 2020
Subject: Re: RV10 primary elec sys wiring
On Thu, Aug 27, 2020 at 3:47 AM airknot wrote: > > Dear Gents, > > Revised my ELA calculations with assumtions: > -Comm system is working continiously during flight > -To start and taxi phases power consumtion for backupbatteries charging > (G5, 2 -Skyview dispalys and IBBS backup battery) was added > -Pitot heating works during takeoff, approach and landing phases(icing > conditions) > -fuel pump works during start, taxi, t/o, landing > > please, critic me ) > > https://tinyurl.com/y6q2jrma > > p.s. feel myself extremely self-doubt... > > to check all real-life consumptions I must build primary elec system on > airplane. and hookup all the devices. or i am not right ( > > You're doing fine; it takes multiple iterations to debug a design. Go through it line by line, and play 'what if' for every situation you can think of, related to that device, that leaves you able to make choices. If night flight, lights on continuously. If mech fuel pump failure, boost pump on continuously. Etc etc. Go through again, line by line, asking how/when the device uses power, and whether it's significant. The big deal is total energy (power * time-active). Stuff that is almost never active can be ignored as part of your continuous load. Ex: trim, flaps, comm's transmit current draw (which is several times higher than 'idle' current), etc etc. Bob mentioned 'watt-seconds' earlier. If you have only one alternator, then after it fails (assume that it will), then the battery(s) has some number of watt-seconds of energy available in its 'tank'. An example battery might have 12V * 18AH = 260 watt-hours, * 60 minutes = 12960 watt-seconds. (Please check the math...) If, for example, the trim is activated for 2 seconds, it uses 0.15A * 12V = 1.8 watts, * 2 seconds = 3.6 watt-seconds of energy while trimming. You probably couldn't use more than 100 watt-seconds from the battery for trimming, on the entire flight. Point is, it's such a tiny percentage that it can be ignored, unless you have dozens of those tiny items which could become significant if all operated, and all operated continuously. Examples of things that could be tweaked in the spreadsheet (not a line-by-line analysis): The comm can be listed at its lowest current. It only consumes the max while transmitting, which is a tiny fraction of total time. If you're going to fly IFR, the pitot heat should be listed as continuous, because you may be in a marginal icing situation where you can't turn it off. (That's what the 'worst case' thing means.) You could probably zero out the trim current. Like the radio, it's only operating a tiny fraction of the time, and shouldn't draw any current at all while not operating. Ask the mfgr about the oil cooler servo; like the trim servos, it shouldn't draw any current unless operating. Boost pump should be shown as continuous. 'Worst case' of mechanical fuel pump failure during cruise would require running boost until landing. Having fun yet? Charlie ________________________________________________________________________________
Subject: Re: RV10 primary elec sys wiring
From: C&K <yellowduckduo(at)gmail.com>
Date: Aug 27, 2020
Very nice tutorial. If the boost pump is normally off in cruise I'd also ignore that load for an alternator failed scenario, same as for landing lights. Ken On 27/08/2020 9:58 AM, Charlie England wrote: > > > On Thu, Aug 27, 2020 at 3:47 AM airknot > wrote: > > > > > Dear Gents, > > Revised my ELA calculations with assumtions: > -Comm system is working continiously during flight > -To start and taxi phases power consumtion for backupbatteries > charging (G5, 2 -Skyview dispalys and IBBS backup battery) was added > -Pitot heating works during takeoff, approach and landing > phases(icing conditions) > -fuel pump works during start, taxi, t/o, landing > > please, critic me ) > > https://tinyurl.com/y6q2jrma > > p.s. feel myself extremely self-doubt... > > to check all real-life consumptions I must build primary elec > system on airplane. and hookup all the devices. or i am not right ( > > You're doing fine; it takes multiple iterations to debug a design. > > Go through it line by line, and play 'what if' for every situation you > can think of, related to that device, that leaves you able to make > choices. If night flight, lights on continuously. If mech fuel pump > failure, boost pump on continuously. Etc etc. > Go through again, line by line, asking how/when the device uses power, > and whether it's significant. The big deal is total energy (power * > time-active). Stuff that is almost never active can be ignored as part > of your continuous load. Ex: trim, flaps, comm's transmit current draw > (which is several times higher than 'idle' current), etc etc. Bob > mentioned 'watt-seconds' earlier. If you have only one alternator, > then after it fails (assume that it will), then the battery(s) has > some number of watt-seconds of energy available in its 'tank'. An > example battery might have 12V * 18AH = 260 watt-hours, * 60 minutes = > 12960 watt-seconds. (Please check the math...) If, for example, the > trim is activated for 2 seconds, it uses 0.15A * 12V = 1.8 watts, * > 2 seconds = 3.6 watt-seconds of energy while trimming. You probably > couldn't use more than 100 watt-seconds from the battery for trimming, > on the entire flight. Point is, it's such a tiny percentage that it > can be ignored, unless you have dozens of those tiny items which could > become significant if all operated, and all operated continuously. > > Examples of things that could be tweaked in the spreadsheet (not a > line-by-line analysis): > > The comm can be listed at its lowest current. It only consumes the max > while transmitting, which is a tiny fraction of total time. > If you're going to fly IFR, the pitot heat should be listed as > continuous, because you may be in a marginal icing situation where you > can't turn it off. (That's what the 'worst case' thing means.) > You could probably zero out the trim current. Like the radio, it's > only operating a tiny fraction of the time, and shouldn't draw any > current at all while not operating. > Ask the mfgr about the oil cooler servo; like the trim servos, it > shouldn't draw any current unless operating. > Boost pump should be shown as continuous. 'Worst case' of mechanical > fuel pump failure during cruise would require running boost until landing. > Having fun yet? > > Charlie > > ________________________________________________________________________________
Subject: Re: RV10 primary elec sys wiring
From: "user9253" <fransew(at)gmail.com>
Date: Aug 27, 2020
The battery contactor only draws 0.75 amps. See https://www.gigavac.com/sites/default/files/GX11B-GX12B-GX21B-Temp-Considerations.pdf The USB sockets will not draw any current unless something is plugged in. Even then, the 12 volt current draw will be less than half of the 5 volt current (assuming switching power supply) The clock only uses 0.1 amp. See https://iflyei.com/wp-content/uploads/SC-5-OI.pdf The com radio only uses 0.6 amps unless the transmit button is continuously held in. The TCW - SERVO VALVE will not run continuously. Do not count its current. -------- Joe Gores Read this topic online here: http://forums.matronics.com/viewtopic.php?p=498037#498037 ________________________________________________________________________________
Subject: Re: RV10 primary elec sys wiring
From: Bill Watson <Mauledriver(at)nc.rr.com>
Date: Aug 27, 2020
On 8/17/2020 9:41 PM, Robert L. Nuckolls, III wrote: > > With that engine, why are you including a backup > battery? A 8-10A standby alternator on a vacuum > pump pad offers UNLIMITED endurance with about > a 4 pound penalty. I go out of my way to avoid > adding batteries to a design over and above > those normally expected to start engines and > back up the primary alternator. > > Do you not PLAN to conduct periodic airworthiness > checks on the main battery? > > Bob . . . > OP - I too have an RV10 with the 2 mag engine and a 3 display GRT panel. I'll just add a couple of observations based on 9+ years and 1200+ hours of IFR intensive operation. Bobs point above is dead on target for RV10 folks in particular - I have a 40 amp main alt and a 20amp alt on the vacuum pad (B&C externally controlled). That 2nd amp removes the main failure point that threatens to actually shorten a flight and force a alternative landing. I had a number of backup batteries connected to various avionic components but found that it was a challenge maintaining them to the proper standard for true reliance in case of failure. They've all come out and I simply focus on my main battery. 2 Alternators and 1 battery is a real sweet spot. But that's not what i have. I installed a Z-14 -2 alts, 2 batts, 2 buses, also known by Bob as 'everything and the kitchen sink' electrical system. It's overkill by many measures but I love it. However I'd recommend that the 'Z' with 2 alts and 1 battery is the optimal config from a price-performance perspective. And like Bob said, get rid of those other backup batteries. It is commonly said that: > Another point is to protect equipment during engine start Absolutely NO equipment needs to be protected during engine start. There is no such thing as voltage spikes during starting. That is an old wive's tale. Are you going to believe rumors based on 1950's technology or Bob N. who has written the book on aircraft electrical systems? True enuff I suppose but my panel did suffer from low voltage sagging during some starts (cold wx combined with panel running before start with a less than fresh battery and no extra avionics switches to manage total power reqmts). It now simply reboots the displays if the voltage is too low but earlier it was possible to corrupt the software if the load was abnormally interrupted. A SW bug for sure but it happened. Those displays, which come on with my master switch, are amp hogs. So I added a now discontinued product from TCW - IPS (intelligent power stabilizer) which has kept me from losing my displays for 5 years now. A well managed, big battery combined with modern avionics should avoid this issue entirely. So I'd suggest adding a small alternator on the vacuum pad, eliminating the extra batteries and not worrying about the 60 amp alternator's ability to handle anything you throw at it. It still has a battery behind it - the most reliable component in the system. The Z designs are really well crafted and don't require modification to use them successfully. BTW, the Odyssey AGM batteries are really easy to manage and maintain with a volt meter. Read the owners manual and follow it to the letter: 1) make sure your charging system is set to run between 14.1 and 14.7 volts 2) Use a proper charger if and when needed, 3) Keep it fully charged when inactive, 4) if 24hours after charging, the voltage is less than 12.5 volts (75% capacity), replace it. You might do that at 12.6 volts. Bill "not reading the owners manual cost me 3 or 4 batteries before I became fully literate" Watson -- This email has been checked for viruses by Avast antivirus software. https://www.avast.com/antivirus ________________________________________________________________________________
Date: Aug 27, 2020
From: "Robert L. Nuckolls, III" <nuckolls.bob(at)aeroelectric.com>
Subject: Re: Use 1/4" hole on starter for engine ground?
At 05:40 AM 8/27/2020, you wrote: > > >On 2020-08-26, at 18:45, rparigoris wrote: > > >> Hi Group I have a Rotax 914 with heavy duty starter. There is > a casting at > >> the rear of the starter with an untapped 1/4" hole perhaps 1/2" thick. > >> Is there any reason why I couldn't use this as my only engine ground? > > > Ron - Tthat's what I have done on the Rotax 912ULS in my Europa > > see > <http://www.rowlandcarson.org.uk/aviation/europa_435/starter_wires_a.php>. > > It seemed the obvious and sensible choice. Rowland, thank you for posting this image . . . for two reasons. First, I was unaware of the existence of that boss on the starter end bell casting . . . you're right, that's a really good place to get a good electrical connection to crankcase! Second, I have concerns for the mechanical integrity of this installation as photographed. I perceived that the fat wires are probably $high$ 'aircraft wire' . . . nice stuff . . . stiff as a board. The wire has no mechanical support TO THE ENGINE in close proximity to the bolted joint. The combination of stiff, heavy wire and no support imparts vibrational stresses to the terminal flag. This is the CRANKCASE GROUND . . . usually a very critical wire in the system. We normally try to make that connection with super flexible conductors that are (1) resistance to flexure failures, (2) does not impart strong flexure forces to the terminal. Alternatively, consider using welding cable. Again, were looking for a bazillion strands of copper cat-hair for conductors and flexible- friendly installation. This article suggests a method for soldering terminals onto fat wires https://tinyurl.com/ct36xen . . . I need to upgrade that article to suggest welding cable with double-wall heat shrink as opposed to the stiff-stuff. https://tinyurl.com/yysahtcf Recommend you consider upgrading the two strands of fat-wire that come off the back of your engine. Welding cable with heat shrink dressing won't require additional mechanical support and you'll enjoy working with this stuff. But thanks for sharing that picture . . . that's indeed an excellent crankcase connection point. Bob . . ________________________________________________________________________________
Subject: Re: Use 1/4" hole on starter for engine ground?
From: "rparigoris" <rparigor(at)hotmail.com>
Date: Aug 27, 2020
Hi Bob "I need to upgrade that article to suggest welding cable with double-wall heat shrink as opposed to the stiff-stuff" I plan on using B&C #4 Super-Flex Battery Leads going from firewall to engine x2: https://bandc.com/product/super-flex-battery-lead-red-raw-material/ when you suggest using double wall heat shrink, are you suggesting to use only at terminal lugs or the entire length of cable?? What type of heat shrink are you suggesting? Thx. Ron Read this topic online here: http://forums.matronics.com/viewtopic.php?p=498041#498041 ________________________________________________________________________________
Subject: Z13-8 main alternator troubleshooting
From: "rvtach" <rvtach(at)msn.com>
Date: Aug 27, 2020
Hello expert electron herders- I'm just finishing the phase one flight test on my O-360 powered RV-7A wired per Z13-8 and I'm trying to troubleshoot a main alternator problem which is that the alternator isn't putting out any voltage. I replaced the voltage regulator (at less than $20 it seemed like a reasonable first step) and still no power from the B&C 60 amp alternator. Due to some poor planning on my part the VR is in a somewhat inaccessible location which may have contributed to me hooking up the IASF leads on the replacement VR incorrectly- I connected the "S" lead (which is jumpered to the "A" lead) to the "I" tab. Wonder if that may have fried the voltage regulator? At the hangar today I took some measurements and here's what I found: Good continuity from the main bus, through the D/C power master switch to the VR-166 voltage regulator A/S terminal. 0.5 ohms. Good continuity from the VR to the Alternator Field plug. 0 ohms. Good continuity from the VR case to Firewall forest of tabs ground. 0 ohms. With Master switch on and battery voltage at 12.5V there is no voltage at the alternator field plug. It seems like there should be something on the order of 12V at the field plug, no? Any suggestions before I just go out and buy a new VR-166? A final note- I truly appreciate all the information I've gleaned from this forum over the past 10+ years. Because of everything I've learned from Bob and all the rest of you guys this is the only electrical problem I've had- everything else has been perfect. Beyond my hopes!! Cheers- Jim -------- Jim McChesney Tucson, AZ RV-7A Finishing Kit Read this topic online here: http://forums.matronics.com/viewtopic.php?p=498043#498043 ________________________________________________________________________________
From: Rowland Carson <rowlandcarson(at)gmail.com>
Subject: Re: Use 1/4" hole on starter for engine ground?
Date: Aug 28, 2020
On 2020-08-27, at 20:07, Robert L. Nuckolls, III wrote: > I have concerns for the mechanical > integrity of this installation as photographed. > > I perceived that the fat wires are probably > $high$ 'aircraft wire' . . . nice stuff . . . > stiff as a board. The wire has no mechanical > support TO THE ENGINE in close proximity to > the bolted joint. > > The combination of stiff, heavy wire and > no support imparts vibrational stresses to > the terminal flag. > > This is the CRANKCASE GROUND . . . usually > a very critical wire in the system. > > We normally try to make that connection > with super flexible conductors that are > (1) resistance to flexure failures, (2) > does not impart strong flexure forces to the > terminal. Bob - thank you so much for pointing out my error. The wire is indeed 4AWG Nyvin - I received a length of it as a gift from a fellow builder. I guess I was so happy to get such high-spec stuff free that I didnt consider the problems its stiffness would cause in a connexion subject to vibration. Like most things, its really obvious when I actually THINK about it! As you probably know, Rotax engines are famous for shaking like a terrier with a rat on both startup and shut-down, so I should have engaged brain a bit sooner. Ill seek a UK supplier of welding cable in short lengths to replace the 2 fat cables to the starter. The terminals on my present wires are crimped using a hand hydraulic crimper - would that still be OK for welding cable, or is it important to solder the joints on such wire terminations? in friendship Rowland | Rowland Carson ... that's Rowland with a 'w' ... | http://www.rowlandcarson.org.uk | Skype, Twitter: rowland_carson Facebook: Rowland Carson ________________________________________________________________________________
Date: Aug 28, 2020
From: "Robert L. Nuckolls, III" <nuckolls.bob(at)aeroelectric.com>
Subject: Re: Use 1/4" hole on starter for engine ground?
>I=99ll seek a UK supplier of welding cable in >short lengths to replace the 2 fat cables to the >starter. The terminals on my present wires are >crimped using a hand hydraulic crimper - would >that still be OK for welding cable, or is it >important to solder the joints on such wire terminations? Here's one option: https://tinyurl.com/y535olwm Crimp or solder . . . either process skillfully applied produces the holy grail of wire termination - gas tightness. Bob . . . ________________________________________________________________________________
From: Ken Ryan <keninalaska(at)gmail.com>
Date: Aug 28, 2020
Subject: Re: Use 1/4" hole on starter for engine ground?
Is there any real advantage to connecting the engine ground close to the starter motor, rather than somewhere else on the engine block, or is the advantage more theoretical? Wouldn't factors other than resistance through the engine block, such as convenience and access, be more important considerations? On Thu, Aug 27, 2020 at 11:58 PM Rowland Carson wrote: > rowlandcarson(at)gmail.com> > > On 2020-08-27, at 20:07, Robert L. Nuckolls, III < > nuckolls.bob(at)aeroelectric.com> wrote: > > > I have concerns for the mechanical > > integrity of this installation as photographed. > > > > I perceived that the fat wires are probably > > $high$ 'aircraft wire' . . . nice stuff . . . > > stiff as a board. The wire has no mechanical > > support TO THE ENGINE in close proximity to > > the bolted joint. > > > > The combination of stiff, heavy wire and > > no support imparts vibrational stresses to > > the terminal flag. > > > > This is the CRANKCASE GROUND . . . usually > > a very critical wire in the system. > > > > We normally try to make that connection > > with super flexible conductors that are > > (1) resistance to flexure failures, (2) > > does not impart strong flexure forces to the > > terminal. > > > Bob - thank you so much for pointing out my error. The wire is indeed 4AW G > Nyvin - I received a length of it as a gift from a fellow builder. I gues s > I was so happy to get such high-spec stuff free that I didn=99t con sider the > problems its stiffness would cause in a connexion subject to vibration. > Like most things, it=99s really obvious when I actually THINK about it! As > you probably know, Rotax engines are famous for shaking like a terrier wi th > a rat on both startup and shut-down, so I should have engaged brain a bit > sooner. > > I=99ll seek a UK supplier of welding cable in short lengths to repl ace the 2 > fat cables to the starter. The terminals on my present wires are crimped > using a hand hydraulic crimper - would that still be OK for welding cable , > or is it important to solder the joints on such wire terminations? > > in friendship > > Rowland > > > | Rowland Carson ... that's Rowland with a 'w' ... > | http://www.rowlandcarson.org.uk > | Skype, Twitter: rowland_carson Facebook: Rowland Carson > > =========== =========== =========== =========== =========== > > ________________________________________________________________________________
Subject: Re: Use 1/4" hole on starter for engine ground?
From: Charlie England <ceengland7(at)gmail.com>
Date: Aug 28, 2020
The advantage I see is that there are fewer 'joints' between the starter winding and that ground return attach point. Not likely to be a 'real world' problem, but tying directly to the starter does eliminate the sum of resistance for all those mating surfaces, which likely have some form of non-conductive sealant used. Again, not likely to be a real world issue. Charlie On 8/28/2020 9:18 AM, Ken Ryan wrote: > Is there any real advantage to connecting the engine ground close to > the starter motor, rather than somewhere else on the engine block, or > is the advantage more theoretical? Wouldn't factors other than > resistance through the engine block, such as convenience and access, > be more important considerations? > > On Thu, Aug 27, 2020 at 11:58 PM Rowland Carson > > wrote: > > > > > On 2020-08-27, at 20:07, Robert L. Nuckolls, III > > wrote: > > > I have concerns for the mechanical > > integrity of this installation as photographed. > > > > I perceived that the fat wires are probably > > $high$ 'aircraft wire' . . . nice stuff . . . > > stiff as a board. The wire has no mechanical > > support TO THE ENGINE in close proximity to > > the bolted joint. > > > > The combination of stiff, heavy wire and > > no support imparts vibrational stresses to > > the terminal flag. > > > > This is the CRANKCASE GROUND . . . usually > > a very critical wire in the system. > > > > We normally try to make that connection > > with super flexible conductors that are > > (1) resistance to flexure failures, (2) > > does not impart strong flexure forces to the > > terminal. > > > Bob - thank you so much for pointing out my error. The wire is > indeed 4AWG Nyvin - I received a length of it as a gift from a > fellow builder. I guess I was so happy to get such high-spec stuff > free that I didnt consider the problems its stiffness would cause > in a connexion subject to vibration. Like most things, its really > obvious when I actually THINK about it! As you probably know, > Rotax engines are famous for shaking like a terrier with a rat on > both startup and shut-down, so I should have engaged brain a bit > sooner. > > Ill seek a UK supplier of welding cable in short lengths to > replace the 2 fat cables to the starter. The terminals on my > present wires are crimped using a hand hydraulic crimper - would > that still be OK for welding cable, or is it important to solder > the joints on such wire terminations? > > in friendship > > Rowland > -- This email has been checked for viruses by Avast antivirus software. https://www.avast.com/antivirus ________________________________________________________________________________
Date: Aug 28, 2020
From: "Robert L. Nuckolls, III" <nuckolls.bob(at)aeroelectric.com>
Subject: Re: Use 1/4" hole on starter for engine ground?
At 09:18 AM 8/28/2020, you wrote: >Is there any real advantage to connecting the engine ground close to >the starter motor, rather than somewhere else on the engine block, >or is the advantage more theoretical? Wouldn't factors other than >resistance through the engine block, such as convenience and access, >be more important considerations? Good question. Absolutely. For example, do you want your crankcase ground to run to the other end of the engine to access a bolt near the starter on a Lycoming or would some fastener local to the firewall ground stud be more appropriate? Variation in path resistance across the engine would be difficult to measure much less drive an architecture decision. Short and sweet is almost always best. Bob . . . ________________________________________________________________________________
Date: Aug 28, 2020
From: "Robert L. Nuckolls, III" <nuckolls.bob(at)aeroelectric.com>
Subject: Re: Use 1/4" hole on starter for engine ground?
At 10:38 AM 8/28/2020, you wrote: >The advantage I see is that there are fewer 'joints' between the >starter winding and that ground return attach point. Not likely to >be a 'real world' problem, but tying directly to the starter does >eliminate the sum of resistance for all those mating surfaces, which >likely have some form of non-conductive sealant used. Again, not >likely to be a real world issue. Yeah, if one could measure point-to-point resistances across the engine, there would no doubt be some variations due to factors you've cited . . . but looking for the the 'best' path would not be easy or productive. Bob . . . ________________________________________________________________________________
Subject: Re: Use 1/4" hole on starter for engine ground?
From: "user9253" <fransew(at)gmail.com>
Date: Aug 28, 2020
Connecting the ground cable directly to the starter is theoretically better, but connecting to the engine block is good enough. But if the starter location is more convenient and uses less cable, then why not? -------- Joe Gores Read this topic online here: http://forums.matronics.com/viewtopic.php?p=498054#498054 ________________________________________________________________________________
Subject: Double wall heat shrink for welding cable?
From: "rparigoris" <rparigor(at)hotmail.com>
Date: Aug 28, 2020
Hi Bob you wrote this on a replyand I don't understand it "I need to upgrade that article to suggest welding cable with double-wall heat shrink as opposed to the stiff-stuff" I plan on using B&C #4 Super-Flex Battery Leads going from firewall to engine x2: https://bandc.com/product/super-flex-battery-lead-red-raw-material/ when you suggest using double wall heat shrink, are you suggesting to use double wall heat shrink tubing only at terminal lugs or for the entire length of the welding cable?? What type of heat shrink is your suggestion? Thx. Ron P. Read this topic online here: http://forums.matronics.com/viewtopic.php?p=498056#498056 ________________________________________________________________________________
Date: Aug 28, 2020
From: "Robert L. Nuckolls, III" <nuckolls.bob(at)aeroelectric.com>
Subject: Re: Z13-8 main alternator troubleshooting
> >Any suggestions before I just go out and buy a new VR-166? Go ahead and get the new regulator then rig this test setup. https://tinyurl.com/yxhknfhk Conduct test as written. This will narrow down your field of investigation by half. Bob . . . ________________________________________________________________________________
Date: Aug 28, 2020
From: "Robert L. Nuckolls, III" <nuckolls.bob(at)aeroelectric.com>
Subject: Re: Double wall heat shrink for welding cable?
At 02:05 PM 8/28/2020, you wrote: > >Hi Bob you wrote this on a replyand I don't understand it "I need to >upgrade that article to suggest welding cable with double-wall heat >shrink as opposed to the stiff-stuff" >I plan on using B&C #4 Super-Flex Battery Leads going from firewall >to engine x2: >https://bandc.com/product/super-flex-battery-lead-red-raw-material/ >when you suggest using double wall heat shrink, are you suggesting >to use double wall heat shrink tubing only at terminal lugs or for >the entire length of the welding cable?? What type of heat shrink is >your suggestion? Thx. Ron P. Only at the ends as illustrated in my article. Here's a good example: https://tinyurl.com/y2eyw8k3 3/4" unshrunk with 3:1 shrink ratio. Short pieces fit easily over your crimped/soldered terminal but will definitely shrink to a good fit with sealant oozing out both ends. The high shrink ratio makes the finished shrink wall thicker for improved flexure support . . . the sealant keeps the moisture out. Good stuff Maynard! Bob . . . ________________________________________________________________________________
From: William Daniell <wdaniell.longport(at)gmail.com>
Date: Aug 28, 2020
Subject: Rotax ignition coil faston size
Does anyone know the size of the faston for the rotax ignition connectors? Thanks Willl William Daniell +1 786 878 0246 ________________________________________________________________________________
Subject: Re: Rotax ignition coil faston size
From: "rparigoris" <rparigor(at)hotmail.com>
Date: Aug 29, 2020
Hi William I have a 914 and am close to installing the grounding cut off pins on the ignition connector. It's a round crimp on pin that has a rubber seal. I purchased an extra set from Rotax to do a practice crimp or two. Is this the pin that you want a dimension for? Rotax wants you to use shielded wire and shield it at both ends. If this is not the connector, where is the faston located? Ron P. Read this topic online here: http://forums.matronics.com/viewtopic.php?p=498063#498063 ________________________________________________________________________________
Date: Aug 30, 2020
From: "Robert L. Nuckolls, III" <nuckolls.bob(at)aeroelectric.com>
Subject: Revmaster 'dual' alterantors
I've had a few weeks (and road trips) to apply some 'asphalt engineering' effort to a combination of threads discussing relative fragility of the Revmaster 'dual' alternators. The latest thread explored the notion that changing from SLVA to LiFePO4 was much more likely to burn an alternator winding. This is not a new topic here on the List . . . found a few other postings on the same problem. Without talking to the designers, it's difficult to KNOW the original thinking behind the design. We also cannot know what efforts the factory has applied to this issue over the years. But I think we're on pretty firm ground to assert that the current design is marginal with respect to thermal robustness. Alternator windings should be as reliable as propeller bolts. You might smoke some regulators, trash some batteries, find yourself wanting for a few more amps of output . . . but suffering destruction of the windings suggests the alternator is a weak link in the system. Based on the drawing that Dan supplied a few weeks ago it seems that the designers intended that only one of the two windings be used at the same time. I'm guessing that this is a "normal-and-spare" design philosophy. Therefore, any time a winding fails due to exceedance of thermal limits, the other winding was off line. We don't know the internal configuration of the recommended rectifier/regulator but it's almost certain to included silicon controlled rectifier and one diode in series on each conduction half- cycle. That's 3, silicon junctions in series that carry alternator output current. My current hypothesis suggests that it would be much better to use BOTH windings all the time. Reduce the current in each winding by 50% or more. It seems better to have one configuration that's bulletproof than two relatively fragile configurations that 'back each other up'. Here's the line of reasoning supporting this design goal. Recall the bits-and-pieces of design? (1) properties of materials, (2) management of energy and (3)refinement of process. In this case, our weak link seems to center on an energy management issue. Some copper windings are heating past practical operating limits. This can be either an insulation failure, wire failure or both. Properties of Materials: We know that copper has a pretty significant positive resistance coefficient for temperature. We observed this in the temperature vs. current observations in battery contactors: https://tinyurl.com/mpcgp3t https://tinyurl.com/k6bwdqo We also considered the physics of why an overloaded wire tends to burn open at the center of a free-air span. As copper heats up, resistance goes up, voltage drop goes up, dissipated energy goes up, temperature rises some more . . . and you can see where this is headed. Take this tid-bit of knowledge about copper wire and consider how many times you've observed or heard of the windings or lead wires of any system failing due to overheat. I've seen some windings burn up on alternators for reasons OTHER than poor thermal management. I've never had a winding burn on my car. I've had one short and start popping fuses . . . but I doubt that it was due to burned insulation or melted wires! The point is, ANY configuration that demonstrates repeated failure to do open wires or cooked insulation is MARGINAL at best; hazardous at worst. Okay, how to reduce the load in Revmaster alternator wires . . . hopefully without compromising ability to deliver ENERGY into the system. Here's the energy management consideration: We know that PM alternators have a checkered history of performance . . . but mostly due to stone simple regulators that DO NOT offer active current limiting. Furhter, many versions use a full wave bridge consisting of two pairs of junction diode and silicon controlled rectifiers. Alternator output current flows through two of these devices with total drop on the order of 2 volts. 2 volts out of a 14v system is a substantial proportion of energy . . . something on the order of 15 to 25 watts that needs to be managed in those cute little castings . . . but that's another story. Harking back to the earliest days of my studies in electron herding, ac rectifiers were vacuum tubes (not junctions) and ac power sources were transformers plugged into the wall (not spinning magnets). Except for the systems with very low energy requirements (table top radios), the rectifiers were dual diodes driven from either side of a center tapped transformer secondary. This configuration had some profound effects in thermal management. EACH diode carries 1/2 the total current. These devices have considerable resistance . . . watts I(squared) x R so if you reduce current by 1/2, watts goes down to 1/4. Same thing happens in the transformer secondary . . . the secondary wire size can be made smaller in a trade off between energy lost and transformer size. Refinement of process: So take a peek at the simplified proposal diagram. Hook the two Revmaster windings in series-aiding and bring out the center-tap. Connect in full wave configuration with only TWO junction rectifiers. Feed this resulting energy off to a filter capacitor size to be determined. This will give us a 'supply voltage' of some value ideally adjust to about 20VDC at cruise RPM and alternators full load design point. This voltage is going to be all over the place depending on load and engine rpm . . . but that's the nature of the PM alternator. Now, let's power condition that energy with a constant current, constant voltage, switch-mode regulator. NOW we have a BIG difference in performance: Voltage is adjustable and controlled by active electronics. Current is adjustable and limited by active electronics. I suggest this configuration offers significant improvement in Revmaster's engine driven power source. Energy being delivered to the regulator is at significantly higher than the output set point, CURRENT in from the alternator will be LOWER than current delivered by the regulator. Further, each winding works 1/2 of the time so it follows that heating effects on each winding are much reduced from the current configuration. Silicon junctions in the rectifier are significantly reduced off-setting some new losses introduced by the electronic CCCV regulator. It seems that we could craft an engine driven power system that almost never fails in exchange for one that fails too many times and for the wrong reasons. I'm tied up in some house remodeling and we've added another grandson to the population of Medicine Lodge Jr High School . . . so refinement of this idea will be slow. But I'm building an alternator drive stand, power supply and load bank out in the mess-making shop. B&C has provided me with both PM and wound-field test articles. I have a couple of those CCCV regulators that we discussed on hand right now. They may not be the right size in all respects but satisfactory for proof-of-concept experiments. I already have some LiFePO4 and SLVA batteries on hand. The pieces are coming together. Comments and considered critical review are most welcome. Bob . . . ________________________________________________________________________________
Subject: Re: Revmaster 'dual' alterantors
From: "user9253" <fransew(at)gmail.com>
Date: Aug 31, 2020
Using both windings is a good idea. What about the phase angle between the two windings? Do we know what it is? Suppose the two windings are 90 degrees out of phase or some other unknown angle. How will that affect the proposed circuit? Will the peak AC voltage be higher? Do capacitors help very much to smooth the DC voltage in a power circuit? It would help to have a Revmaster alternator to play with. -------- Joe Gores Read this topic online here: http://forums.matronics.com/viewtopic.php?p=498085#498085 ________________________________________________________________________________
Date: Aug 31, 2020
From: "Robert L. Nuckolls, III" <nuckolls.bob(at)aeroelectric.com>
Subject: Revmaster installation manual?
Does anyone have a .pdf copy of a Revmaster installation manual for 2300, 3000 engines . . . or both? If there's a download link for either . . . that would be fine too. Thanks! Bob . . . ________________________________________________________________________________
Date: Aug 31, 2020
From: "Robert L. Nuckolls, III" <nuckolls.bob(at)aeroelectric.com>
Subject: Re: Revmaster 'dual' alterantors
At 10:39 AM 8/31/2020, you wrote: > >Using both windings is a good idea. >What about the phase angle between the two windings? Do we know what it is? No . . . >Suppose the two windings are 90 degrees out of phase or some other >unknown angle. >How will that affect the proposed circuit? Will the peak AC voltage >be higher? Good question. I doubt they are polyphase . . . you gotta go to a lot of mechanical fuss to achieve it. It's a function of stator/magnet ring geometry. They might be different but it's doubtful. Issue 80 of Contact! magazine has some nice pictures of the stator arrangement: http://www.contactmagazine.com/Issue80/Issue-80L.pdf >Do capacitors help very much to smooth the DC voltage in a power circuit? They can . . . it's all about current/capacitance ratio versus frequency. If a capacitor is necessary, it will be dictated by the CCCV down converter tolerance for ripple. >It would help to have a Revmaster alternator to play with. You got that right! Bob . . . ________________________________________________________________________________
Date: Aug 31, 2020
From: Ernest Christley <echristley(at)att.net>
Subject: Re: Revmaster 'dual' alterantors
Would this scheme work for other PM generators?=C2- I'm using a 32-amp g enerator from a Harley-Davidson.=C2- The 74518-88 model regulator for the se seems to be as plentiful as light bulbs, but I can't find any good detai ls of what the internals look like.=C2- It has two wire input from the st ator, and one wire out to the battery positive.=C2- The case serves as gr ound. No means of control or way to indicate failure, prompting me to build my own. uckolls.bob(at)aeroelectric.com> wrote: I've had a few weeks (and road trips) to apply some 'asphalt engineering' effort to a combination of threads discussing relative fragility of the Revmaster 'dual' alternators. The latest thread explored the notion that changing from SLVA to LiFePO4 was much more likely to burn an alternator winding. This is not a new topic here on the List . . . found a few other postings on the same problem. Without talking to the=C2- designers, it's difficult to KNOW the original thinking behind the design. We also cannot know what efforts the factory has applied to this issue over the years. But I think we're on pretty firm ground to assert that the current design is marginal with respect to thermal robustness. Alternator windings should be as reliable as propeller bolts. You might smoke some regulators, trash some batteries, find yourself wanting for a few more amps of output . . . but suffering destruction of the windings suggests the alternator is a weak link in the system. Based on the drawing that Dan supplied a few weeks ago it seems that the designers intended that only one of the two windings be used at the same time. I'm guessing that this is a "normal-and-spare" design philosophy. Therefore, any time a winding fails due to exceedance of thermal limits, the other winding was off line. We don't know the internal configuration of the recommended rectifier/regulator but it's almost certain to included silicon controlled rectifier and one diode in series on each conduction half- cycle. That's 3, silicon junctions in series that carry alternator output current. My current hypothesis suggests that it would be much better to use BOTH windings all the time. Reduce the current in each winding by 50% or more. It seems better to have one configuration that's bulletproof than two relatively fragile configurations that 'back each other up'. Here's the line of reasoning supporting this design goal. Recall the bits-and-pieces of design? (1) properties of materials, (2) management of energy and (3)refinement of process. In this case, our weak link seems to center on an energy management issue. Some copper windings are heating past practical operating limits. This can be either an insulation failure, wire failure or both. Properties of Materials: We know that copper has a pretty significant positive resistance coefficient for temperature. We observed this in the temperature vs. current observations in battery contactors: https://tinyurl.com/mpcgp3t https://tinyurl.com/k6bwdqo We also considered the physics of why an overloaded wire tends to burn open at the center of a free-air span. As copper heats up, resistance goes up, voltage drop goes up, dissipated energy goes up, temperature rises some more . . . and you can see where this is headed. Take this tid-bit of knowledge about copper wire and consider how many times you've observed or heard of the windings or lead wires of any system failing due to overheat. I've seen some windings burn up on alternators for reasons OTHER than poor thermal management. I've never had a winding burn on my car. I've had one short and start popping fuses . . . but I doubt that it was due to burned insulation or melted wires! The point is, ANY configuration that demonstrates repeated failure to do open wires or cooked insulation is MARGINAL at best; hazardous at worst. Okay, how to reduce the load in Revmaster alternator wires . . . hopefully without compromising ability to deliver ENERGY into the system. Here's the energy management consideration: We know that PM alternators have a checkered history of performance . . . but mostly due to stone simple regulators that DO NOT offer active current limiting. Furhter, many versions use a full wave bridge consisting of two pairs of junction diode and silicon controlled rectifiers. Alternator output current flows through two of these devices with total drop on the order of 2 volts. 2 volts out of a 14v system is a substantial proportion of energy . . . something on the order of 15 to 25 watts that needs to be managed in those cute little castings . . . but that's another story. Harking back to the earliest days of my studies in electron herding, ac rectifiers were vacuum tubes (not junctions) and ac power sources were transformers plugged into the wall (not spinning magnets). Except for the systems with very low energy requirements (table top radios), the rectifiers were dual diodes driven from either side of a center tapped transformer secondary. This configuration had some profound effects in thermal management. EACH diode carries 1/2 the total current. These devices have considerable resistance . . . watts I(squared) x R so if you reduce current by 1/2, watts goes down to 1/4.=C2- Same thing happens in the transformer secondary . . . the secondary wire size can be made smaller in a trade off between energy lost and transformer size. Refinement of process: So take a peek at the simplified proposal diagram. Hook the two Revmaster windings in series-aiding and bring out the center-tap. Connect in full wave configuration with only TWO junction rectifiers. Feed this resulting energy off to a filter capacitor size to be determined. This will give us a 'supply voltage' of some value ideally adjust to about 20VDC at cruise RPM and alternators full load design point. This voltage is going to be all over the place depending on load and engine rpm . . . but that's the nature of the PM alternator. Now, let's power condition that energy with a constant current, constant voltage, switch-mode regulator. NOW we have a BIG difference in performance: Voltage is adjustable and controlled by active electronics. Current is adjustable and limited by active electronics. I suggest this configuration offers significant improvement in Revmaster's engine driven power source. Energy being delivered to the regulator is at significantly higher than the output set point, CURRENT in from the alternator will be LOWER than current delivered by the regulator. Further, each winding works 1/2 of the time so it follows that heating effects on each winding are much reduced from the current configuration. Silicon junctions in the rectifier are significantly reduced off-setting some new losses introduced by the electronic CCCV regulator. It seems that we could craft an engine driven power system that almost never fails in exchange for one that fails too many times and for the wrong reasons. I'm tied up in some house remodeling and we've added another grandson to the population of Medicine Lodge Jr High School . . . so refinement of this idea will be slow. But I'm building an alternator drive stand, power supply and load bank out in the mess-making shop. B&C has provided me with both PM and wound-field test articles. I have a couple of those CCCV regulators that we discussed on hand right now. They may not be the right size in all respects but satisfactory for proof-of-concept experiments. I already have some LiFePO4 and SLVA batteries on hand. The pieces are coming together. Comments and considered critical review are most welcome. =C2- Bob . . . ________________________________________________________________________________
Subject: Re: Revmaster installation manual?
From: "user9253" <fransew(at)gmail.com>
Date: Aug 31, 2020
Evidently Revmaster offers a 30 amp alternator on new engines. http://revmasteraviation.com/?tag=cdi-ignition - This is not an installation manual, but is a description of the Revmaster engine that includes a picture of the alternator coils and a brief description. From CONTACT Magazine. http://www.revmasteraviation.com/wp-content/uploads/2017/07/Revmaster-R-2300.pdf -------- Joe Gores Read this topic online here: http://forums.matronics.com/viewtopic.php?p=498089#498089 ________________________________________________________________________________
From: "rasmussenRE" <rasmussenre(at)gmail.com>
Subject: Z-11 with 1 slick mag and 1 e-mag
Date: Aug 31, 2020
Hi Bob, Using Z-11 for my RV8 build. Z-11 depicts two traditional mags, and there are several different switch configurations in the other Z diagrams that have one mag and one e-mag. Could you tell me which you recommend? Thanks for all you do. Regards, Robert ________________________________________________________________________________
From: Sebastien <cluros(at)gmail.com>
Date: Aug 31, 2020
Subject: Brownout Step Up Converter
My friend has a Harmon Rocket with a GRT EFIS. During start the voltage sags just enough to mess up the GRT so that it usually needs a reset after start. Was there ever consensus on a good ebay dc-dc step up converter? I'm thinking of ordering this one: https://www.ebay.ca/itm/5A-DC-DC-Boost-Step-up-Volt-Converter-Power-Supply-Module-5V-9V-12V-19V-24V-36V/192243452240 The EFIS needs 1A so this should have no problem keeping up. Any thoughts? ________________________________________________________________________________
Subject: Re: Brownout Step Up Converter
From: "user9253" <fransew(at)gmail.com>
Date: Aug 31, 2020
I don't know if there was ever a recommendation as to which DC-DC converter to buy, but there are some that cost between $5 and $10 that have a copper coil and look impressive. Beware of exaggerated ampacity claims. -------- Joe Gores Read this topic online here: http://forums.matronics.com/viewtopic.php?p=498092#498092 ________________________________________________________________________________
Date: Aug 31, 2020
From: "Robert L. Nuckolls, III" <nuckolls.bob(at)aeroelectric.com>
Subject: Re: Brownout Step Up Converter
At 03:13 PM 8/31/2020, you wrote: >My friend has a Harmon Rocket with a GRT EFIS. >During start the voltage sags just enough to >mess up the GRT so that it usually needs a reset >after start. Was there ever consensus on a good >ebay dc-dc step up converter? I'm thinking of ordering this one: > >https://www.ebay.ca/itm/5A-DC-DC- Boost-Step-up-Volt-Converter-Power-Supply-Module-5V-9V-12V-19V-24V-36V/19224 3452240=C2 >=C2 > >The EFIS needs 1A so this should have no problem keeping up. Any thoughts? email a shipping address and I'll send you one to try. I've not had time to characterize these on the test bench yet. But if we're picking one based on advertised performance, this one is as good as any. Bob . . . ________________________________________________________________________________
Date: Aug 31, 2020
From: "Robert L. Nuckolls, III" <nuckolls.bob(at)aeroelectric.com>
Subject: Re: Z-11 with 1 slick mag and 1 e-mag
At 12:38 PM 8/31/2020, you wrote: >Hi Bob, > >Using Z-11 for my RV8 build. Z-11 depicts two traditional mags, and >there are several different switch configurations in the other Z >diagrams that have one mag and one e-mag. Could you tell me which >you recommend? Thanks for all you do. > >Regards, > >Robert I've not reviewed the latest recommendations by Emagair for their products. Are you using toggle switches? Wire the Mag per Z-11, the eMag per manufacturer's instructions. Which model of eMag are you installing? Bob . . . ________________________________________________________________________________
Date: Aug 31, 2020
From: "Robert L. Nuckolls, III" <nuckolls.bob(at)aeroelectric.com>
Subject: Harley-Davidson 32A r/r
At 11:29 AM 8/31/2020, you wrote: >Would this scheme work for other PM generators? I'm using a 32-amp >generator from a Harley-Davidson. The 74518-88 model regulator for >these seems to be as plentiful as light bulbs, but I can't find any >good details of what the internals look like. It has two wire input >from the stator, and one wire out to the battery positive. The case >serves as ground. No means of control or way to indicate failure, >prompting me to build my own. I've got no reason to believe that the most ROBUST of PM rectifier/regulators would not work with the smallest of PM alternators. You can control ANY rectifier/regulator with a relay in one of the AC leads out of the alternator . . . as depicted in many AEC drawings. LOW Volts warning is your primary failure warning. 'Real' low volts warning is built into B&C regulators . . . most other 'warnings' are driven by less than elegant sensing systems (including the venerable VR166). I would judge your probability of success to be high. Bob . . . ________________________________________________________________________________
From: Paul Eckenroth <N509RV(at)eckenroth.com>
Date: Aug 31, 2020
Subject: Re: Revmaster installation manual?
Bob Included is the section from the Revmaster manual that deals with the generator and CDI ignition. I'm looking forward to more from you on the Revmaster problem so I can order a new stator and rewire with confidence that the second time around will be the final time. Paul On Mon, Aug 31, 2020 at 12:03 PM Robert L. Nuckolls, III < nuckolls.bob(at)aeroelectric.com> wrote: > Does anyone have a .pdf copy of a Revmaster installation > manual for 2300, 3000 engines . . . or both? > > If there's a download link for either . . . that would > be fine too. > > Thanks! > > > Bob . . . > ________________________________________________________________________________
Date: Aug 31, 2020
From: "Robert L. Nuckolls, III" <nuckolls.bob(at)aeroelectric.com>
Subject: Re: Revmaster installation manual?
At 05:38 PM 8/31/2020, you wrote: >Bob > >Included is the section from the Revmaster=C2 >manual that deals with the generator and CDI >ignition. I'm looking forward to more from you >on the Revmaster problem so I can order a new >stator and rewire with confidence that the >second time around will be the final time. Yeah, I've seen this 'wiring diagram' . . . sad Everything else I've seen/read about this engine engenders a lot of confidence. When you pay out killobux for a modern, strongly promoted engine you should get something better than this in the way of installation, performance and system integration. Can I have your burned stator? Bob . . . ________________________________________________________________________________
From: Paul Eckenroth <N509RV(at)eckenroth.com>
Date: Aug 31, 2020
Subject: Re: Revmaster installation manual?
The stator is still installed in the engine. I am currently flying on battery only. When the solution to the Revmaster debacle is clear, I will get a new stator. Replacement of the stator is not trivial as the engine must be removed from the airframe. However when it is finally removed I will send it your way if still interested. Paul On Mon, Aug 31, 2020 at 7:21 PM Robert L. Nuckolls, III < nuckolls.bob(at)aeroelectric.com> wrote: > At 05:38 PM 8/31/2020, you wrote: > > Bob > > Included is the section from the Revmaster=C3=82 manual that deals with t he > generator and CDI ignition. I'm looking forward to more from you on the > Revmaster problem so I can order a new stator and rewire with confidence > that the second time around will be the final time. > > > Yeah, I've seen this 'wiring diagram' . . . sad > > Everything else I've seen/read about this engine > engenders a lot of confidence. When you pay out > killobux for a modern, strongly promoted engine you > should get something better than this in the way > of installation, performance and system integration. > > > Can I have your burned stator? > > > Bob . . . > ________________________________________________________________________________
Date: Aug 31, 2020
From: "Robert L. Nuckolls, III" <nuckolls.bob(at)aeroelectric.com>
Subject: Re: Revmaster installation manual?
At 07:22 PM 8/31/2020, you wrote: >The stator is still installed in the engine.=C2 I >am currently flying on battery only.=C2 When the >solution to the Revmaster debacle is clear, I >will get a new stator.=C2 Replacement of the >stator is not trivial as the engine must be >removed from the airframe.=C2 However when it is >finally removed I will send it your way if still interested. Sounds like a deal . . . Do I surmise correctly that you ran one alternator until it toasted, then ran the second? Bob . . . ________________________________________________________________________________
Subject: How do you turn 13 D-Sub pins into 5 wires?
From: "rparigoris" <rparigor(at)hotmail.com>
Date: Aug 31, 2020
Hi Group I have 13 pins on a D-Sub connector coming from Avionics that need to go to single point ground firewall field of tabs. I need to turn into ~ 5 wires that will go to field of tabs. I think I remember seeing how to do this but can't find it. Thx. Ron P. Read this topic online here: http://forums.matronics.com/viewtopic.php?p=498100#498100 ________________________________________________________________________________
Subject: 25 amp slow blow fuse
From: "rparigoris" <rparigor(at)hotmail.com>
Date: Aug 31, 2020
Hi Group I need a 25 amp slow blow fuse for my Rotax generator. Anyone know a source? I would prefer something robust like a slow blow ANL fuse that can be mounted on stud. Thx. Ron P. Read this topic online here: http://forums.matronics.com/viewtopic.php?p=498101#498101 ________________________________________________________________________________
Subject: Re: Brownout Step Up Converter
From: "johnbright" <john_s_bright(at)yahoo.com>
Date: Aug 31, 2020
In case this is interesting. Analog Devices offers a demo board manufactured by Linear Technology. Googling "Demo Circuit 1641A" will find an Analog Devices page with much detail as well as a Mouser page where it sells for $128. "Demonstration circuit DC1641 is DC/DC boost converter featuring the LTC3786 constant frequency current mode boost controller. The DC1641A operates over 5V to 24V input and provides over 5A at 6V input voltage and over 9A of output current at 10V input as shown in figure 3. The 350kHz constant frequency operation results in small and efficient circuit. The converter provides high output voltage accuracy (typically 3%) over wide load range with no minimum load requirement. The demonstration circuit can be easily modified to generate different output voltages." Someone on VAF posted about it: https://vansairforce.net/community/showpost.php?p=1458960&postcount=2 -------- John Bright, RV-6A, at FWF, O-360 Z-101 single batt dual alt SDS EM-5-F. john_s_bright(at)yahoo.com, Newport News, Va https://drive.google.com/drive/folders/1u6GeZo6pmBWsKykLNVQMvu4o1VEVyP4K Read this topic online here: http://forums.matronics.com/viewtopic.php?p=498102#498102 ________________________________________________________________________________
From: Charlie England <ceengland7(at)gmail.com>
Date: Sep 01, 2020
Subject: Re: How do you turn 13 D-Sub pins into 5 wires?
On Mon, Aug 31, 2020 at 10:58 PM rparigoris wrote: > rparigor(at)hotmail.com> > > Hi Group I have 13 pins on a D-Sub connector coming from Avionics that > need to go to single point ground firewall field of tabs. I need to turn > into ~ 5 wires that will go to field of tabs. I think I remember seeing how > to do this but can't find it. Thx. Ron P. Hi Ron, Are you talking about the 'panel ground' Dsub described in Z15L1 in the book? I used a slightly different technique. I used a solder style female connector for the ground buss, and soldered a larger gauge wire bridged across all the pins, with a larger gauge wire soldered to each end of the Dsub, and run to the firewall ground. Not '5 wire' redundant, but still redundant, and all panel ground currents can be safely/adequately carried by either of the two bigger wires to the firewall. Regardless of the number of ground wires going back to the firewall, the drawing shows all tied to a ground 'bus'. That would mean that all the pins in the Dsub should be common to each other at the Dsub. If you used a regular crimp style Dsub on the panel, you can get to the same point by using 'ballast' wires, ~6" long, from each pin. All the wires would be brought together at their free ends, and the 5 ground wires would attach there. For something that big, I'd solder/heat shrink. But if you prefer all-crimps, you can do it in 'stages', but having multiple 'Y's with a convenient number of wires (3 or 4) in each crimp, and those 'outputs' gathered into a last crimp to mate with your 5 ground wires to the firewall. Only downside to this is that the final sum-point to the 5 ground wires is a single point of failure (if you do a really lousy job in making up that joint). :-) Note that if you used subgroups of 3 or 4 pins to each of the 5 grounds back to the firewall (without tying all the pins together at/near the panel ground Dsub), and one of the 5 wires breaks, you'd lose ground for all the devices grounded through that particular wire. Hope that gives you some ideas on how to proceed, Charlie Virus-free. www.avast.com <#DAB4FAD8-2DD7-40BB-A1B8-4E2AA1F9FDF2> ________________________________________________________________________________
Subject: Re: How do you turn 13 D-Sub pins into 5 wires?
From: "johnbright" <john_s_bright(at)yahoo.com>
Date: Sep 01, 2020
Ref "Avionics and Panel Ground System" in chapter 18, page 18-11, of the Aeroelectric Connection book. http://www.aeroelectric.com/Books/Connection/ -------- John Bright, RV-6A, at FWF, O-360 Z-101 single batt dual alt SDS EM-5-F. john_s_bright(at)yahoo.com, Newport News, Va https://drive.google.com/drive/folders/1u6GeZo6pmBWsKykLNVQMvu4o1VEVyP4K Read this topic online here: http://forums.matronics.com/viewtopic.php?p=498108#498108 ________________________________________________________________________________
Subject: Re: 25 amp slow blow fuse
From: "user9253" <fransew(at)gmail.com>
Date: Sep 01, 2020
There are more than 500 Van's RV-12s flying with Rotax 912ULS eninges. They all use 30 amp ATC or ATO fuses for the alternator output. -------- Joe Gores Read this topic online here: http://forums.matronics.com/viewtopic.php?p=498110#498110 ________________________________________________________________________________
From: Paul Eckenroth <N509RV(at)eckenroth.com>
Date: Sep 01, 2020
Subject: Re: Revmaster installation manual?
I did run one generator at a time. Unfortunately, when it toasted it also removed the second generator. This also happened to a friend with the same engine and battery. So much for redundancy. Paul On Mon, Aug 31, 2020 at 11:12 PM Robert L. Nuckolls, III < nuckolls.bob(at)aeroelectric.com> wrote: > At 07:22 PM 8/31/2020, you wrote: > > The stator is still installed in the engine.=C3=82 I am currently flying on > battery only.=C3=82 When the solution to the Revmaster debacle is clear, I will > get a new stator.=C3=82 Replacement of the stator is not trivial as the engine > must be removed from the airframe.=C3=82 However when it is finally remo ved I > will send it your way if still interested. > > > Sounds like a deal . . . > > Do I surmise correctly that you ran one alternator until > it toasted, then ran the second? > > Bob . . . > ________________________________________________________________________________
Date: Sep 01, 2020
From: "Robert L. Nuckolls, III" <nuckolls.bob(at)aeroelectric.com>
Subject: Re: 25 amp slow blow fuse
At 10:58 AM 9/1/2020, you wrote: > >There are more than 500 Van's RV-12s flying with Rotax 912ULS eninges. >They all use 30 amp ATC or ATO fuses for the alternator output. > >-------- >Joe Gores My recommendation as well. If you want to install something a bit more robust, you could step up to the MAXI line of plastic fuses. The HHX fuseholder has a mounting hole for mechanically secure installation. This one comes with 30A fuse already installed. https://tinyurl.com/y2htcz4u . . . but the ATO/ATC fuses have amassed a good track record in this application as well. Bob . . . ________________________________________________________________________________
Date: Sep 01, 2020
From: "Robert L. Nuckolls, III" <nuckolls.bob(at)aeroelectric.com>
Subject: Re: Revmaster installation manual?
At 11:06 AM 9/1/2020, you wrote: >I did run one generator at a >time.=C2 Unfortunately, when it toasted it also >removed the second generator.=C2 This also >happened to a friend=C2 with the same engine and >battery.=C2 So much for redundancy. > >Paul Oh . . . the windings share a set of pole pieces . . . yeah . . . so much for FMEA as well! Bob . . . ________________________________________________________________________________
Date: Sep 01, 2020
From: "Robert L. Nuckolls, III" <nuckolls.bob(at)aeroelectric.com>
Subject: Re: Brownout Step Up Converter
At 10:59 PM 8/31/2020, you wrote: > > >In case this is interesting. > >Analog Devices offers a demo board manufactured by Linear Technology. > >Googling "Demo Circuit 1641A" will find an Analog Devices page with >much detail as well as a Mouser page where it sells for $128. This is one of a large family of such devices. Some are quite sophisticated. There's always a push to achieve the greatest possible efficiency. For our purposes, we need a relatively 'tiny' boost lasting a second or so at relatively low power . . . perhaps 50 watts in a worst case (lot's of vulnerable accessories on the brownout bus). If your battery is in good shape, the brownout event is over before the engine starts. See https://tinyurl.com/y294j7zl The starter characteristic that precipitates the brown out is INRUSH current. This can be 5 to 10 times greater than cranking current. However, as you can see in the plot above, it lasts but a handful of milliseconds. Once the starter motor spins up, about 100 mS, the battery rises back above 9V. The performance questions for commercial off-the- shelf supplies is not so much about carrying a load but how fast the device comes on line and how fast it responds to the dynamics of it's bus supply voltage. From the time the starter button is pressed until the starter inrush event begins is on the order of 5-10 milliseconds depending on contactor designs. So, from the time the starter push button is pressed, we want the boost supply to be on the job . . . further, it needs to respond to the sudden dip in supply voltage during inrush time. But the time that the booster needs to grunt a load is very short. This is why some of the early, super-capacitor based brown-out mitigation schemes were successful. But the demands on brown-out bus performance outgrew those ideas. The exemplar boost supplies I have need to be peeked at on the bench. I'll be out of town tomorrow but I think I can get some 'scope pictures of their performance before I mail one out. Bob . . . ________________________________________________________________________________
Date: Sep 01, 2020
From: Bobby Paulk <bobbypaulk(at)comcast.net>
Subject: Re: OBAM and PM Alternators
Thanks Bob and the others that gave their definition of OBAM. I like it. I have a Jabiru engine that HAD a PM Alternator. I started messing with it to get more voltage at a lower RPM. Long story short ( pun intended ) I managed to get a wire just close enough to the magnets that it shorted the coils that I had wired in series. I sent the fried coils to Bob a couple of years ago. Bought a 40 amp alternator with adapter and belt drive and have had no more problems. I have learned a lot from this list Bobby BTW this list comes through every day on G-Mail but seldom on Comcast. When it does come in on Comcast it and others are labeled as Spam ________________________________________________________________________________
Subject: Re: Z13-8 main alternator troubleshooting
From: "rvtach" <rvtach(at)msn.com>
Date: Sep 01, 2020
So this test confirmed that the old regulator was not functioning. I installed the trial Ford regulator permanently. So nice to know with some confidence that my alternator is working normally. Thank you for this tip and for everything I have learned lurking here over the years! -Jim -------- Jim McChesney Tucson, AZ RV-7A Finishing Kit Read this topic online here: http://forums.matronics.com/viewtopic.php?p=498127#498127 ________________________________________________________________________________
Subject: AV 30 Wiring
From: "farmrjohn" <faithvineyard(at)yahoo.com>
Date: Sep 02, 2020
Now that the AV 30 has STC approval for certificated airplanes I have some questions on the wiring differences. The experimental say power is to be from the battery or essential bus only, the "approved" manual says power is from the avionics bus or main master relay. Can the experimental version also be powered from the avionics bus? The certified calls for a 3/16" braided ground strap from the of the d-sub back shell, maximum of 8". What would be a similar sized stranded wire, and would the also be a good idea for the experimental version? On the wiring diagrams some of the shielded wire symbols have an X in them. Does this indicate twisted pair? Read this topic online here: http://forums.matronics.com/viewtopic.php?p=498136#498136 Attachments: http://forums.matronics.com//files/av30_experimental_107.jpg http://forums.matronics.com//files/av30_cert_636.jpg ________________________________________________________________________________
Subject: Lycoming EIS wirig recommendation
From: Werner Schneider <glastar(at)gmx.net>
Date: Sep 02, 2020
Got today SI 1569A which is talking about the new Lycoming EIS which are power dependent. Interesting their wiring recommendation, looks so familiar: Cheers Werner ________________________________________________________________________________
Subject: Re: Revmaster 'dual' alterantors
From: "dj_theis" <djtheis58(at)gmail.com>
Date: Sep 03, 2020
>>Without talking to the designers, it's >>difficult to KNOW the original thinking behind the >>design. I've spoken to Joe Horvoth a couple of times, before and just after purchasing my engine. I spoke to him at least once regarding the alternator and ignition system. Joe is not a particularly "chatty" fella but I think he is rightly proud of his design. His innovation in the design and improvements of the VW are truly impressive. Joe is clearly gifted when it comes to mechanical design and engine building. Unfortunately, I think the electrical systems were done by a hired gun and not by Joe directly (an unconfirmed rumor I am starting). As noted in the earlier posts and comments, Joe has strongly recommended not to run both alternator sides at the same time and has also strongly discouraged the use of anything but traditional lead acid (wet) batteries. I've never heard him go into any detail on the reasons behind these recommendations. I should have asked when I visited him a few years back. >>So take a peek at the simplified proposal diagram. >>Hook the two Revmaster windings in series-aiding >>and bring out the center-tap. Connect in full wave >>configuration with only TWO junction rectifiers. I like the approach and two questions come to mind: How well the current will be equally divided by the two windings? This is half wave rectification, right? I have not spun my engine yet but from examination of the windings I expect to see the two PMAs "IN PHASE" and as noted, single phase, not polyphase. Does this configuration change the expectation of roughly evenly divided current? If the two PMAs were out of phase by 180 degrees (which is possible if my talents with the "right hand rule" are off abit) I assume the center tap would not operate the same, if at all. I have to pull my engine once more before I start it (hopefully, yet this fall) and will examine the windings on the PMAs closely to convince myself of the phase relationship between the two halves. One final note. I've thought about this a little (clearly not as much as Bob). I plan on installing thermo-couples as close to the center of each stator base as I can. My thought is to run the engine as deigned, with OEM regulators and running one halve at a time. I am planning on a Odyssey battery (PC680) with added (2) shunt resistors in the charge circuit to increase the load seen by the PMAs. I plan on trying to identify what the conditions are that lead to the high temperature in the PMAs. I hope to be able to install the TCs close enough to the windings to obtain a warning before they overheat. I think the expected meltdown of the "enamel" insulation is a bit over 200C. Can anyone confirm? In the long term, I'd like to install something like the CCCV regulator mentioned. Is there actually such a regulator designed and available for a PMA on an OBAM aircraft? Dan Theis -------- Scratch building Sonex #1362 Read this topic online here: http://forums.matronics.com/viewtopic.php?p=498148#498148 ________________________________________________________________________________
Subject: Re: Revmaster 'dual' alterantors
From: "user9253" <fransew(at)gmail.com>
Date: Sep 04, 2020
Actually it is desired that the two windings be 180 degrees out of phase. That way, only one winding will conduct at a time and the output will be pulsing DC. If the two windings are in phase, no problem, just reverse the leads on one of the windings. Then the windings will be 180 degrees out of phase. -------- Joe Gores Read this topic online here: http://forums.matronics.com/viewtopic.php?p=498149#498149 ________________________________________________________________________________
Date: Sep 04, 2020
From: "Robert L. Nuckolls, III" <nuckolls.bob(at)aeroelectric.com>
Subject: Re: Revmaster 'dual' alterantors
> >>So take a peek at the simplified proposal diagram. > >>Hook the two Revmaster windings in series-aiding > >>and bring out the center-tap. Connect in full wave > >>configuration with only TWO junction rectifiers. > >I like the approach and two questions come to mind: > How well the current will be equally divided by the two windings? > This is half wave rectification, right? No, full wave . . . the current doesn't NEED to be exactly shared . . . each winding conduct on 1/2 cycle of the time independently of each other. Unlike the legacy, single winding PMA and bridge rectifier approach, this configuration splits the two halves onto separate windings . . . but it's still full wave rectification. >I have not spun my engine yet but from examination of the windings I >expect to see the two PMAs "IN PHASE" and as noted, single phase, >not polyphase. Does this configuration change the expectation of >roughly evenly divided current? If the two PMAs were out of phase >by 180 degrees (which is possible if my talents with the "right hand >rule" are off abit) I assume the center tap would not operate the >same, if at all. From what I understand, the Revmaster PMA is wound BI-FILAR meaning two strands of wire side-by-by side. This produces two, identical windings that COULD and perhaps SHOULD be wired in parallel. This would cut the current in each winding to 1/2 of the total. >I have to pull my engine once more before I start it (hopefully, yet >this fall) and will examine the windings on the PMAs closely to >convince myself of the phase relationship between the two halves. The result is predictable. You have two, identical windings sharing the same physical space on the stator. Voltages induced by the mechanics and magnetics are identical. Given that the burning the 'active' winding also burns the 'standby' winding, you do not have redundant systems. You might as well run the two windings in parallel which would greatly reduce the stress on the wires and may well drive the 'lithium' failure rate to zero. >One final note. I've thought about this a little (clearly not as >much as Bob). I plan on installing thermo-couples as close to the >center of each stator base as I can. My thought is to run the >engine as deigned, with OEM regulators and running one halve at a >time. I am planning on a Odyssey battery (PC680) with added (2) >shunt resistors in the charge circuit to increase the load seen by >the PMAs. I plan on trying to identify what the conditions are that >lead to the high temperature in the PMAs. I hope to be able to >install the TCs close enough to the windings to obtain a warning >before they overheat. I think the expected meltdown of the "enamel" >insulation is a bit over 200C. Can anyone confirm? I don't know of any manufacturer of aviation hardware that doesn't call out Class H insulation for their magnet wire https://tinyurl.com/y3a7o26s Without going to some exotic wire coated with un-obtainium, Class H is the best you can buy off the shelf. So yeah, 200C max operating for the copper . . . Thermocoupling the windings is not a bad idea. Are those stator windings varnished? Getting a 'real' copper temperature number on a running machine is not easy. The closest I ever got to 'real' measurements was by bringing leads for winding-under-observation outside so that it could be quickly switched from 'service' to 'measure' mode by exciting the winding with a calibrated current and then measuring the voltage drop. Copper temperature can be calculated by knowing the temperature coefficient of resistance for copper. This had to be done quickly (under 100 mS) 'cause the copper cools very rapidly when the loads are removed and the heat soaks out. Generally speaking, with your 200C rated wire, getting thermocouple readings over 170-180C would be cause for concern. We KNOW this design is thermally deficient . . . too many of them have burned up in service. Too many band-aids have been suggested and attempted with little if any relief. >In the long term, I'd like to install something like the CCCV >regulator mentioned. Is there actually such a regulator designed >and available for a PMA on an OBAM aircraft? Sure, they're commercial off the shelf items. I've got a couple on the bench now that I'm going to use in a proof-of-concept study. But after reading about the bi-filar windings design, I'm not sure there's much value to be secured by 'upgrading' the rectifier/regulator design for this engine. As currently configured, there is NO REDUNDANCY of engine driven power sources. I'm beginning to think one would be well advised to simply parallel the two windings and treat them as one. This alone would produce a profound drop in wire temperatures for any given load. Bob . . . ________________________________________________________________________________
Subject: Re: Revmaster 'dual' alterantors
From: "dj_theis" <djtheis58(at)gmail.com>
Date: Sep 04, 2020
[quote] Actually it is desired that the two windings be 180 degrees out of phase. That way, only one winding will conduct at a time and the output will be pulsing DC. If the two windings are in phase, no problem, just reverse the leads on one of the windings. Then the windings will be 180 degrees out of phase. [/quote] Ah, yes. that makes perfect sense. The end result is a full wave rectified waveform output. Thanks for the clarification Joe. Dan Theis.[/quote] -------- Scratch building Sonex #1362 Read this topic online here: http://forums.matronics.com/viewtopic.php?p=498151#498151 ________________________________________________________________________________
Subject: Re: Lycoming EIS wirig recommendation
From: "eschlanser" <eschlanser(at)yahoo.com>
Date: Sep 05, 2020
Thanks for the link. Is the Lycoming EIS an off-the-shelf or a clean sheet system and who makes it? Read this topic online here: http://forums.matronics.com/viewtopic.php?p=498152#498152 ________________________________________________________________________________
Subject: Re: Lycoming EIS wirig recommendation
From: Werner Schneider <glastar(at)gmx.net>
Date: Sep 05, 2020
It looks Lycoming branded but identical to the Surefly, for certified it has fixed timing :) Cheers Werner On 05.09.2020 15:18, eschlanser wrote: > > Thanks for the link. > Is the Lycoming EIS an off-the-shelf or a clean sheet system and who makes it? > > > Read this topic online here: > > http://forums.matronics.com/viewtopic.php?p=498152#498152 > > ________________________________________________________________________________
Subject: Re: WireViz
From: "prestonkavanagh" <preston.kavanagh(at)gmail.com>
Date: Sep 05, 2020
I asked about an easy package for creating schematics. Last night I chased down a license for Visio Pro 2019. It has a generous set of symbols and has a lot of useful output formats (see the screen shots below). It plays well with my Office 365 set up. At $4 for the license (see https://www.ebay.com/itm/133504007620), it seems worth a try. -------- PBK3 PA-12, BD-4, RV6a, gliders Read this topic online here: http://forums.matronics.com/viewtopic.php?p=498153#498153 Attachments: http://forums.matronics.com//files/visio_outputs_116.png http://forums.matronics.com//files/visio1_135.png ________________________________________________________________________________
Subject: Re: Brownout Step Up Converter
From: "johnbright" <john_s_bright(at)yahoo.com>
Date: Sep 05, 2020
Dynon Skyview will operate down to 6V for ten seconds with no backup battery. I wonder how it is done and if we can learn something there. -------- John Bright, RV-6A, at FWF, O-360 Z-101 single batt dual alt SDS EM-5-F. john_s_bright(at)yahoo.com, Newport News, Va https://drive.google.com/drive/folders/1u6GeZo6pmBWsKykLNVQMvu4o1VEVyP4K Read this topic online here: http://forums.matronics.com/viewtopic.php?p=498155#498155 ________________________________________________________________________________
Subject: Re: Revmaster 'dual' alterantors
From: "dj_theis" <djtheis58(at)gmail.com>
Date: Sep 05, 2020
Bob, for some reason your post made it to me on the digest but does not show up on the forum (as far as I can find). You posted it just before Joe on Friday morning (I think). It begins as follows: >> >>So take a peek at the simplified proposal diagram. >> >>Hook the two Revmaster windings in series-aiding >> >>and bring out the center-tap. Connect in full wave >> >>configuration with only TWO junction rectifiers. > >>I like the approach and two questions come to mind: >> How well the current will be equally divided by the two windings? >> This is half wave rectification, right? > No, full wave . . . the current doesn't > NEED to be exactly shared . . . each winding > conduct on 1/2 cycle of the time independently > of each other. Unlike the legacy, single winding > PMA and bridge rectifier approach, this > configuration splits the two halves onto > separate windings . . . but it's still > full wave rectification OK, I've convinced myself that the OEM design made use of 1/2 wave rectification. Unless I'm wrong on this, running both sides with the OEM rectifiers (which is discouraged) would have the same current (total) heat generation as this very elegant center tap as you've shown. The other issue is heat from the engine. I have to wonder if the real culprit is not the windings at all but the proximity of the PMAs to the crankcase heat. If we consider the report on the Onex where both sides failed (burned) when only one was operating, perhaps it is the engine heat and inability to reject that, along with the PMAs internal heating that conspire against us. Not to throw another apple into the barrel but can we effectively add a blast tube to the environment around the PMAs? As far as I can see, there is one tiny little inlet hole for ventilation from above the engine into the PMA area. This hole is mostly covered by the flywheel. Not much of any opportunity to reject heat other than conduction through the surface of the casting at the back of the engine. Dan Theis -------- Scratch building Sonex #1362 Read this topic online here: http://forums.matronics.com/viewtopic.php?p=498156#498156 ________________________________________________________________________________
Subject: Re: Z101 has been released under Rev A
From: Werner Schneider <glastar(at)gmx.net>
Date: Sep 06, 2020
Dear Bob, is the Z101 source as well available as dwg file so I can taylor it to my new project (SDS/emag/no brownout). Thanks a lot for your help, BTW will the next edition of your book include these new diagrams? Would love to update my edition 10 (which was still paper ;) to a new edition with all your updates. Cheers Werner (Glastar Z11 and now RV-10 with Z101) On 26.05.2020 02:57, Robert L. Nuckolls, III wrote: > > > I've combed Z101 several times and I think > the major 'nits' have been eliminated. I removed > 'preliminary' status and published as Rev -A- > at: > > https://tinyurl.com/yavoutjh > > Doesn't mean it's 'carved in stone' . . . just > ready for some serious builder consideration. > Error call outs and suggestions always welcome. > > > Bob . . . > > ________________________________________________________________________________
From: Tim Mikus <timmikus(at)icloud.com>
Date: Sep 06, 2020
Subject: Registration
I have tried a couple of times under two different email addresses to register, both times I received a message that said the administrator would need to approve my request (second try was yesterday). How can I get signed up? Thank you. Sent from my iPad ________________________________________________________________________________
Date: Sep 06, 2020
From: Matt Dralle <dralle(at)matronics.com>
Subject: Re: Registration
What were the requested usernames? Matt At 9/6/2020 12:40 PM Sunday, you wrote: > >I have tried a couple of times under two different email addresses >to register, both times I received a message that said the >administrator would need to approve my request (second try was >yesterday). How can I get signed up? Thank you. > >Sent from my iPad ________________________________________________________________________________
Subject: Re: Brownout Step Up Converter
From: "donjohnston" <don@velocity-xl.com>
Date: Sep 07, 2020
I used the IPS from TCW Tech. It worked fine. But then I pullen it and installed a backup battery for the HXr. This allows me to power up the EFIS well before turning on the master or starting the engine in addition to providing brownout prevention. It also provides an additional power source for the EFIS should things go really sideways in the electrical system. GRT also sells a backup battery (12v only). Read this topic online here: http://forums.matronics.com/viewtopic.php?p=498166#498166 ________________________________________________________________________________
Date: Sep 07, 2020
From: "Robert L. Nuckolls, III" <nuckolls.bob(at)aeroelectric.com>
Subject: Re: Z101 has been released under Rev A
At 07:34 AM 9/6/2020, you wrote: > >Dear Bob, > >is the Z101 source as well available as dwg file so I can taylor it >to my new project (SDS/emag/no brownout). Yes, both files of RevB are posted. The next upgrade to the Z-Figures will probably 'retire' most if not all of the legacy figures applicable to aircraft engines (lyc, cont). Z101 iterations cover all previous variations on the theme. Bob . . . ________________________________________________________________________________
From: Rick Beebe <rick(at)beebe.org>
Subject: Alternator/Strobe whine
Date: Sep 07, 2020
I could use a little help debugging this problem. A few months ago I complete a major panel overhaul on my GlaStar basically using Z-12. The plane came with a B&C L40 alternator with an LR-3 voltage regulator. There's always been strobe noise in the headset. A couple years ago I replaced the "old" Whelen strobes with AeroFlash LED strobes and all new shielded wiring. I still had noise but it wasn't bad. As part of this overhaul, I added a B&C BC-410H accessory pad alternator and SB1B-14 standby regulator. Since either alternator can handle the worst-case load, there's no essential bus. For space reasons I ended up relocating and almost entirely rewiring the LR-3. I also totally replaced all the avionics including audio panel, intercom, headset jacks and all associated wiring. The problem--the strobe noise is worse. But it's only present when the primary alternator is on. On just battery or just standby alternator the system is silent. Any thoughts on how to silence this little demon? Thanks --Rick ________________________________________________________________________________
Subject: Re: Z101 has been released under Rev A
From: Werner Schneider <glastar(at)gmx.net>
Date: Sep 07, 2020
Thanks Bob, I downloaded the 101B but my Intellicad says that it was created with an unsupported version of Autocad and can not open it. Any chance you can save a version with an older Autocad version Z14P from 2013 still works up to Z34 ( 11/2014) from Z11P (2016 upwards it fails) Thanks Werner On 07.09.2020 15:54, Robert L. Nuckolls, III wrote: > At 07:34 AM 9/6/2020, you wrote: >> >> >> Dear Bob, >> >> is the Z101 source as well available as dwg file so I can taylor it to >> my new project (SDS/emag/no brownout). > > Yes, both files of RevB are posted. > > The next upgrade to the Z-Figures will probably > 'retire' most if not all of the legacy figures > applicable to aircraft engines (lyc, cont). Z101 > iterations cover all previous variations on the > theme. > > > Bob . . . > ________________________________________________________________________________
From: Charlie England <ceengland7(at)gmail.com>
Date: Sep 07, 2020
Subject: Re: Alternator/Strobe whine
On Mon, Sep 7, 2020 at 1:58 PM Rick Beebe wrote: > I could use a little help debugging this problem. A few months ago I > complete a major panel overhaul on my GlaStar basically using Z-12. The > plane came with a B&C L40 alternator with an LR-3 voltage regulator. > There's always been strobe noise in the headset. A couple years ago I > replaced the "old" Whelen strobes with AeroFlash LED strobes and all new > shielded wiring. I still had noise but it wasn't bad. > > As part of this overhaul, I added a B&C BC-410H accessory pad alternator > and SB1B-14 standby regulator. Since either alternator can handle the > worst-case load, there's no essential bus. For space reasons I ended up > relocating and almost entirely rewiring the LR-3. I also totally replaced > all the avionics including audio panel, intercom, headset jacks and all > associated wiring. > > The problem--the strobe noise is worse. But it's only present when the > primary alternator is on. On just battery or just standby alternator the > system is silent. Any thoughts on how to silence this little demon? > > Thanks > > --Rick > Hi Rick, Where are the strobe wire shield grounds? Did you keep the strobe ground isolated from the local (at the strobe) airframe ground? Are your headphone & mic jacks isolated from the airframe, and grounded only at the intercom? I've had moderate luck using the old (new?) single point ground technique shown in 'the book'. Since this is an ongoing issue with two different kinds of strobes, if it were me, I'd look at running an isolated strobe ground back to battery negative, or as close as is practical. What you *don't* want is the strobe and your low level audio stuff sharing the same ground path back to the battery negative. If that happens, the audio equipment's power supply (and likely the audio signal itself) sees the 'ripple' of noise from the strobes. It's conceivable that the old strobes were radiating into the radio's RF input, but I wouldn't have thought that the LED version would be as bad. Last (probably should have been first), you say 'strobe noise', but say it stops when the alternator is off. That doesn't sound like strobe noise; it sounds like alternator noise. Is the sound there when the alternator is on, but the strobes are off? Is (was) it the 'woop, woop' sound as the strobe fires, or continuous whine, or static, or ??? Does it vary in pitch or volume with engine rpm? If it's actually alternator noise, you probably still need to look at grounds. Make sure the alternator has a 'zero resistance' (clean) connection to the engine, and that the engine has a perfect, low impedance ground back to the battery negative (can be through a firewall ground stud, but that should go directly to the battery). If you're using the steel cage in the Glastar for ground, make sure that *all* ground terminations to the cage Battery location could be a factor; is it at the firewall or behind the luggage compartment? If all the grounds truly are good (only way to know for sure, without specialized test gear, is to unbolt and examine the mating surfaces), and it really is alternator noise instead of strobe noise, you might have a bad diode or two in the alternator itself. Charlie ________________________________________________________________________________
Date: Sep 07, 2020
From: "Robert L. Nuckolls, III" <nuckolls.bob(at)aeroelectric.com>
Subject: Re: Z101 has been released under Rev A
At 04:11 PM 9/7/2020, you wrote: > >Thanks Bob, > >I downloaded the 101B but my Intellicad says that it was created with an >unsupported version of Autocad and can not open it. > >Any chance you can save a version with an older Autocad version try this one http://www.aeroelectric.com/PPS/ACAD_Architecture_Dwgs/Z101B(R14).dwg Bob . . . ________________________________________________________________________________
From: Richard Girard <aslsa.rng(at)gmail.com>
Date: Sep 07, 2020
Subject: Re: Z101 has been released under Rev A
Werner, Try the free trial version of Draftsjght. It works all rhe way back to Acad 13. Rick <http://www.avg.com/email-signature?utm_medium=email&utm_source=link&ut m_campaign=sig-email&utm_content=webmail> Virus-free. www.avg.com <http://www.avg.com/email-signature?utm_medium=email&utm_source=link&ut m_campaign=sig-email&utm_content=webmail> <#DAB4FAD8-2DD7-40BB-A1B8-4E2AA1F9FDF2> On Mon, Sep 7, 2020 at 4:21 PM Werner Schneider wrote: t > > > > Thanks Bob, > > I downloaded the 101B but my Intellicad says that it was created with an > unsupported version of Autocad and can not open it. > > Any chance you can save a version with an older Autocad version > > Z14P from 2013 still works up to Z34 ( 11/2014) from Z11P (2016 upwards > it fails) > > Thanks Werner > > On 07.09.2020 15:54, Robert L. Nuckolls, III wrote: > > At 07:34 AM 9/6/2020, you wrote: > >> > >> > >> Dear Bob, > >> > >> is the Z101 source as well available as dwg file so I can taylor it to > >> my new project (SDS/emag/no brownout). > > > > Yes, both files of RevB are posted. > > > > The next upgrade to the Z-Figures will probably > > 'retire' most if not all of the legacy figures > > applicable to aircraft engines (lyc, cont). Z101 > > iterations cover all previous variations on the > > theme. > > > > > > Bob . . . > > > =========== =========== =========== =========== =========== > > -- =9CBlessed are the cracked, for they shall let in the light.=9D Groucho Marx <http://www.goodreads.com/author/show/43244.Groucho_Marx> ________________________________________________________________________________
Subject: Re: Alternator/Strobe whine
From: Rick Beebe <rick(at)beebe.org>
Date: Sep 07, 2020
Thanks for the help. The strobe noise is the 'woop woop' sound. It was much woopier with the Whelen strobes but it's clearly happening when the strobes flash. And it does stop when the strobes are off. There are sort-of two ground points now. There's a terminal strip in the forward cockpit that all the non-avionics stuff such as lights are grounded to. The ground wire from the battery runs to a large stud at one end of that terminal strip. There's another large stud on the other end with a large wire going to the engine. When I built the new panel I incorporated a DB-25 central ground connector on the panel with redundant connections to that terminal strip. The strobes are not grounded at the wingtip. I ran the wire back to the grounding terminal strip. I guess it's safe to say the strobe ground does share the audio ground back to the battery. That said, I suppose it could be alternator noise. The strobes do cause a fairly significant current spike when they flash. Maybe I'm really hearing the alternator working harder during those moments. --Rick On 9/7/2020 5:41 PM, Charlie England wrote: > Hi Rick, > > Where are the strobe wire shieldgrounds? Did you keep the strobe > ground isolated from the local (at the strobe) airframe ground? Are > your headphone& mic jacks isolated from the airframe, and grounded > only at the intercom? I've had moderate luck using the old (new?) > single point ground technique shown in 'the book'. Since this is an > ongoing issue with two different kinds of strobes, if it were me, I'd > look at running an isolated strobe ground back to battery negative, or > as close as is practical. What you *don't* want is the strobe and your > low level audio stuff sharing the same ground path back to the battery > negative. If that happens, the audio equipment'spower supply (and > likely the audio signal itself) sees the 'ripple' of noise from the > strobes. > > It's conceivable that the old strobes were radiating into the radio's > RF input, but I wouldn't have thought that the LED version would be as > bad. > > Last (probably should have been first), you say 'strobe noise', but > say it stops when the alternator is off. That doesn't sound like > strobe noise; it sounds like alternator noise. Is the sound there when > the alternator is on, but the strobes are off? Is (was) it the 'woop, > woop' sound as the strobe fires, or continuous whine, or static, or > ??? Does it vary in pitch or volume with engine rpm? > > If it's actually alternator noise, you probably still need to look at > grounds. Make sure the alternator has a 'zero resistance' (clean) > connection to the engine, and that the engine has a perfect, low > impedance ground back to the battery negative (can be through a > firewall ground stud, but that should go directly to the battery). If > you're using the steel cage in the Glastar for ground, make sure that > *all* ground terminations to the cage Battery location could be a > factor; is it at the firewall or behind the luggage compartment? > > If all the grounds truly are good (only way to know for sure, without > specialized test gear, is to unbolt and examine the mating surfaces), > and it really is alternator noise instead of strobe noise, you might > have a bad diode or two in the alternator itself. > > Charlie ________________________________________________________________________________
Subject: Test; Ignore
From: "Matt Dralle" <dralle(at)matronics.com>
Date: Sep 07, 2020
Test, please ignore... -------- Matt Dralle Matronics Email List Administrator Read this topic online here: http://forums.matronics.com/viewtopic.php?p=498200#498200 ________________________________________________________________________________
Subject: Re: Z101 has been released under Rev A
From: Werner Schneider <glastar(at)gmx.net>
Date: Sep 08, 2020
Woked perfectly, many thanks!! Cheers Werner On 08.09.2020 00:49, Robert L. Nuckolls, III wrote: > At 04:11 PM 9/7/2020, you wrote: >> >> >> Thanks Bob, >> >> I downloaded the 101B but my Intellicad says that it was created with an >> unsupported version of Autocad and can not open it. >> >> Any chance you can save a version with an older Autocad version > > > try this one > > > http://www.aeroelectric.com/PPS/ACAD_Architecture_Dwgs/Z101B(R14).dwg > > > Bob . . . > ________________________________________________________________________________
Subject: Re: Z101 has been released under Rev A
From: Werner Schneider <glastar(at)gmx.net>
Date: Sep 08, 2020
Thanks Richard, might give it a try, it's just after 17 years and fluent working with Intellicad ;) Cheers Werner On 08.09.2020 01:58, Richard Girard wrote: > Werner, Try the free trial versionof Draftsjght. It works all rheway > back to Acad 13. > > Rick > > <http://www.avg.com/email-signature?utm_medium=email&utm_source=link&utm_campaign=sig-email&utm_content=webmail> > Virus-free. www.avg.com > <http://www.avg.com/email-signature?utm_medium=email&utm_source=link&utm_campaign=sig-email&utm_content=webmail> > > > On Mon, Sep 7, 2020 at 4:21 PM Werner Schneider > wrote: > > > > > Thanks Bob, > > I downloaded the 101B but my Intellicad says that it was created > with an > unsupported version of Autocad and can not open it. > > Any chance you can save a version with an older Autocad version > > Z14P from 2013 still works up to Z34 ( 11/2014) from Z11P (2016 > upwards > it fails) > > Thanks Werner > > On 07.09.2020 15:54, Robert L. Nuckolls, III wrote: > > At 07:34 AM 9/6/2020, you wrote: > >> > > >> > >> Dear Bob, > >> > >> is the Z101 source as well available as dwg file so I can > taylor it to > >> my new project (SDS/emag/no brownout). > > > > Yes, both files of RevB are posted. > > > > The next upgrade to the Z-Figures will probably > > 'retire' most if not all of the legacy figures > > applicable to aircraft engines (lyc, cont). Z101 > > iterations cover all previous variations on the > > theme. > > > > > > Bob . . . > > > > ========== > - > Electric-List" rel="noreferrer" > target="_blank">http://www.matronics.com/Navigator?AeroElectric-List > ========== > FORUMS - > eferrer" target="_blank">http://forums.matronics.com > ========== > WIKI - > errer" target="_blank">http://wiki.matronics.com > ========== > b Site - > -Matt Dralle, List Admin. > rel="noreferrer" target="_blank">http://www.matronics.com/contribution > ========== > > > -- > > > Blessed are the cracked, for they shall let in the light. Groucho > Marx <http://www.goodreads.com/author/show/43244.Groucho_Marx> > > ________________________________________________________________________________
From: "Email Security Gateway" <dralle(at)matronics.com>
Subject: User Quarantine Account Information
Date: Sep 08, 2020
V2VsY29tZSB0byB0aGUgIEVtYWlsIFNlY3VyaXR5IEdhdGV3YXkgLiAgVGhpcyBtZXNzYWdlIGNv bnRhaW5zIHRoZSBpbmZvcm1hdGlvbiB5b3Ugd2lsbCBuZWVkIHRvIGFjY2VzcyB5b3VyIFNwYW0g UXVhcmFudGluZSBhbmQgUHJlZmVyZW5jZXMuCgpZb3VyIGFjY291bnQgaGFzIGJlZW4gc2V0IHRv IHRoZSBmb2xsb3dpbmcgdXNlcm5hbWUgYW5kIHBhc3N3b3JkOgogICAgVXNlcm5hbWU6ICBhZXJv ZWxlY3RyaWMtbGlzdEBtYXRyb25pY3MuY29tIAogICAgUGFzc3dvcmQ6ICBRTWNuWnZEKk1WQ1o9 d3lIIAoKVG8gbG9naW4gdG8geW91ciBTcGFtIFF1YXJhbnRpbmUgdXNlIHRoZSBmb2xsb3dpbmcg bGluazogIGh0dHBzOi8vOTYuNjguMTcxLjIxIAoKICAK ________________________________________________________________________________
Subject: Re: User Quarantine Account Information
From: GTH <gilles.thesee(at)free.fr>
Date: Sep 08, 2020
/Le 08/09/2020 12:00, Email Security Gateway a crit: / > /Welcome to the Email Security Gateway . This message contains the > information you will need to access your Spam Quarantine and Preferences. > / / > Your account has been set to the following username and password: > Username: aeroelectric-list(at)matronics.com > Password: ****** > / / > To login to your Spam Quarantine use the following link: > https://96.68.171.21/ Hi all, anyone received the above message ? Is it safe ? Thanks -- Best regards, Gilles http://contrails.free.fr http://lapierre.skunkworks.free.fr ________________________________________________________________________________
Subject: Re: User Quarantine Account Information
From: Werner Schneider <glastar(at)gmx.net>
Date: Sep 08, 2020
Hi Gilles, the IP belongs to Matt, so it should be safe, however I see no reason to check for SPAM/Quarantine messages on his server :) I guess Matt will enlight us soon if that one escaped by accident or it is something we should use. I guess he is installing new modules and that message escaped to the list instead of him. Cheers Werner On 08.09.2020 12:26, GTH wrote: > /Le 08/09/2020 12:00, Email Security Gateway a crit: > / >> /Welcome to the Email Security Gateway . This message contains the >> information you will need to access your Spam Quarantine and Preferences. >> / / >> Your account has been set to the following username and password: >> Username: aeroelectric-list(at)matronics.com >> Password: ****** >> / / >> To login to your Spam Quarantine use the following link: >> https://96.68.171.21/ > > > Hi all, anyone received the above message ? > > Is it safe ? > > Thanks > > -- > Best regards, > Gilles > http://contrails.free.fr > http://lapierre.skunkworks.free.fr ________________________________________________________________________________
Subject: Re: Alternator/Strobe whine
From: "user9253" <fransew(at)gmail.com>
Date: Sep 08, 2020
Try running power and ground to the strobes using a twisted pair of wires. Increasing the main alternator "B" terminal wire size might help. Make sure the main alternator terminals are clean and tight. -------- Joe Gores Read this topic online here: http://forums.matronics.com/viewtopic.php?p=498258#498258 ________________________________________________________________________________
Subject: Cigarette lighter socket to power a Garmin Aera 660
From: "Argonaut36" <fmlibrino(at)msn.com>
Date: Sep 08, 2020
I have recently purchased a Garmin AERA 660 portable GPS navigator. The unit holder to be used in the airplane is equipped with a cigarette lighter connector plug (outfitted with a 3A fast fuse). Could somebody recommend a good quality cigarette lighter connector socket that I can connect to the electrical system of my airplane? I would like to buy a socket that can be installed using an Adel clamp. Thanks Read this topic online here: http://forums.matronics.com/viewtopic.php?p=498259#498259 ________________________________________________________________________________
From: John M Tipton <john(at)tipton.me.uk>
Subject: Re: Cigarette lighter socket to power a Garmin Aera
660
Date: Sep 08, 2020
Cut off the plug and connect to your electrical system John Sent from my iPad ----x--O--x---- > On 8 Sep 2020, at 3:37 pm, Argonaut36 wrote: > > > I have recently purchased a Garmin AERA 660 portable GPS navigator. The unit holder to be used in the airplane is equipped with a cigarette lighter connector plug (outfitted with a 3A fast fuse). > > Could somebody recommend a good quality cigarette lighter connector socket that I can connect to the electrical system of my airplane? I would like to buy a socket that can be installed using an Adel clamp. > > Thanks > > > > > Read this topic online here: > > http://forums.matronics.com/viewtopic.php?p=498259#498259 > > > > > > > > > ________________________________________________________________________________
From: Paul Eckenroth <N509RV(at)eckenroth.com>
Date: Sep 08, 2020
Subject: Re: Revmaster 'dual' alterantors
I think that there might be a misunderstanding of the Revmaster architecture in that I don't believe that the windings for the two generators are in tandem but are a series of 5 coils each for two generators. I have enclosed a picture of the stator from my friend's engine. He was running one generator at the time and you can see how severely it was destroyed. The Phenolic base was also destroyed. The second generator coils don't look bad but they are also ruined. To me one of the interesting things is that the ignition coils are still functional. My primary purpose in starting this thread was to hopefully end up with a definite fix for the problem. Trying this or that is not practicable due to the destructive nature of the overheating. The stator is over $500 to replace and requires pulling the engine to do so. I'll only really know that the problem is fixed when the smoke stays in the coils. It seems to me that the most important change will be to use the B&C AVC1 volltage regulator since it is designed for firewall forward and can be adjusted down to 13.7 v which is the minimum voltage that will fully charge the EarthX battery. With the voltage reduction should come current reduction. What I don't know (and hopefully the educated on the list can explain) is whether restricting the voltage will also restrict the current. Can the generator still create abnormal current and therefore heat. I will try and incorporate thermocouples in with the coils but I'm not sure how much advance warning they can give. So, with the new voltage regulator set at 13.7 can the generator still pump out current based on engine speed and battery resistance or is the current capped since the voltage is restricted. I'm trying to determine if I have a good chance of success here. Does anybody want to redesign the generator to make it more robust. Paul [image: IMG-2451.jpeg] On Fri, Sep 4, 2020 at 9:36 AM Robert L. Nuckolls, III < nuckolls.bob(at)aeroelectric.com> wrote: > >>So take a peek at the simplified proposal diagram. > >>Hook the two Revmaster windings in series-aiding > >>and bring out the center-tap. Connect in full wave > >>configuration with only TWO junction rectifiers. > > I like the approach and two questions come to mind: > How well the current will be equally divided by the two windings? > This is half wave rectification, right? > > > No, full wave . . . the current doesn't > NEED to be exactly shared . . . each winding > conduct on 1/2 cycle of the time independently > of each other. Unlike the legacy, single winding > PMA and bridge rectifier approach, this > configuration splits the two halves onto > separate windings . . . but it's still > full wave rectification. > > I have not spun my engine yet but from examination of the windings I > expect to see the two PMAs "IN PHASE" and as noted, single phase, not > polyphase. Does this configuration change the expectation of roughly > evenly divided current? If the two PMAs were out of phase by 180 degrees > (which is possible if my talents with the "right hand rule" are off abit) I > assume the center tap would not operate the same, if at all. > > > From what I understand, the Revmaster > PMA is wound BI-FILAR meaning two strands > of wire side-by-by side. This produces > two, identical windings that COULD and > perhaps SHOULD be wired in parallel. This > would cut the current in each winding to > 1/2 of the total. > > I have to pull my engine once more before I start it (hopefully, yet this > fall) and will examine the windings on the PMAs closely to convince myself > of the phase relationship between the two halves. > > > The result is predictable. You have two, identical > windings sharing the same physical space > on the stator. Voltages induced by the > mechanics and magnetics are identical. > Given that the burning the 'active' winding > also burns the 'standby' winding, you do > not have redundant systems. You might as > well run the two windings in parallel > which would greatly reduce the stress on > the wires and may well drive the 'lithium' > failure rate to zero. > > One final note. I've thought about this a little (clearly not as much as > Bob). I plan on installing thermo-couples as close to the center of each > stator base as I can. My thought is to run the engine as deigned, with OEM > regulators and running one halve at a time. I am planning on a Odyssey > battery (PC680) with added (2) shunt resistors in the charge circuit to > increase the load seen by the PMAs. I plan on trying to identify what the > conditions are that lead to the high temperature in the PMAs. I hope to be > able to install the TCs close enough to the windings to obtain a warning > before they overheat. I think the expected meltdown of the "enamel" > insulation is a bit over 200C. Can anyone confirm? > > > I don't know of any manufacturer of aviation > hardware that doesn't call out Class H insulation > for their magnet wire > > https://tinyurl.com/y3a7o26s > > Without going to some exotic wire coated > with un-obtainium, Class H is the best > you can buy off the shelf. So yeah, 200C > max operating for the copper . . . > > Thermocoupling the windings is not a bad idea. > Are those stator windings varnished? Getting > a 'real' copper temperature number on a running > machine is not easy. The closest I ever > got to 'real' measurements was by bringing > leads for winding-under-observation outside > so that it could be quickly switched from > 'service' to 'measure' mode by exciting the > winding with a calibrated current and then > measuring the voltage drop. Copper temperature > can be calculated by knowing the temperature coefficient > of resistance for copper. This had to be done > quickly (under 100 mS) 'cause the copper cools > very rapidly when the loads are removed and the > heat soaks out. > > Generally speaking, with your 200C rated wire, > getting thermocouple readings over 170-180C > would be cause for concern. > > We KNOW this design is thermally deficient . . . > too many of them have burned up in service. > Too many band-aids have been suggested > and attempted with little if any relief. > > > In the long term, I'd like to install something like the CCCV regulator > mentioned. Is there actually such a regulator designed and available for a > PMA on an OBAM aircraft? > > > Sure, they're commercial off the shelf items. > I've got a couple on the bench now that I'm > going to use in a proof-of-concept study. But > after reading about the bi-filar windings > design, I'm not sure there's much value to > be secured by 'upgrading' the rectifier/regulator > design for this engine. > > As currently configured, there is NO REDUNDANCY > of engine driven power sources. I'm beginning > to think one would be well advised to > simply parallel the two windings and treat > them as one. This alone would produce a profound > drop in wire temperatures for any given load. > > Bob . . . > ________________________________________________________________________________
Date: Sep 08, 2020
From: "Robert L. Nuckolls, III" <nuckolls.bob(at)aeroelectric.com>
Subject: Re: Alternator/Strobe whine
Alternator, strobe or both? Alternator whine is almost always a ground loop issue. Pitch of whine will go up and down with engine rpm. It often gets louder when alternator loads are highest like right after starting engine (recharging battery) and all loads turned on. Strobe whine pitches up and down in synchronization with flashes from the strobes and is generally of constant volume. Which one(s) disappear when appliance is OFF. Alternator, strobe or both? Bob . . . ________________________________________________________________________________
Subject: Re: Alternator/Strobe whine
From: Charlie England <ceengland7(at)gmail.com>
Date: Sep 08, 2020
On 9/8/2020 3:41 PM, Robert L. Nuckolls, III wrote: > Alternator, strobe or both? > > Alternator whine is almost always a ground loop issue. > Pitch of whine will go up and down with engine rpm. > It often gets louder when alternator loads are highest > like right after starting engine (recharging battery) > and all loads turned on. > > Strobe whine pitches up and down in synchronization > with flashes from the strobes and is generally of > constant volume. > > Which one(s) disappear when appliance is OFF. > Alternator, strobe or both? > > > Bob . . . > I think he mentioned in his reply to me that it was the 'woop woop' indicating it is a strobe sound, but he also said that it stops when the main alternator is turned off. (?) -- This email has been checked for viruses by Avast antivirus software. https://www.avast.com/antivirus ________________________________________________________________________________
Subject: Re: User Quarantine Account Information
From: GTH <gilles.thesee(at)free.fr>
Date: Sep 09, 2020
Le 08/09/2020 13:41, Werner Schneider a crit: > / > / / > I guess Matt will enlight us soon if that one escaped by accident or it > is something we should use. > / / > I guess he is installing new modules and that message escaped to the > list instead of him. > / Thank you Werner ! -- Best regards, Gilles http://contrails.free.fr http://lapierre.skunkworks.free.fr ________________________________________________________________________________
From: "Email Security Gateway" <dralle(at)matronics.com>
Subject: Spam Quarantine Summary - (1)
Date: Sep 08, 2020
RGVhciAgYWVyb2VsZWN0cmljLWxpc3RAbWF0cm9uaWNzLmNvbSAsIHRoaXMgaXMgeW91ciBxdWFy YW50aW5lIHN1bW1hcnkgZnJvbSB0aGUgIEVtYWlsIFNlY3VyaXR5IEdhdGV3YXkgLgoKWW91IGhh dmUgIDEgIG1lc3NhZ2VzIGluIHlvdXIgc3BhbSBxdWFyYW50aW5lIGluYm94LiAgVG8gdmlldyB0 aGUgY29udGVudHMgb2YgeW91ciBxdWFyYW50aW5lIGluYm94IG9yIHRvIG1hbmFnZSB5b3VyIHNw YW0gcHJlZmVyZW5jZXMsIHBsZWFzZSBvcGVuIHRoZSBmb2xsb3dpbmcgbGluayBpbiB5b3VyIGJy b3dzZXI6Cmh0dHBzOi8vOTYuNjguMTcxLjIxL2NnaS1tb2QvaW5kZXguY2dpP2V0PTE2MDAwMzY1 MDImcGFzc3dvcmQ9YWZiMTQ1YmZjOTJjOWE5NDg0NjViZGFhYjNjMDUwMWQmdXNlcj1hZXJvZWxl Y3RyaWMtbGlzdCU0MG1hdHJvbmljcy5jb20mbG9jYWxlPWVuX1VTJnJvbGU9X25vZG9tYWlucwo R0lGODlhAwAXAMQAAABMhQBJggBUjQBQiQBelwBZkgBknQBwqABqogCCuwB8tAB2rgCOxwCIwQCg 2ACa0gCUzQC17QCy6gCu5gCq4gCl3czx/AAAAAAAAAAAAAAAAAAAAAAAAAAAAAAAAAAAACwAAAAA AwAXAAAFJqAlRqRkTiilVqzjPjAkM3RjJ7iiL/zhI0CDkEAsGAXIgRLADDhDADs R0lGODlhUgEfANU/AO7u7rW1tREREWtra+Xl5SIiIt3d3UxMTMzMzDo6OisrK3d3d0RERAkJCVVV Vfn5+Z2dnYSEhNDQ0BoaGsnJybKysqqqqsHBwTIyMurq6qWlpeDg4FlZWfHx8XJycvz8/EBAQLy8 vNTU1GZmZvLy8paWlnl5eX19fTU1NaGhoa2trfT09JKSkuzs7FFRUcTExGFhYaioqIqKipubm46O jtra2ri4uF5eXkdHR7u7u4iIiJmZmYmJiff39wAAAP///yH/C1hNUCBEYXRhWE1QPD94cGFja2V0 IGJlZ2luPSLvu78iIGlkPSJXNU0wTXBDZWhpSHpyZVN6TlRjemtjOWQiPz4gPHg6eG1wbWV0YSB4 bWxuczp4PSJhZG9iZTpuczptZXRhLyIgeDp4bXB0az0iQWRvYmUgWE1QIENvcmUgNS42LWMwNjcg NzkuMTU3NzQ3LCAyMDE1LzAzLzMwLTIzOjQwOjQyICAgICAgICAiPiA8cmRmOlJERiB4bWxuczpy ZGY9Imh0dHA6Ly93d3cudzMub3JnLzE5OTkvMDIvMjItcmRmLXN5bnRheC1ucyMiPiA8cmRmOkRl c2NyaXB0aW9uIHJkZjphYm91dD0iIiB4bWxuczp4bXBNTT0iaHR0cDovL25zLmFkb2JlLmNvbS94 YXAvMS4wL21tLyIgeG1sbnM6c3RSZWY9Imh0dHA6Ly9ucy5hZG9iZS5jb20veGFwLzEuMC9zVHlw ZS9SZXNvdXJjZVJlZiMiIHhtbG5zOnhtcD0iaHR0cDovL25zLmFkb2JlLmNvbS94YXAvMS4wLyIg eG1wTU06T3JpZ2luYWxEb2N1bWVudElEPSJ4bXAuZGlkOmM2OWUzODFmLTcwNDItNGE5Yy04YWYw LWM0ZjIyNDYxZmY5NiIgeG1wTU06RG9jdW1lbnRJRD0ieG1wLmRpZDpBNDFGNzI0M0I3QUIxMUU1 OEFDRkMwODE5RTA3NUM1RSIgeG1wTU06SW5zdGFuY2VJRD0ieG1wLmlpZDpBNDFGNzI0MkI3QUIx MUU1OEFDRkMwODE5RTA3NUM1RSIgeG1wOkNyZWF0b3JUb29sPSJBZG9iZSBQaG90b3Nob3AgQ0Mg MjAxNSAoTWFjaW50b3NoKSI+IDx4bXBNTTpEZXJpdmVkRnJvbSBzdFJlZjppbnN0YW5jZUlEPSJ4 bXAuaWlkOmEwMGM3MmJmLTU1NTMtNDNmMC1iNDExLTRmNjA3NWE3NjhhYiIgc3RSZWY6ZG9jdW1l bnRJRD0iYWRvYmU6ZG9jaWQ6cGhvdG9zaG9wOmI5YzU1Mzc3LTllYmItMTE3Ny05MGE4LThiNWNk OWYxZjMzMiIvPiA8L3JkZjpEZXNjcmlwdGlvbj4gPC9yZGY6UkRGPiA8L3g6eG1wbWV0YT4gPD94 cGFja2V0IGVuZD0iciI/PgH//v38+/r5+Pf29fTz8vHw7+7t7Ovq6ejn5uXk4+Lh4N/e3dzb2tnY 19bV1NPS0dDPzs3My8rJyMfGxcTDwsHAv769vLu6ubi3trW0s7KxsK+urayrqqmop6alpKOioaCf np2cm5qZmJeWlZSTkpGQj46NjIuKiYiHhoWEg4KBgH9+fXx7enl4d3Z1dHNycXBvbm1sa2ppaGdm ZWRjYmFgX15dXFtaWVhXVlVUU1JRUE9OTUxLSklIR0ZFRENCQUA/Pj08Ozo5ODc2NTQzMjEwLy4t LCsqKSgnJiUkIyIhIB8eHRwbGhkYFxYVFBMSERAPDg0MCwoJCAcGBQQDAgEAACH5BAEAAD8ALAAA AABSAR8AAAb/wJ9wSCwaj8fKKYNsOp/QqHRKrVqv2Kx2y+16nS+GL/Qtm8/otHrNbldFDp+P4q7b 7/i8fn98mOQ+AXyDhIWGh4MWBYA0iI6PkJGSTRlxgDCTmZqbnGwxE4A+Cp2kpaanTh6hPg0bqK+w sWsfK1YECas+KbK8vb5XEjlVL6CrLr/IycpEPBBUFQ25AgDLUz7V2JAMzlIauXIz2VHX4uWDHQ0q 3d+i5k/k7vF2IT7qUBXsPsJIFzoe//9KEJhEwMYHK/COAMjBsGFDal4WIhCCIAfEIwZ26NCxDw/D Lzk26jCAiocPDVAQ5EPhBIcPEDBBJADFDRIEHx0QOsmRryMX/54MhIjxSWTBqgIT78gZshFLjkWh GFx8EpKonhs+WDwBUOwbyiYgWBL5AMOHiEgaBJDQ2YSnDwZw4zJI2gUBgwVC9R3RIWeEjgWLBJC0 A3fIUisW5BRYoGOEHAxTm/DVUQiFjwFPxLAT0MMJAwxHfJhQ82DglA+mVahli8TtnaFHBPigLATA Irx7DlMBIHsEEQSyaT+ZTOiB7KBNZOTzwSEzaCM+ZAgxoKA6gxdCLET4AfgFAcwlCmhYUV3B9SEA OBRQUID2gHBDENzI+WPA+gIpckxY+wFE9QRfNZGQEa4hsREAOsC1ww8ALKAgEQjC5cCC0+lAIWzQ +WABETvchf9eggzoEJkFDjAw4RAGWEhEUz9oZAACIwTVVEhybIQAixThyJQPBUS2A48clhgiRAm+ xdEQO5Q4wmAhUfiDBTpsKIRG++QQ412DQSlldlHWIMcEBx3h5XJaeSYWEQMIwMQPJATgpmMX/GCC Ag4UUAIAFUwwwAQ6bPCBDW/6gB0JEygQggQ3YSZAc0MkthYI6YhggX8YrPVDCG6e4AN8SAxYRIGh vRXKCLIBgpttqyAHVF5W/YCBDwKMdAQCUMkhAF2OhfLcqoaRI4YOpf6wFF+hGCDHYD845lsRYuAG 4UdCWALINMKGQhkAmgGyoErwvPpctRMRG4owPxZAxCI72AD/CF3MLudDnE5w0AAHDjjAgQsTTBDZ ECgElYIPxwhxQQMo1GIECAf84IAARKiAUwKYDVEBw/is+cMHDXBmhAcTOOEpEW7JBRduj5FE7AjU 8MUwd2+RZMCrE/GKYRHAAVLACE7+8GvKQLYIqzAvz/YDr0IcJoYAGFggjG66/TqEbFbJ0SoRiRUg DACO0UacEHwtObRsJC1yNSDUVM2gHBTy5QCDsiWl0jQ3ySGdEeq6e1a8ArigtwMHJLDYER0oAMIP JeA0xAWCAk6nsLsQEcAKGEQsxMQ/HBDwEJp2RkQPHHQs4E7sIHdsr8ge5uIQRgkjs14YAaYrNSot G62GOtMu/0QOHhJdbV7UFg2Pbj8+Z3aGHbkFiOpHZif0D1v/IIC5O+JlFF6JOSbM9D+k6CQAbwnh GG5G+RYBIGcSYZm7EjwhkxGIe0DRAA7ggMO8P8jg+eQN0PcDAvDjwEADC+iAD5B1Lsn9IAAC+IAC tkM1AdRiAx7gwAEYMIHyQQd0+nBIDpKim931Cj1QipHUhta92k0NRUnqC/N4JJdFUKaDICuh7/Li LA/akHsDTNbyiiCbLdkFLlAbQkgcFByuLY8nApDLq4LCE9AYxViUgVl8NsKARSBneGL7gSoAUYMi IM5dPoiBc45glB/QwAc44IENNsCBwckAepPr3fjSaAMDOP8ABugg4BAKYEAEKpCBQ1DND9SFghNU QAQ8gGNoMCighHTwMBaQDQb88irVlXBmQlTRiobFQpEtCIZClKEHnfbBGQ7he87LoRHiIDsiVDJ7 iyiAAxqzvK0hUWQeSiUAMAAaDDDAWHA8Woj4gpwfLMICKoFeruTgPiJIazmXQ8JnjsCCBnipJj/A gRsVicAWZM8H2MTBMWw3hAFsAAPNFMIMGpDNaP4gAg14gJ5WtLJOMbJTjsynELLIKhIiB5NGVCQJ r/GjGhIBbChSke6MtkMbdrBqiSlmEX4kmCIYSy9Z25HWlmesbxnBMQVl2Y+W9aNiEi18RqENB0LR AIP9IAP/0QDjHMx0hAIwYAM+sNQPBNicNzpOAN6kgOFqw5xs1vMH3gAADhJABAVcIzEu/UED9BWI gwq0CB8LZVZtyNXDyAEivNHL6qxyUZTVJg5BMVbvarcgZQ0hDpR5G5EYKhyH+mAqi3hVzorgLbog 4FV6EcOWSMkXZ/FzhctKzKs2VDXbFXZHxVTrIgbjklBgQgiFk6kcsIMEEExABBIILQI08KqzPE8E BIDAARoQFE1RzQdMkGcBUAuB/yWsBQ3AAAJSK5onZYUABrhBvnKCAgGEgAA58Nso4uCdABxAAKP4 XFt60lV9uopHf4GKAwww1r3YCohySIpRBMAYb03nOEXq/11eaUlXIuhGksU0Cqz2FZ9S+fJVAojD 9WBV3scIQ2UMWFBifOCXOFRUCKWiBg7rOeAC28pJgC3mwgRAYQE0IH0/gEGFN8zhCsNqBg84wgAy xmEQYNgAxZ0ABnKgAerdgAgUcAB9UCwAFQdAAycQQgYoWIACNAKzz8OACTYwAG/+QLgFwAANvvOB D7jgeQqYAQVaecHpsmNp1jUlrQDhF6l19wg7qNVb2AUsLl9ky4rxa6kE8CNfNRR4wZqOCp2AtVAg JTFXy1aAa8QgzdBGEYCYizN98K1XtVK+PIqkDNu8V3msYatoqAhEEMCuKXDXIq3BtBEqUunbaVoL 3Drhp0NyoEcUkXo6ot7fqa8gkdp8WmX0dTQaIC3rLVAjDh6VhYJvU2s30LrXWMhWqk0xrVgD+wu/ PnYVHOSAYZsCLl4jQhAAADs R0lGODlhBwAXANUAAABMhQBJggJNhjprkXiQoouapgBUjQBQiRlejwBelwBZkgBknQBwqABqogCC uwB8tAB2rgCOxwCIwQCg2ACa0gCUzQC17QCy6gCu5gCq4gCl3Qa06iS56lTN88/x/K7n+Mzx/OH3 /f///7CwsAAAAAAAAAAAAAAAAAAAAAAAAAAAAAAAAAAAAAAAAAAAAAAAAAAAAAAAAAAAAAAAAAAA AAAAAAAAAAAAAAAAAAAAAAAAAAAAAAAAAAAAAAAAAAAAACwAAAAABwAXAAAGXkCRUBQCgUSejmVp +WwuUCgHQ61mrtiMZsvVTL7gCWVMplTO6EpkzY5I3nCJY053PO74B2TPhzD+gAwNg4QNC4eICwmL jAkKj5AKBpOUBggHmJkEAgCdnSMFAwGjAUEAOw= R0lGODlhBwAXANUAAABMhQBJggJNhjprkXiQoouapgBUjQBQiRlejwBelwBZkgBknQBwqABqogCC uwB8tAB2rgCOxwCIwQCg2ACa0gCUzQC17QCy6gCu5gCq4gCl3QOz6iS56lTN88/x/K7n+Mzx/OH3 /f///7CwsAAAAAAAAAAAAAAAAAAAAAAAAAAAAAAAAAAAAAAAAAAAAAAAAAAAAAAAAAAAAAAAAAAA AAAAAAAAAAAAAAAAAAAAAAAAAAAAAAAAAAAAAAAAAAAAACwAAAAABwAXAAAGXkAQKCQqiixIS8cj ujidmw9mSuVkrtiMZsvVTL7gCWVMplTO6EpkzY5I3nCJY053PO74B2TPhzD+gAwNg4QNC4eICwmL jAkKj5AKBpOUBgeXmAgAm5sCBAGgAQMFI0EAOw= ________________________________________________________________________________
Subject: Re: AV 30 Wiring
From: "farmrjohn" <faithvineyard(at)yahoo.com>
Date: Sep 08, 2020
Bump, primarily to re-ask the question about the diagram for the certified version with the "X" in the shield symbol. Thanks, John Read this topic online here: http://forums.matronics.com/viewtopic.php?p=498276#498276 ________________________________________________________________________________
Subject: Re: Alternator/Strobe whine
From: Rick Beebe <rick(at)beebe.org>
Date: Sep 08, 2020
I'll be back at the plane on Saturday and I will do some controlled tests. The noise pitch is definitely in sync with the strobes. When I turn the main alternator off the noise goes away completely. My memory is that it also goes away when I turn the strobes off (main alternator on), but I'm not sure if there's a residual whine. --Rick On 9/8/2020 4:41 PM, Robert L. Nuckolls, III wrote: > Alternator, strobe or both? > > Alternator whine is almost always a ground loop issue. > Pitch of whine will go up and down with engine rpm. > It often gets louder when alternator loads are highest > like right after starting engine (recharging battery) > and all loads turned on. > > Strobe whine pitches up and down in synchronization > with flashes from the strobes and is generally of > constant volume. > > Which one(s) disappear when appliance is OFF. > Alternator, strobe or both? > > > Bob . . . > ________________________________________________________________________________
Date: Sep 08, 2020
From: "Robert L. Nuckolls, III" <nuckolls.bob(at)aeroelectric.com>
Subject: Re: AV 30 Wiring
At 06:21 PM 9/8/2020, you wrote: > >Bump, primarily to re-ask the question about the diagram for the >certified version with the "X" in the shield symbol. Thanks, John I would infer that to indicate twisted conductors . . . but then I think one would be hard pressed to find a shielded pair that was not twisted. Bob . . . ________________________________________________________________________________
From: Paul Eckenroth <N509RV(at)eckenroth.com>
Date: Sep 09, 2020
Subject: Re: Revmaster 'dual' alterantors
I think that there might be a misunderstanding of the Revmaster architecture in that I don't believe that the windings for the two generators are in tandem but are a series of 5 coils each for two generators. I have enclosed a picture of the stator from my friend's engine. He was running one generator at the time and you can see how severely it was destroyed. The Phenolic base was also destroyed. The second generator coils don't look bad but they are also ruined. To me one of the interesting things is that the ignition coils are still functional. My primary purpose in starting this thread was to hopefully end up with a definite fix for the problem. Trying this or that is not practicable due to the destructive nature of the overheating. The stator is over $500 to replace and requires pulling the engine to do so. I'll only really know that the problem is fixed when the smoke stays in the coils. It seems to me that the most important change will be to use the B&C AVC1 volltage regulator since it is designed for firewall forward and can be adjusted down to 13.7 v which is the minimum voltage that will fully charge the EarthX battery. With the voltage reduction should come current reduction. What I don't know (and hopefully the educated on the list can explain) is whether restricting the voltage will also restrict the current. Can the generator still create abnormal current and therefore heat. I will try and incorporate thermocouples in with the coils but I'm not sure how much advance warning they can give. So, with the new voltage regulator set at 13.7 can the generator still pump out current based on engine speed and battery resistance or is the current capped since the voltage is restricted. I'm trying to determine if I have a good chance of success here. Does anybody want to redesign the generator to make it more robust. Paul On Mon, Aug 31, 2020 at 12:25 PM Robert L. Nuckolls, III < nuckolls.bob(at)aeroelectric.com> wrote: > At 10:39 AM 8/31/2020, you wrote: > > > Using both windings is a good idea. > What about the phase angle between the two windings? Do we know what it > is? > > > No . . . > > Suppose the two windings are 90 degrees out of phase or some other unknown > angle. > How will that affect the proposed circuit? Will the peak AC voltage be > higher? > > > Good question. > > I doubt they are polyphase . . . you gotta go to a > lot of mechanical fuss to achieve it. It's a function > of stator/magnet ring geometry. They might be different > but it's doubtful. Issue 80 of Contact! magazine has > some nice pictures of the stator arrangement: > > http://www.contactmagazine.com/Issue80/Issue-80L.pdf > > > Do capacitors help very much to smooth the DC voltage in a power circuit? > > > They can . . . it's all about current/capacitance ratio > versus frequency. If a capacitor is necessary, it will > be dictated by the CCCV down converter tolerance for > ripple. > > It would help to have a Revmaster alternator to play with. > > > You got that right! > > > Bob . . . > ________________________________________________________________________________
Subject: Re: Cigarette lighter socket to power a Garmin Aera
660
From: "allenmaris" <allenmaris(at)gmail.com>
Date: Sep 09, 2020
Argonaut36 wrote: > I have recently purchased a Garmin AERA 660 portable GPS navigator. The unit holder to be used in the airplane is equipped with a cigarette lighter connector plug (outfitted with a 3A fast fuse). > > Could somebody recommend a good quality cigarette lighter connector socket that I can connect to the electrical system of my airplane? I would like to buy a socket that can be installed using an Adel clamp. > > Thanks I've heard good things about these on VAF. https://www.amazon.com/Blue-Sea-Systems-Charge-Charger/dp/B001U4ZZPK/ref=sr_1_1?ie=UTF8&qid=1521418596&sr=8-1&keywords=blue%2Bsea%2Busb&th=1 Read this topic online here: http://forums.matronics.com/viewtopic.php?p=498285#498285 ________________________________________________________________________________
Subject: Re: Revmaster 'dual' alterantors
From: "user9253" <fransew(at)gmail.com>
Date: Sep 09, 2020
Current is not capped. Current is determined by the load. The load takes what it wants. Some loads such as batteries and resistance type loads will take less current when the voltage drops. Loads with switching power supplies will take more current as the voltage drops because they operate with constant power. Many modern avionics have switching power supplies. Lowering the voltage will reduce the EarthX battery charging current. -------- Joe Gores Read this topic online here: http://forums.matronics.com/viewtopic.php?p=498286#498286 ________________________________________________________________________________
Subject: Re: Revmaster 'dual' alterantors
From: Charlie England <ceengland7(at)gmail.com>
Date: Sep 09, 2020
On 9/9/2020 12:14 PM, user9253 wrote: > > Current is not capped. Current is determined by the load. The load takes > what it wants. Some loads such as batteries and resistance type loads will > take less current when the voltage drops. Loads with switching power > supplies will take more current as the voltage drops because they operate with > constant power. Many modern avionics have switching power supplies. > Lowering the voltage will reduce the EarthX battery charging current. > > -------- > Joe Gores > Ohm's Law sets the current. The battery has a source impedance when supplying energy, and it has a load impedance when being charged. The battery mfgr should be able to tell you the battery's load impedance while charging (ask for worst case). That number, and the charge voltage, will give you the charging current. Off-the-shelf 'smart' battery chargers manage current by adjusting voltage 'on the fly', tailored to optimum charge rates for a particular battery. If I were dealing with a 'locked in' flaky design like the Revmaster, I'd be looking at using a constant current source between the alternator-regulator and the rest of the airframe. With lithium tech being found everywhere now, constant voltage/constant current (misleading term...) battery chargers are everywhere at very reasonable prices. By inserting one between the regulator's output and the airframe's bus, you can set the current limit in the charging module at a point lower than the 'at risk' point for your alternator. BTW, with a $500 price point, I'd be finding the original source for that armature. That style alternator is used in dozens (hundreds?) of products. I'd bet that with a little research, you could find that model from the actual mfgr for a lot less money. If not, I'd be finding a way to install an external alternator. I wouldn't be able to stand it. :-) Charlie -- This email has been checked for viruses by Avast antivirus software. https://www.avast.com/antivirus ________________________________________________________________________________
Subject: Re: AV 30 Wiring
From: "donjohnston" <don@velocity-xl.com>
Date: Sep 09, 2020
I would agree. On the second drawing, the only place TPS is shown is on those wires with the "X". Although it does appear the shields are grounded in two places. I thought the idea was to always ground the shield at one end. Read this topic online here: http://forums.matronics.com/viewtopic.php?p=498289#498289 ________________________________________________________________________________
Subject: Re: Lycoming EIS wirig recommendation
From: "donjohnston" <don@velocity-xl.com>
Date: Sep 09, 2020
It's a SureFly. https://www.lycoming.com/eis > The Lycoming EIS was developed by SureFly Partners, Ltd. with input from Lycomings engineering teams to meet Lycomings exacting specifications. Read this topic online here: http://forums.matronics.com/viewtopic.php?p=498290#498290 ________________________________________________________________________________
Date: Sep 09, 2020
From: Ernest Christley <echristley(at)att.net>
Subject: Re: Revmaster 'dual' alterantors
Well, for $500 you could rewind the stator by hand, and use a heavier gauge wire. I don't understand why this would be so expensive. You can get the entire c harging system for a Harley for around $150. Sent from AT&T Yahoo Mail on Android On Wed, Sep 9, 2020 at 6:45, Paul Eckenroth wrote: I think that there might be a misunderstanding of the Revmaster architect ure in that I don't believe that the windings for the two generators are in tandem but are a series=C2-of 5 coils each for two generators.=C2- =C2 -I have enclosed a picture of the stator from my friend's engine.=C2- H e was running one generator at the time and you can see how severely it was destroyed.=C2- The Phenolic=C2-base was also destroyed.=C2- The seco nd generator coils don't look bad but they are also ruined.=C2- To me one of the interesting things is that the ignition=C2-coils are still functi onal.=C2-=C2-My primary purpose in starting this thread was to hopefull y end up with a definite fix for the problem.=C2- Trying this or that is not practicable due to the destructive nature of the overheating.=C2- The stator is over $500 to replace and requires=C2-pulling the engine to do so.=C2- I'll only really know that the problem is fixed when the smoke st ays in the coils.=C2-It seems to me that the most important change will b e to use the B&C AVC1 volltage regulator since it is designed for firewall forward and can be adjusted down to 13.7 v=C2- which is the minimum volta ge that will fully charge the EarthX battery.=C2- With the voltage reduct ion should come current reduction.=C2- What I don't know (and hopefully t he educated on the list can explain) is whether restricting the voltage wil l also restrict the current.=C2- Can the generator still create abnormal current and therefore heat. I will try and incorporate thermocouples in wit h the coils but I'm not sure how much advance warning they can give.So, wit h the new voltage regulator set at 13.7 can the generator still pump out cu rrent based on engine speed and battery resistance or is the current capped since the voltage is restricted. I'm trying to determine if I have a good chance of success here.Does anybody want to redesign the generator to make it more robust. Paul On Mon, Aug 31, 2020 at 12:25 PM Robert L. Nuckolls, III <nuckolls.bob@aero electric.com> wrote: At 10:39 AM 8/31/2020, you wrote: --> AeroElectric-List messageposted by: "user9253" Using both windings is a good idea. What about the phase angle between the two windings?=C2- Do we knowwhat i t is? =C2- No . . . Suppose the two windings are 90degrees out of phase or some other unknown a ngle.=C2- How will that affect the proposed circuit?=C2- Will the peak AC voltagebe higher? =C2- =C2- Good question. =C2- I doubt they are polyphase . . . you gotta go to a =C2- lot of mechanical fuss to achieve it. It's a function =C2- of stator/magnet ring geometry. They might be different =C2- but it's doubtful.=C2- Issue 80 of Contact! magazine has =C2- some nice pictures of the stator arrangement: http://www.contactmagazine.com/Issue80/Issue-80L.pdf Do capacitors help very much tosmooth the DC voltage in a power circuit? =C2- They can . . . it's all about current/capacitance ratio =C2- versus frequency. If a capacitor is necessary, it will =C2- be dictated by the CCCV down converter tolerance for =C2- ripple. It would help to have aRevmaster alternator to play with. =C2- You got that right! =C2- Bob . . . ________________________________________________________________________________
Date: Sep 10, 2020
From: "Robert L. Nuckolls, III" <nuckolls.bob(at)aeroelectric.com>
Subject: Re: Revmaster 'dual' alterantors
At 08:43 AM 9/9/2020, you wrote: >I think that there might be a misunderstanding of the Revmaster >architecture in that I don't believe that the windings for the >two generators are in tandem but are a series of 5 coils each >for two generators. Hmmmm . . . if that's the case, then it's probably accurate to call it a 'dual' alternator system . . . or perhaps 1/2 + 1/2 alternators system. The energy generation potential for these alternators is a function of the magnetics/mechanics. Magnet strength, magnetic cross-section of the stator and rotor, permeability of the steels, air gaps between moving parts, rates of change for magnetic lines of force acting on the windings (rpm) and a few other little details like losses in the steels and windings. Bottom line is that by dividing the two windings between stator 'halves', the ability to do the mechanical to electrical energy conversion for each alternator is about 1/2 that of the potential whole. It would seem more practical to have all the stator magnetics to participate in the energy conversion efforts using fewer turns per pole of heavier wire. One might then achieve more robustness and more energy output at the same time. > I have enclosed a picture of the stator from my friend's engine. I don't see that image . . . how did you 'enclose' it? >It seems to me that the most important change will be to use the B&C AVC1 >volltage regulator since it is designed for firewall forward and can be >adjusted down to 13.7 v which is the minimum voltage that will fully charge >the EarthX battery. With the voltage reduction should come current reduction. Probably not. Keep in mind that as you rotate of the runway into the blue, there are two kinds of loads on your alternator. Battery recharge + system running loads. The TOTAL of these two values is exceeding the alternator's limitations for thermal management and has not much to do with the bus voltage set-point. The fact that an EarthX battery seems to exacerbate the failure rate only speaks to that battery's lower internal resistance and willingness to accept charge at a lower voltage than SLVA. I.e. the LiFEPO4 battery is NOT ROOT CAUSE of the unhappy condition . . . only a slightly greater stress on the alternator seems to exacerbate failure rates. Adjusting the voltage down by exploiting the features of the AVC1 might reduce the failure rate but I kinda doubt it. I've got a fully discharged ETX36D on the bench that I'll try to plot the recharge profile when impressed with a 13.7V set-point. But I'm confident that an adjustable regulator will not 'fix' fundamental shortcomings of the alternator. >What I don't know (and hopefully the educated on the list can explain) >is whether restricting the voltage will also restrict the current. Maybe a little >Can the generator still create abnormal current and therefore heat. You betcha . . . but system loads are much more significant than characteristics of the battery. >I will try and incorporate thermocouples in with the coils but I'm >not sure how much advance warning they can give. Lots of warning. Those wires don't smoke in tens of milliseconds like fuses. The insulation degrades over time . . . probably hours of operation at temperatures exceeding design limits of the INSULATION. I think this system would benefit greatly by some extensive testing on a drive stand to optimize wire size and turns against instrumented performance measurements. The SD8 from B&C got tested with about a half dozen exploratory configurations before the final configuration went to production. I'd like to see the performance characteristics of the existing stator wound with fewer turns of fatter wires on ALL available energy production poles. Bob . . . ________________________________________________________________________________
Date: Sep 10, 2020
From: "Robert L. Nuckolls, III" <nuckolls.bob(at)aeroelectric.com>
Subject: Re: Revmaster 'dual' alterantors
At 05:25 PM 9/9/2020, you wrote: >Well, for $500 you could rewind the stator by hand, and use a >heavier gauge wire. THAT would be a really good experiment! Bob . . . ________________________________________________________________________________
Date: Sep 10, 2020
From: "Robert L. Nuckolls, III" <nuckolls.bob(at)aeroelectric.com>
Subject: Re: AV 30 Wiring
At 04:28 PM 9/9/2020, you wrote: > >I would agree. On the second drawing, the only place TPS is shown >is on those wires with the "X". > >Although it does appear the shields are grounded in two places. I >thought the idea was to always ground the shield at one end. Yes . . . but. If the design goal for shielding wires is to BREAK and electro-static coupling mode between potential antagonist/victim wires, then yes . . . connect shield to one ground only. SOME installations exploit the shield as part of the energy management goal. One example can be seen in the Z-figures where I recommend GROUNDING a p-lead shield at the magneto end then using the shield to provide a GROUND PATH for the switch at the other end. Some authors have suggested that p-leads need to be shield grounded at BOTH ends which has no foundation in physics and raises risks for some ground loop issues. I've also used the shield as a return path for incandescent bulbs illuminating a whisky compass. Providing a coaxially concentric out-and-back path for bulb currents cancels magnetic fields around the wire and prevents illumination induced heading errors. The best thing to do is follow the manufacturer's installation instructions. I cannot assert they are always 'golden' but there just might be a good reason for doubling up shield terminations. Bob . . . ________________________________________________________________________________
Subject: Re: Revmaster 'dual' alterantors
From: "dj_theis" <djtheis58(at)gmail.com>
Date: Sep 10, 2020
nuckolls.bob(at)aeroelect wrote: > At 08:43 AM 9/9/2020, you wrote: > > > I think that there might be a misunderstanding of the Revmaster > > architecture in that I don't believe that the windings for the > > two generators are in tandem but are a series of 5 coils each > > for two generators. > > Hmmmm . . . if that's the case, then it's probably > accurate to call it a 'dual' alternator system . . . > or perhaps 1/2 + 1/2 alternators system. > > > > > I have enclosed a picture of the stator from my friend's engine. > > I don't see that image . . . how did you 'enclose' > it? > > Images as attachments of the R2300 from when I first received my engine in 2017. There are two separate stator windings, 5 coils on each stator with relatively large wires (12 AWG ?). Next time I pull the engine, I'll mic the wires on the coils. Note, setting this up as a center tap as suggested should be straight forward. Again I have to state that I think the average current with your center tap arrangement will be the same as the OEM half wave R/R with both sides enabled. > > I think this system would benefit greatly by some > extensive testing on a drive stand to optimize > wire size and turns against instrumented performance > measurements. The SD8 from B&C got tested with about > a half dozen exploratory configurations before the > final configuration went to production. > > I'd like to see the performance characteristics > of the existing stator wound with fewer turns > of fatter wires on ALL available energy > production poles. > > > > Bob . . . I will try to obtain some fundamental characteristics of the alternators within the next 30 days. (i.e. short circuit current, open circuit voltage at starter rpm, inductance...). Let me point out the obvious,... "redesigning this system puts the builder in a situation of complete ownership of the problem as it is doubtful Revmaster will support a modified system." The OBAM pilot takes on that responsibility (or should he/she does) regardless of the OEM support, so maybe that is not worth mentioning. Adding an external alternator is not obvious (to me) where it would mount but earlier versions of the engine had external alternators (I believe), so the design components could be available Revmaster. I amazed (concerned) that with the close proximity of the CDI power coils and ignition electronics that there are no reported failures of that system, related to the alternator failures. makes me want to consider returning to 2 mags and an external alternator for the R2300. Again, maybe easier said the executed but certainly a design that was once available. One thing that puzzles me though. At least one failure of BOTH alternators, as reported by Eric or Paul, occurred with only one alternator activated. How is it possible to overheat both sides simultaneously (through heat conduction I have to assume) without damaging the ingnition coils that sit between the two alternators ?? (Molex connectors and wires in my photo from the two ignition coils and the timing signals are shown). I have to suspect there is another heat source or cause that is contributing to these PMA failures. Engine heat? Dan Theis -------- Scratch building Sonex #1362 Read this topic online here: http://forums.matronics.com/viewtopic.php?p=498299#498299 Attachments: http://forums.matronics.com//files/20200910_154510000_ios_107.jpg http://forums.matronics.com//files/20200910_152443000_ios_116.jpg ________________________________________________________________________________
Subject: Re: Revmaster 'dual' alterantors
From: "user9253" <fransew(at)gmail.com>
Date: Sep 10, 2020
There are 4 black screws in the picture that look like they might hold a cover on. Is it possible to pipe pressurized air into that cover to cool the coils? If the coils were kept cool, their ampacity would be greatly increased. -------- Joe Gores Read this topic online here: http://forums.matronics.com/viewtopic.php?p=498300#498300 ________________________________________________________________________________
Date: Sep 10, 2020
From: FRANK MCDONALD <frank3phyl(at)comcast.net>
Subject: Z101 RevB
Is there a way to get a simple pdf version of the newest revision of Z101? I am not a CAD person, though I admire those that are. Thanks much. Frank ________________________________________________________________________________
Subject: Re: Z101 RevB
From: Werner Schneider <glastar(at)gmx.net>
Date: Sep 10, 2020
Yes Frank http://www.aeroelectric.com/PPS/Adobe_Architecture_Pdfs/ there you find all as PDF's On 10.09.2020 20:42, FRANK MCDONALD wrote: > Is there a way to get a simple pdf version of the newest revision of > Z101? I am not a CAD person, though I admire those that are. > Thanks much. > Frank ________________________________________________________________________________
From: Paul Eckenroth <N509RV(at)eckenroth.com>
Date: Sep 10, 2020
Subject: Re: Revmaster 'dual' alterantors
Hopefully here is the image of the stator that I thought was sent previously. Paul On Thu, Sep 10, 2020 at 12:51 PM dj_theis wrote: > > > nuckolls.bob(at)aeroelect wrote: > > At 08:43 AM 9/9/2020, you wrote: > > > > > I think that there might be a misunderstanding of the Revmaster > > > architecture in that I don't believe that the windings for the > > > two generators are in tandem but are a series of 5 coils each > > > for two generators. > > > > Hmmmm . . . if that's the case, then it's probably > > accurate to call it a 'dual' alternator system . . . > > or perhaps 1/2 + 1/2 alternators system. > > > > > > > > > I have enclosed a picture of the stator from my friend's engine. > > > > I don't see that image . . . how did you 'enclose' > > it? > > > > > > > Images as attachments of the R2300 from when I first received my engine in > 2017. > There are two separate stator windings, 5 coils on each stator with > relatively large wires (12 AWG ?). Next time I pull the engine, I'll mic > the wires on the coils. > Note, setting this up as a center tap as suggested should be straight > forward. Again I have to state that I think the average current with your > center tap arrangement will be the same as the OEM half wave R/R with both > sides enabled. > > > > > > I think this system would benefit greatly by some > > extensive testing on a drive stand to optimize > > wire size and turns against instrumented performance > > measurements. The SD8 from B&C got tested with about > > a half dozen exploratory configurations before the > > final configuration went to production. > > > > I'd like to see the performance characteristics > > of the existing stator wound with fewer turns > > of fatter wires on ALL available energy > > production poles. > > > > > > > > Bob . . . > > > I will try to obtain some fundamental characteristics of the alternators > within the next 30 days. (i.e. short circuit current, open circuit voltage > at starter rpm, inductance...). > > Let me point out the obvious,... "redesigning this system puts the builder > in a situation of complete ownership of the problem as it is doubtful > Revmaster will support a modified system." The OBAM pilot takes on that > responsibility (or should he/she does) regardless of the OEM support, so > maybe that is not worth mentioning. > > Adding an external alternator is not obvious (to me) where it would > mount but earlier versions of the engine had external alternators (I > believe), so the design components could be available Revmaster. I amazed > (concerned) that with the close proximity of the CDI power coils and > ignition electronics that there are no reported failures of that system, > related to the alternator failures. makes me want to consider returning to > 2 mags and an external alternator for the R2300. Again, maybe easier said > the executed but certainly a design that was once available. > > One thing that puzzles me though. At least one failure of BOTH > alternators, as reported by Eric or Paul, occurred with only one alternator > activated. How is it possible to overheat both sides simultaneously > (through heat conduction I have to assume) without damaging the ingnition > coils that sit between the two alternators ?? > > (Molex connectors and wires in my photo from the two ignition coils and > the timing signals are shown). > > I have to suspect there is another heat source or cause that is > contributing to these PMA failures. Engine heat? > > Dan Theis > > -------- > Scratch building Sonex #1362 > > > Read this topic online here: > > http://forums.matronics.com/viewtopic.php?p=498299#498299 > > > Attachments: > > http://forums.matronics.com//files/20200910_154510000_ios_107.jpg > http://forums.matronics.com//files/20200910_152443000_ios_116.jpg > > ________________________________________________________________________________
Subject: Re: Revmaster 'dual' alterantors
From: "dj_theis" <djtheis58(at)gmail.com>
Date: Sep 10, 2020
user9253 wrote: > There are 4 black screws in the picture that look like they might hold a cover > on. Is it possible to pipe pressurized air into that cover to cool the coils? > If the coils were kept cool, their ampacity would be greatly increased. It is possible and I agree it's worth considering. I'll look at the CDI eletronics casting (which is what those screws hold in place) to see if there is a pathway to generate an inlet and egress for air. I don't have any good photos of the inside of that casting. Maybe I can pull that casting off without pulling the engine. I'll look tonight. Thanks for the suggestion. Dan Theis -------- Scratch building Sonex #1362 Read this topic online here: http://forums.matronics.com/viewtopic.php?p=498307#498307 ________________________________________________________________________________
Date: Sep 10, 2020
From: "Robert L. Nuckolls, III" <nuckolls.bob(at)aeroelectric.com>
Subject: Re: Revmaster 'dual' alterantors
At 03:18 PM 9/10/2020, you wrote: >Hopefully here is the image of the stator that I thought was sent previously. > >Paul Excellent . . . thank you. I agree, this definitely appears to be a PAIR of 5-pole alternators. Good data point. Bob . . . ________________________________________________________________________________
Subject: Re: Z101 RevB
From: "frank3" <frank3phyl(at)comcast.net>
Date: Sep 10, 2020
Thanks very much. Frank -------- Frank McDonald Kitfox S7 912S, Sensenich Composite 3 Blade Acworth, GA Read this topic online here: http://forums.matronics.com/viewtopic.php?p=498309#498309 ________________________________________________________________________________
From: Paul Eckenroth <N509RV(at)eckenroth.com>
Date: Sep 10, 2020
Subject: Re: Revmaster 'dual' alterantors
Dan I'll try and get a picture of that casting for you. Paul On Thu, Sep 10, 2020 at 4:54 PM dj_theis wrote: > > > user9253 wrote: > > There are 4 black screws in the picture that look like they might hold a > cover > > on. Is it possible to pipe pressurized air into that cover to cool the > coils? > > If the coils were kept cool, their ampacity would be greatly increased. > > > It is possible and I agree it's worth considering. I'll look at the CDI > eletronics casting (which is what those screws hold in place) to see if > there is a pathway to generate an inlet and egress for air. I don't have > any good photos of the inside of that casting. Maybe I can pull that > casting off without pulling the engine. I'll look tonight. > > Thanks for the suggestion. > > Dan Theis > > -------- > Scratch building Sonex #1362 > > > Read this topic online here: > > http://forums.matronics.com/viewtopic.php?p=498307#498307 > > ________________________________________________________________________________
Subject: Re: Revmaster 'dual' alterantors
From: "dj_theis" <djtheis58(at)gmail.com>
Date: Sep 11, 2020
N509RV(at)eckenroth.com wrote: > Dan > > I'll try andget a picture of that casting for you. > > > Paul > > Thanks Paul, I was able to get an image (attached) of the outside of the casting. With 5e CDI casting removed, does it appear that there is an air flow path that would help ventilate the PMA coils? I am imagining having a fitting to feed air from the top of the engine on one side (shown in the photo) and allow the exit air on the other side of the casting to exit into the lower engine area. Dan -------- Scratch building Sonex #1362 Read this topic online here: http://forums.matronics.com/viewtopic.php?p=498312#498312 Attachments: http://forums.matronics.com//files/c29bc5c2_f51d_4e86_8c7c_d7d5a3a18286_175.jpeg ________________________________________________________________________________
Subject: How do you figure C-Rating of a battery?
From: "rparigoris" <rparigor(at)hotmail.com>
Date: Sep 11, 2020
Hi Group I'm a bit confused how to figure C-rating of a battery. Here's my dilemma, here's a 5,000mA battery that say's it's good for 10C continuous and 15C burst: https://hobbyking.com/en_us/turnigy-sub-c-1-2v-5000mah-high-power-series-nimh-single-cell.html?queryID=3ed79812e155e159b8e96c63dba8378a&objectID=41335&indexName=hbk_live_magento_en_us_products Initial thoughts is just multiply 5 times 10 or 15. Then I read a review and the battery never made it to 5amp capacity and the capacity goes down the more amps you draw from it. Here's the review: https://lygte-info.dk/review/batteries2012/Turnigy%20Sub-C%205000mAh%20(Gray)%20UK.html If I wanted to know what the continuous C rating is for this battery, would you take the unrealistic too high 5aH capacity rating from the Mfg times 10? Take the ~3.5 20 amp discharge capacity rating from the review times 10? Or?? Now for another hard question, how would you figure out burst amp rating? Thx. Ron P. Read this topic online here: http://forums.matronics.com/viewtopic.php?p=498313#498313 ________________________________________________________________________________
From: Jared Yates <email(at)jaredyates.com>
Date: Sep 11, 2020
Subject: Re: How do you figure C-Rating of a battery?
Multiply 5000 by 10 for continuous, which is 50,000 milliamps, or 50 amps. On Fri, Sep 11, 2020 at 3:38 PM rparigoris wrote: > rparigor(at)hotmail.com> > > Hi Group I'm a bit confused how to figure C-rating of a battery. Here's my > dilemma, here's a 5,000mA battery that say's it's good for 10C continuous > and 15C burst: > https://hobbyking.com/en_us/turnigy-sub-c-1-2v-5000mah-high-power-series-nimh-single-cell.html?queryID=3ed79812e155e159b8e96c63dba8378a&objectID=41335&indexName=hbk_live_magento_en_us_products > Initial thoughts is just multiply 5 times 10 or 15. Then I read a review > and the battery never made it to 5amp capacity and the capacity goes down > the more amps you draw from it. Here's the review: > https://lygte-info.dk/review/batteries2012/Turnigy%20Sub-C%205000mAh%20(Gray)%20UK.html > If I wanted to know what the continuous C rating is for this battery, > would you take the unrealistic too high 5aH capacity rating from the Mfg > times 10? Take the ~3.5 20 amp discharge capacity rating from the review > times 10? Or?? > Now for another hard question, how would you figure out burst amp rating? > Thx. Ron P. > > > Read this topic online here: > > http://forums.matronics.com/viewtopic.php?p=498313#498313 > > ________________________________________________________________________________
From: <billhuntersemail(at)gmail.com>
Subject: Nav Light CB Rating
Date: Sep 11, 2020
What is the suggested CB rating of a navigation light circuit on a 14 VDC system? The airplane is fiberglass so the Nav light circuit is 53 feet from the CB to the switch to the Nav light fixture and back to the forest of grounds and it is wired with 18 AWG wire? My current system has a 5 AMP CB that services only the two navigation light bulb fixtures that each has two bulbs (total of 4 incandescent light bulbs). I do not know the manufacturer of the light fixture so I do not have any idea what these light bulbs draw. It was suggested that I install a 5 AMP CB and when I turn on the switch the lights all illuminate and they stay on for about 20 minutes and then the CB pops. On hotter days the time before the CB pops is more like 10 minutes so it seems that the thermal load on the current 5 AMP CB is too great. Of course I can upgrade the CB but I would prefer to know if a 7 AMP or 10 amp is preferred so that is why I am reaching out to the experts on the subject. As always THANKS for your help!!!. ________________________________________________________________________________
Date: Sep 11, 2020
From: Jeff Luckey <jluckey(at)pacbell.net>
Subject: Re: Nav Light CB Rating
Bill, If you get the numbers from those bulbs you can look them up and find their current rating.=C2- That way you know for sure what you've got. But #18 wire is fine at 10 amps, so just upgrade to a 10 amp breaker and ca ll it good. Remember, the CB is there to protect the wire not the load. mail.com wrote: What is the suggested CB rating of a navigation light circuit on a 14 VDC system? The airplane is fiberglass so the Nav light circuit is 53 feet from the CB to the switch to the Nav light fixture and back to the forest of grounds an d it is wired with 18 AWG wire?=C2- My current system has a 5 AMP CB that services only the two navigation light bulb fixtures that each has two bulb s (total of 4 incandescent light bulbs). I do not know the manufacturer of the light fixture so I do not have any idea what these light bulbs draw.=C2- It was suggested that I install a 5 AMP CB and when I turn on the switch the lights all illuminate and they stay on for about 20 minutes and then the CB pops.=C2- On hotter days the time be fore the CB pops is more like 10 minutes so it seems that the thermal load on th e current 5 AMP CB is too great.=C2- Of course I can upgrade the CB but I w ould prefer to know if a 7 AMP or 10 amp is preferred so that is why I am reaching out to the experts on the subject. As always THANKS for your help!!!. - S - WIKI - - =C2- =C2- =C2- =C2- =C2- -Matt Dralle, List Admin. ________________________________________________________________________________
From: <billhuntersemail(at)gmail.com>
Subject: Nav Light CB Rating
Date: Sep 11, 2020
Thanks Jeffgreatly appreciate the help!!! From: owner-aeroelectric-list-server(at)matronics.com On Behalf Of Jeff Luckey Sent: Friday, September 11, 2020 2:25 PM Subject: Re: AeroElectric-List: Nav Light CB Rating Bill, If you get the numbers from those bulbs you can look them up and find their current rating. That way you know for sure what you've got. But #18 wire is fine at 10 amps, so just upgrade to a 10 amp breaker and call it good. Remember, the CB is there to protect the wire not the load. -Jeff billhuntersemail(at)gmail.com > wrote: > What is the suggested CB rating of a navigation light circuit on a 14 VDC system? The airplane is fiberglass so the Nav light circuit is 53 feet from the CB to the switch to the Nav light fixture and back to the forest of grounds and it is wired with 18 AWG wire? My current system has a 5 AMP CB that services only the two navigation light bulb fixtures that each has two bulbs (total of 4 incandescent light bulbs). I do not know the manufacturer of the light fixture so I do not have any idea what these light bulbs draw. It was suggested that I install a 5 AMP CB and when I turn on the switch the lights all illuminate and they stay on for about 20 minutes and then the CB pops. On hotter days the time before the CB pops is more like 10 minutes so it seems that the thermal load on the current 5 AMP CB is too great. Of course I can upgrade the CB but I would prefer to know if a 7 AMP or 10 amp is preferred so that is why I am reaching out to the experts on the subject. As always THANKS for your help!!!. _/Navigator?AeroElectric-List" target="_blank">http://www.matronics.com/Na= - MATRONICS WEB FORUMS= --> <http://forums.matronics.com> /wiki.matronics.com <http://forums.matronics.com> " target="_blank">http://wiki.matronics.com http://www.mat=== ________________________________________________________________________________
From: mike Pienaar <mikepienaar09(at)gmail.com>
Subject: Re: Nav Light CB Rating
Date: Sep 11, 2020
I have Whelen position lights and current draw is 7.5 amp Sent from my iPhone > On Sep 11, 2020, at 2:38 PM, Jeff Luckey wrote: > > =EF=BB > > Bill, > > If you get the numbers from those bulbs you can look them up and find thei r current rating. That way you know for sure what you've got. > > But #18 wire is fine at 10 amps, so just upgrade to a 10 amp breaker and c all it good. > > Remember, the CB is there to protect the wire not the load. > > > -Jeff wrote: > > > > What is the suggested CB rating of a navigation light circuit on a 14 VDC > system? > > The airplane is fiberglass so the Nav light circuit is 53 feet from the CB > to the switch to the Nav light fixture and back to the forest of grounds a nd > it is wired with 18 AWG wire? My current system has a 5 AMP CB that > services only the two navigation light bulb fixtures that each has two bul bs > (total of 4 incandescent light bulbs). > > I do not know the manufacturer of the light fixture so I do not have any > idea what these light bulbs draw. It was suggested that I install a 5 AMP > CB and when I turn on the switch the lights all illuminate and they stay o n > for about 20 minutes and then the CB pops. On hotter days the time before > the CB pops is more like 10 minutes so it seems that the thermal load on t he > current 5 AMP CB is too great. Of course I can upgrade the CB but I would > prefer to know if a 7 AMP or 10 amp is preferred so that is why I am > reaching out to the experts on the subject. > > As always THANKS for your help!!!. > _/Navigator?AeroElectric-List" target="_blank">http://www.matronics.com/ Na= - MATRONICS WEB FORUMS= --> > /wiki.matronics.com" target="_blank">http://wiki.matronics.com > http://www.mat=== > > > ________________________________________________________________________________
Date: Sep 11, 2020
From: Jeff Luckey <jluckey(at)pacbell.net>
Subject: Re: How do you figure C-Rating of a battery?
Ron, The Amp Hour rating for a battery is kind of a theoretical number.=C2- In theory, a 5 Ah rating should deliver 5 amps for one hour but real batterie s rarely, if ever, do.=C2- It is more likely to deliver 1 A for 5 hours. =C2-=C2- Theoretically:=C2- Ah = Time (in hours) x Amps That is a linear equation, but as the current drain increases the length of time decreases in a non-linear fashion. Those 10C & 15C ratings are typical for batteries used in model aircraft & cars and are usually a little optimistic. The 10C rating is telling you that the battery supposedly has enough "grunt " to deliver 50 amps, but only for a very short period of time, like a minu te or two, if that.=C2- A lesser battery could not deliver 50 A at all, a nd if you tried to pull that kind of current, the battery voltage would dro p-off precipitously. I fly electric R/C airplanes using Li-po batteries (as opposed to the nicke l-metal batts you are referring to) and they can deliver 20C+.=C2- That m eans, for a 2 Ah battery, it can deliver up to 40 amps in short bursts, lik e when doing a full-throttle climb for 5-10 seconds.=C2- But the typical total flight time for RC airplanes is less than 10 minutes.=C2- We pull a lot of power for a short (no pun intended) duration. I'm not sure that answers you questions, but may give you some context. If we knew more about you application, we might be able to provide more use ful information. -Jeff dyates.com> wrote: Multiply 5000 by 10 for continuous, which is 50,000 milliamps, or 50 amps. On Fri, Sep 11, 2020 at 3:38 PM rparigoris wrote: > Hi Group I'm a bit confused how to figure C-rating of a battery. Here's my dilemma, here's a 5,000mA battery that say's it's good for 10C continuous a nd 15C burst: https://hobbyking.com/en_us/turnigy-sub-c-1-2v-5000mah-high-p ower-series-nimh-single-cell.html?queryID=3ed79812e155e159b8e96c63dba8378 a&objectID=41335&indexName=hbk_live_magento_en_us_products Initial thoughts is just multiply 5 times 10 or 15. Then I read a review an d the battery never made it to 5amp capacity and the capacity goes down the more amps you draw from it. Here's the review: https://lygte-info.dk/revie w/batteries2012/Turnigy%20Sub-C%205000mAh%20(Gray)%20UK.html If I wanted to know what the continuous C rating is for this battery, would you take the unrealistic too high 5aH capacity rating from the Mfg times 1 0? Take the ~3.5 20 amp discharge capacity rating from the review times 10? Or?? Now for another hard question, how would you figure out burst amp rating? T hx. Ron P. Read this topic online here: http://forums.matronics.com/viewtopic.php?p=498313#498313 - Electric-List" rel="noreferrer" target="_blank">http://www.matronics.co m/Navigator?AeroElectric-List FORUMS - eferrer" target="_blank">http://forums.matronics.com WIKI - errer" target="_blank">http://wiki.matronics.com b Site - =C2- =C2- =C2- =C2- =C2- -Matt Dralle, List Admin. rel="noreferrer" target="_blank">http://www.matronics.com/contribution ________________________________________________________________________________
Subject: Re: How do you figure C-Rating of a battery?
From: "rparigoris" <rparigor(at)hotmail.com>
Date: Sep 11, 2020
"If we knew more about you application, we might be able to provide more useful information." I have a Europa XS with both long and short wings powered by a Rotax 914. Although true if you have enough altitude you can put the prop in course pitch and air start, but with a cold engine it's hard on the engine and you need enough altitude and a good place to land if restart is unsuccessful to do it safety. Gettin a battery cold soaked can hurt performance for inflight restart or in the middle of nowhere ground start. I too was an electric model guy 15 years ago when the hottest sub C cell was a GP 2200mA. http://www.europaowners.org/main.php?g2_itemId=27335 I made up a pack that I could wire direct to fuel pump 2 to keep at least 1 fuel pump running if all else failed. So my initial mission was to just run a 2 amp fuel pump with a total loss battery. Since sub C NiMh batteries have been growing in capacity and ability to dump amps I was thinking I could use it for double duty as a jump pack for ships battery in a pinch. I will be using a Earth-X 680. My concern is when connecting NiMh pack to a 680 that is low on charge amperage draw between the 2 could be quite high. Ron P. Read this topic online here: http://forums.matronics.com/viewtopic.php?p=498320#498320 ________________________________________________________________________________
Date: Sep 12, 2020
From: Jeff Luckey <jluckey(at)pacbell.net>
Subject: Re: How do you figure C-Rating of a battery?
... so you are looking for some kind of power source to start your Rotax i n an emergency?=C2- Is that correct? The issue of batteries of different type charging each other would be a fun ction of the difference in voltage between them.=C2- I don't know anythin g about EarthX batts but since you would be building you own pack from indi vidual NiMh cells, you could build a pack that is close in voltage to the E arthX, maybe? There is also the possibility of isolating the EarthX batt from the NiMh ba ttery using diodes. I'm not sure but, I think it would take a bunch of NiMh batts to make a pac k with enough power to crank an engine. I believe that the EarthX are lithium based batteries (again, don't know mu ch about them) but that battery chemistry has very high energy density.=C2 - Much higher than NiMh, so if you're looking for some kind of backup bat tery, you're probably better off with the lithium-based batts and their hig her energy density and lower weight. -Jeff tmail.com> wrote: m> "If we knew more about you application, we might be able to provide more us eful information." I have a Europa XS with both long and short wings powere d by a Rotax 914. Although true if you have enough altitude you can put the prop in course pitch and air start, but with a cold engine it's hard on th e engine and you need enough altitude and a good place to land if restart i s unsuccessful to do it safety. Gettin a battery cold soaked can hurt perfo rmance for inflight restart or in the middle of nowhere ground start. I too was an electric model guy 15 years ago when the hottest sub C cell wa s a GP 2200mA. http://www.europaowners.org/main.php?g2_itemId=27335 I mad e up a pack that I could wire direct to fuel pump 2 to keep at least 1 fuel pump running if all else failed. So my initial mission was to just run a 2 amp fuel pump with a total loss battery. Since sub C NiMh batteries have b een growing in capacity and ability to dump amps I was thinking I could use it for double duty as a jump pack for ships battery in a pinch. I will be using a Earth-X 680. My concern is when connecting NiMh pack to a 680 that is low on charge amperage draw between the 2 could be quite high. Ron P. Read this topic online here: http://forums.matronics.com/viewtopic.php?p=498320#498320 - S - WIKI - - =C2- =C2- =C2- =C2- =C2- -Matt Dralle, List Admin. ________________________________________________________________________________
Subject: Management Assignment Help and Writing Service in Australia
From: "eillajones309" <eillajones309(at)gmail.com>
Date: Sep 11, 2020
Searching for Management Assignment Help and Writing Service in Australia? Students can now avail assignment help Australia (https://sourceessay.com/australia/) from SourceEssay essay writers who have ample experience in writing assignments on management. We have a team of highly qualified professional essay writers who write unique assignments for students adhering to the university guidelines. We make sure the assignments are proofread by subject matter experts to ensure they are unique and non-plagiarized before they are handed over to the students. Contact us to write high-quality management assignments today.[/list] Read this topic online here: http://forums.matronics.com/viewtopic.php?p=498323#498323 ________________________________________________________________________________
Date: Sep 12, 2020
From: "Robert L. Nuckolls, III" <nuckolls.bob(at)aeroelectric.com>
Subject: Re: How do you figure C-Rating of a battery?
At 02:22 PM 9/11/2020, you wrote: > >Hi Group I'm a bit confused how to figure C-rating of a battery. >Here's my dilemma, here's a 5,000mA battery that say's it's good for >10C continuous and 15C burst: >https://hobbyking.com/en_us/turnigy-sub-c-1-2v-5000mah-high-power-series-nimh-single-cell.html?queryID=3ed79812e155e159b8e96c63dba8378a&objectID=41335&indexName=hbk_live_magento_en_us_products >Initial thoughts is just multiply 5 times 10 or 15. Then I read a >review and the battery never made it to 5amp capacity and the >capacity goes down the more amps you draw from it. Here's the >review: >https://lygte-info.dk/review/batteries2012/Turnigy%20Sub-C%205000mAh%20(Gray)%20UK.html >If I wanted to know what the continuous C rating is for this >battery, would you take the unrealistic too high 5aH capacity rating >from the Mfg times 10? Take the ~3.5 20 amp discharge capacity >rating from the review times 10? Or?? >Now for another hard question, how would you figure out burst amp >rating? Thx. Ron P. One generally doesn't 'figure' any of that stuff. I'm pleasantly surprised when bench tests echo the nameplate ratings on any consumer production cell. Incorporation of any particular device into a project requires good DATA. Brands like A123, Samsung, Panasonic, Enersys, et. als. will publish data sheets with realistic performance metrics. Here are some sample engineering data sheets on products offered by reputable manufacturers. https://tinyurl.com/y425k3x6 Note that they offer detailed information, particularly discharge data that allows the system integrator to incorporate the device with confidence. No way in this universe are these cells capable of 6000 mAH . . . https://tinyurl.com/yyf64qe6 Another example of 'blue smoke' merchandising https://tinyurl.com/y2j9dkoh Note the 12v, 2A power input jack . . . 200 watts? Gimme a break. Now, I've used numerous examples of these amplifiers over the past 20 years . . . and they're quite capable of the 2-3 watts per channel called out in the spec sheets for the integrated circuits from which they are built. Attached are data plots on some 18650 cells marketed as '5500 mAH' devices with '10A pulse capability'. I was considering these cells for modifying a 20+ year old Makita drill from MiMh to Lithium. Tests on the drill demonstrated peak loads on the order of 10A. Based on some 'marketing' assertions I purchased some cells for consideration. The cells clearly tested at MUCH less capacity and sagged by 25% voltage under just a 5 Amps load. Clearly incapable of practical service at 10A. Note the difference in stored energy for having been charged to 3.7 versus 4.2 Volts. If you charged these cells in a charger optimized for LiFePO4, you would achieve about 1/2 the chemical capacity of the cell. Sorry Ron, the short answer is, "Lacking engineering performance data on the proposed device, you're searching around in a coal mine for a black cat with no flashlight." I'm not implying that ALL marketing hype is suspect . . . I've purchased many products that met their nameplate performance ratings quite well. But lacking hard data from the manufacturer or your own measurements, then it's a bit of a crap shoot. If you have exemplar cells you're considering for the task, mail them to me and I'll suck out their precious bodily fluids to MEASURE from what stuff they are made. By the way, the performance benchmark "Ampere-Hour" is useful only when comparing one device with a competing device. It has no definition in the science of energy measurement. The scientific description for energy will be in units like Watt-Hours, Joules, Ergs, etc. Definition of an Erg: "The energy required to lift a postage stamp to a height equal to its thickness" == Robert A. Heinlein, "Rocket Ship Galileo" 1947 = Bob . . . ________________________________________________________________________________
From: Christopher Cee Stone <rv8iator(at)gmail.com>
Date: Sep 12, 2020
Subject: Re: How do you figure C-Rating of a battery?
Regarding 18650 cells... etc. I purchased a few samples from different suppliers with ratings up to 4500 mAh. The ones rated at 4500 mah felt lighter in weight than the similar cells rated at 3000 mAh. A friend designs medical x-ray machines so I asked if he would x-ray one of the light weight cells and one "normal" cell. Unfortunately I don't have an image to share. A description will suffice. The light weight cell housed a much smaller cell. Hmmm! Simerly I was replacing the output transistor in a regulated bench power supply. I recall it was a 2N3055. I found a "deal" on Ebay. Five for $5... Upon receipt I noticed the marking on the case didn't appear quite right. It was labeled STM... lightly scratching the marking it readily came off. Hmmm. Using a Dremel tool with a cut-off wheel I removed the top of the case. Voila! A tiny junction with .002" bond wires! That someone can make a living going to the trouble of counterfeiting transistors! Caveat Emptor Chris Stone On Sat, Sep 12, 2020 at 9:36 AM Robert L. Nuckolls, III < nuckolls.bob(at)aeroelectric.com> wrote: > At 02:22 PM 9/11/2020, you wrote: > > rparigor(at)hotmail.com> > > Hi Group I'm a bit confused how to figure C-rating of a battery. Here's my > dilemma, here's a 5,000mA battery that say's it's good for 10C continuous > and 15C burst: > https://hobbyking.com/en_us/turnigy-sub-c-1-2v-5000mah-high-power-series-nimh-single-cell.html?queryID=3ed79812e155e159b8e96c63dba8378a&objectID=41335&indexName=hbk_live_magento_en_us_products > Initial thoughts is just multiply 5 times 10 or 15. Then I read a review > and the battery never made it to 5amp capacity and the capacity goes down > the more amps you draw from it. Here's the review: > https://lygte-info.dk/review/batteries2012/Turnigy%20Sub-C%205000mAh%20(Gray)%20UK.html > If I wanted to know what the continuous C rating is for this battery, > would you take the unrealistic too high 5aH capacity rating from the Mfg > times 10? Take the ~3.5 20 amp discharge capacity rating from the review > times 10? Or?? > Now for another hard question, how would you figure out burst amp rating? > Thx. Ron P. > > > One generally doesn't 'figure' any of that stuff. > > I'm pleasantly surprised when bench tests echo > the nameplate ratings on any consumer production > cell. Incorporation of any particular > device into a project requires good DATA. > Brands like A123, Samsung, Panasonic, Enersys, > et. als. will publish data sheets with realistic > performance metrics. > > Here are some sample engineering data sheets > on products offered by reputable manufacturers. > > https://tinyurl.com/y425k3x6 > > Note that they offer detailed information, particularly > discharge data that allows the system integrator > to incorporate the device with confidence. > > No way in this universe are these cells > capable of 6000 mAH . . . > > https://tinyurl.com/yyf64qe6 > > Another example of 'blue smoke' merchandising > > https://tinyurl.com/y2j9dkoh > > Note the 12v, 2A power input jack . . . 200 > watts? Gimme a break. Now, I've used numerous > examples of these amplifiers over the past 20 > years . . . and they're quite capable of the > 2-3 watts per channel called out in the spec > sheets for the integrated circuits from which > they are built. > > Attached are data plots on some 18650 cells > marketed as '5500 mAH' devices with '10A > pulse capability'. > > I was considering these cells for modifying > a 20+ year old Makita drill from MiMh to > Lithium. Tests on the drill demonstrated > peak loads on the order of 10A. > > Based on some 'marketing' assertions I purchased > some cells for consideration. The cells clearly > tested at MUCH less capacity and sagged by 25% > voltage under just a 5 Amps load. Clearly > incapable of practical service at 10A. > > Note the difference in stored energy for having > been charged to 3.7 versus 4.2 Volts. If you > charged these cells in a charger optimized > for LiFePO4, you would achieve about 1/2 the > chemical capacity of the cell. > > Sorry Ron, the short answer is, "Lacking > engineering performance data on the proposed > device, you're searching around in a coal > mine for a black cat with no flashlight." > > I'm not implying that ALL marketing hype is > suspect . . . I've purchased many products that met > their nameplate performance ratings quite > well. But lacking hard data from the manufacturer > or your own measurements, then it's a bit of > a crap shoot. > > If you have exemplar cells you're considering > for the task, mail them to me and I'll suck > out their precious bodily fluids to MEASURE > from what stuff they are made. > > By the way, the performance benchmark "Ampere-Hour" > is useful only when comparing one device with > a competing device. It has no definition in the > science of energy measurement. The scientific > description for energy will be in units > like Watt-Hours, Joules, Ergs, etc. > > Definition of an Erg: "The energy required to lift > a postage stamp to a height equal to its thickness" > == Robert A. Heinlein, "Rocket Ship Galileo" 1947 = > > Bob . . . > ________________________________________________________________________________
Subject: Re: Alternator/Strobe whine
From: Rick Beebe <rick(at)beebe.org>
Date: Sep 13, 2020
So it is alternator whine. It's present when the strobes are off. When they're on, it does warble a bit. There is a bit of strobe noise with both alternators off and just the battery on but it's not obnoxious at all. There's no noise at when running on the backup alternator. --Rick -------- Forwarded Message -------- Subject: Re: AeroElectric-List: Re: Alternator/Strobe whine Date: Tue, 8 Sep 2020 19:47:49 -0400 From: Rick Beebe <rick(at)beebe.org> I'll be back at the plane on Saturday and I will do some controlled tests. The noise pitch is definitely in sync with the strobes. When I turn the main alternator off the noise goes away completely. My memory is that it also goes away when I turn the strobes off (main alternator on), but I'm not sure if there's a residual whine. --Rick On 9/8/2020 4:41 PM, Robert L. Nuckolls, III wrote: > Alternator, strobe or both? > > Alternator whine is almost always a ground loop issue. > Pitch of whine will go up and down with engine rpm. > It often gets louder when alternator loads are highest > like right after starting engine (recharging battery) > and all loads turned on. > > Strobe whine pitches up and down in synchronization > with flashes from the strobes and is generally of > constant volume. > > Which one(s) disappear when appliance is OFF. > Alternator, strobe or both? > > > Bob . . . > ________________________________________________________________________________
Subject: Re: How do you figure C-Rating of a battery?
From: "rparigoris" <rparigor(at)hotmail.com>
Date: Sep 13, 2020
Hi Group Thank you for replies. Again I want a dual mission total loss battery, one to run a Rotax 914 fuel pump which I'm certain the 5,000mA cells are up to the task and two is to jump start main battery in a pinch. My dilemma is if I hook up NiMh pack to a flattened Earth-X battery the amp draw can be quite high. Not sure just how high and not sure how many amps the NiMh cells can dump. OK let's take a different tact for in a pinch jump starting. I posted a link to discharge table up to 20 amps for NiMh cells. I feel confident those cells can handle a discharge rate of 30 amps. What's thoughts of using this to limit amperage draw on NiMh cells when using them to jump?: http://vi.raptor.ebaydesc.com/ws/eBayISAPI.dll?ViewItemDescV4&item=184069714833&category=117000&pm=1&ds=0&t=1596298535000&ver=0 I have EZ pilot access on passenger headrest where I could make a series converter with red and black Anderson Power Pole connectors. Ron P. Read this topic online here: http://forums.matronics.com/viewtopic.php?p=498332#498332 ________________________________________________________________________________
Subject: Re: How do you figure C-Rating of a battery?
From: "rparigoris" <rparigor(at)hotmail.com>
Date: Sep 13, 2020
Hi Group Thank you for replies. Again I want a dual mission total loss battery, one to run a Rotax 914 fuel pump which I'm certain the 5,000mA cells are up to the task and two is to jump start main battery in a pinch. My dilemma is if I hook up NiMh pack to a flattened Earth-X battery the amp draw can be quite high. Not sure just how high and not sure how many amps the NiMh cells can dump. OK let's take a different tact for in a pinch jump starting. I posted a link to discharge table up to 20 amps for NiMh cells. I feel confident those cells can handle a discharge rate of 30 amps. What's thoughts of using this to limit amperage draw on NiMh cells when using them to jump?: http://vi.raptor.ebaydesc.com/ws/eBayISAPI.dll?ViewItemDescV4&item=184069714833&category=117000&pm=1&ds=0&t=1596298535000&ver=0 I have EZ pilot access on passenger headrest where I could make a series converter with red and black Anderson Power Pole connectors. Ron P. Read this topic online here: http://forums.matronics.com/viewtopic.php?p=498333#498333 ________________________________________________________________________________
Subject: Re: How do you figure C-Rating of a battery?
From: Charlie England <ceengland7(at)gmail.com>
Date: Sep 13, 2020
On 9/13/2020 9:53 AM, rparigoris wrote: > > Hi Group Thank you for replies. Again I want a dual mission total loss battery, one to run a Rotax 914 fuel pump which I'm certain the 5,000mA cells are up to the task and two is to jump start main battery in a pinch. My dilemma is if I hook up NiMh pack to a flattened Earth-X battery the amp draw can be quite high. Not sure just how high and not sure how many amps the NiMh cells can dump. OK let's take a different tact for in a pinch jump starting. I posted a link to discharge table up to 20 amps for NiMh cells. I feel confident those cells can handle a discharge rate of 30 amps. What's thoughts of using this to limit amperage draw on NiMh cells when using them to jump?: http://vi.raptor.ebaydesc.com/ws/eBayISAPI.dll?ViewItemDescV4&item=184069714833&category=117000&pm=1&ds=0&t=1596298535000&ver=0 > I have EZ pilot access on passenger headrest where I could make a series converter with red and black Anderson Power Pole connectors. Ron P. > > I think we still don't have enough detail on your hypothetical 'situation' needs. On the one hand, you're talking about powering just a fuel pump (implying a total electrical failure), and the other about either jump-starting the engine, or recharging a dead battery, but we don't know if that means a failed charging system or just a run-down battery. We do know that NiMh cells have much lower energy density than any of the lithium tech. Transferring energy to a lithium tech battery from NiMh cells just doesn't sound like a reasonable path to me, for a variety of reasons. I'm pretty 'cheap'; I've been known to expend prodigious effort to save a few $. But from what I think I understand about your goals, I'd find it hard to resist just picking up one of the dozens of lithium 'jump start' packs. They have high current cables for jump starting, and almost all of them now include smaller terminals, adapters, etc that allow connection of devices with lower current demands. $55-$75 will get you pre-packaged cells, cables, a handful of adapters, etc, very close to 'plug&play'. I'll bet that by the time you gather up all the stuff you need to make your idea work, you'll have close to that much money tied up, and a *lot* more work. Way more starting current than you'd ever need, and even the cheapest versions will have more total energy to run the fuel pump than the NiMh cells. Charlie -- This email has been checked for viruses by Avast antivirus software. https://www.avast.com/antivirus ________________________________________________________________________________
Date: Sep 13, 2020
From: "Robert L. Nuckolls, III" <nuckolls.bob(at)aeroelectric.com>
Subject: Re: How do you figure C-Rating of a battery?
At 09:53 AM 9/13/2020, you wrote: > >Hi Group Thank you for replies. Again I want a dual mission total >loss battery, one to run a Rotax 914 fuel pump which I'm certain the >5,000mA cells are up to the task and two is to jump start main >battery in a pinch. Is there a history for the failure modes you are hypothesizing? Under what series of events would you find it necessary to operated a fuel pump from it's own back up battery? Jump start? Lithium main battery? Going airborne with an essentially dead main battery after jump starting the engine is poor practice. Not recommended in any advice I've read or written. How hard is it to (1) prevent the main battery from becoming depleted while parked and (2) maintaining the main battery in a manner that guarantees availability for alternator-out situations? I think we're all having trouble understanding justification for what you propose. Bob . . . ________________________________________________________________________________
Date: Sep 13, 2020
From: "Robert L. Nuckolls, III" <nuckolls.bob(at)aeroelectric.com>
Subject: Re: How do you figure C-Rating of a battery?
>I'm pretty 'cheap'; I've been known to expend prodigious effort to >save a few $. But from what I think I understand about your goals, >I'd find it hard to resist just picking up one of the dozens of >lithium 'jump start' packs. They have high current cables for jump >starting, and almost all of them now include smaller terminals, >adapters, etc that allow connection of devices with lower current >demands. $55-$75 will get you pre-packaged cells, cables, a handful >of adapters, etc, very close to 'plug&play'. I'll bet that by the >time you gather up all the stuff you need to make your idea work, >you'll have close to that much money tied up, and a *lot* more work. >Way more starting current than you'd ever need, and even the >cheapest versions will have more total energy to run the fuel pump >than the NiMh cells. > >Charlie Sure. If he drops the 'jump start' requirement but has an FMEA driven justification for a stand-by battery on the fuel pump, these batteries connected in series would run the pump for a very long time . . . probably duration of fuel aboard. https://tinyurl.com/y4oulfy9 Down side is the legacy stand-by-battery-cost- of-ownership for insuring continued air worthiness. I'd like to understand what conditions would drive the need in the first place. Bob . . . ________________________________________________________________________________
Subject: Re: How do you figure C-Rating of a battery?
From: Charlie England <ceengland7(at)gmail.com>
Date: Sep 13, 2020
On 9/13/2020 11:06 AM, Robert L. Nuckolls, III wrote: >> I'm pretty 'cheap'; I've been known to expend prodigious effort to >> save a few $. But from what I think I understand about your goals, >> I'd find it hard to resist just picking up one of the dozens of >> lithium 'jump start' packs. They have high current cables for jump >> starting, and almost all of them now include smaller terminals, >> adapters, etc that allow connection of devices with lower current >> demands. $55-$75 will get you pre-packaged cells, cables, a handful >> of adapters, etc, very close to 'plug&play'. I'll bet that by the >> time you gather up all the stuff you need to make your idea work, >> you'll have close to that much money tied up, and a *lot* more work. >> Way more starting current than you'd ever need, and even the cheapest >> versions will have more total energy to run the fuel pump than the >> NiMh cells. >> >> Charlie > > > Sure. If he drops the 'jump start' requirement > but has an FMEA driven justification for a > stand-by battery on the fuel pump, these batteries > connected in series would run the pump for a very > long time . . . probably duration of fuel aboard. > > https://tinyurl.com/y4oulfy9 > > Down side is the legacy > stand-by-battery-cost- > of-ownership for insuring continued air worthiness. > I'd like to understand what conditions would drive > the need in the first place. > > > Bob . . . > As I said in the earlier section of that post, we don't know his actual needs. But if 'jump start' or 'recharge' involves being stuck out in the boonies with no access to service or a charger (which could easily happen at a paved, government-run airport in some parts of the country), then I can see having a 'jump start' pack in my emergency kit. There may be no reason to have a dead battery, but unfortunately, *we* have to be figured into the equation. I managed to kill a fairly expensive 'certified' SLA battery by leaving the 'mag switch' controlling the electronic ignition turned on for about a week. No excuses; my stupidity, but that didn't keep the battery from being dead. Fortunately the plane was in my hangar at home. Charlie -- This email has been checked for viruses by Avast antivirus software. https://www.avast.com/antivirus ________________________________________________________________________________
Subject: Re: How do you figure C-Rating of a battery?
From: "Martymason" <captainmarty(at)bellsouth.net>
Date: Sep 13, 2020
[quote="nuckolls.bob(at)aeroelect"]At 09:53 AM 9/13/2020, you wrote: Is there a history for the failure modes you are hypothesizing? The Europa with long wings is a motorglider. Hence the need to restart in air. I think Ron is looking for belt & suspenders. Marty Mason -------- Marty Mason Norcross, GA Read this topic online here: http://forums.matronics.com/viewtopic.php?p=498340#498340 ________________________________________________________________________________
Subject: Re: Fwd: Re: Alternator/Strobe whine
From: "user9253" <fransew(at)gmail.com>
Date: Sep 13, 2020
Check the AC component of the alternator output. It should be less than 1 volt AC. More than that means a diode is bad. -------- Joe Gores Read this topic online here: http://forums.matronics.com/viewtopic.php?p=498341#498341 ________________________________________________________________________________
Subject: Re: How do you figure C-Rating of a battery?
From: "rparigoris" <rparigor(at)hotmail.com>
Date: Sep 13, 2020
Hi Bob Here's a few pics of under 2 lb battery: http://www.europaowners.org/main.php?g2_itemId=27335 and http://www.europaowners.org/main.php?g2_itemId=30614 the batteries are getting old and I want to replace them with 10 year newer NiMh technology. You sent a link for a LiPo battery, true they have great energy density but LiPo is also one active Lithium technology, no place in an aeroplane IMHO. My reasons for wanting a independent battery for a fuel pump is in case main battery stops working for whatever reason. Side note, Europa has a main tank and a reserve just like a motorcycle. Run out of fuel and turn to reserve and you know how long it will last. My reason for total loss battery to run a pump is pretty much the same. If you can no longer supply power to a pump, I'll call on my "reserve battery". Potential things that would cause a failure of E-Bus is a battery that fails opened. Yes yes yes I have read read read read that if you capacity test a battery once a year and replace often that the chance of a failure to occur after the motor is started is very slim. I have had at least 2 failures on a car when driving and battery opened and engine quit. Once on a motorcycle where things went dark but engine still ran. Now I'm introducing a LiFe Earth-X battery that is not a lead acid and as far as I'm concerned unquestionably has the potential to offer up no potential for other reasons besides internal connections going very high resistance. I have a SD20S with LR3C on vacuum pad of Rotax 914, I haven't any experience with SD20S and LR3C, but I did have a Suzuki Samari with a ND alternator, it had a solid state internal regulator, anyway the regulator failed, cooked the battery and I got stuck on the side of the road. True LR3C has over-voltage protection but could it fail and cook my battery? I'm using Eric Jones aluminium clad wire (battery in back), I crimped and soldered and paid a lot of attension to smooth bends and support, but there are several connections that could fail and go high resista! nce and make my E-bus inoperative. If I take a long and extended flight with additonal fuel onboard, if charging system failed and I was flying off of E-Bus, then engine quit because battery went flat either for flying too long or let's say the fool Rotax TCU decided to draw a lot more power than it normally does and engine quits. I just connect total loss "reserve battery". I haven't tested it yet but if I were on total loss battery, I think I could cycle it on and off and probably double range. Anyway that's my reasons for being able to run a fuel pump. Just recently I figured since I have this battery already, I could use it as a in a pinch jump start pack. This would be just a value added. I agree with you taking off with a battery that's flat is a bad idea. Let's say I'm in the middle of nowhere and camping out. Some load was left on and battery is a bit flat. If I could let the total loss battery charge the main battery for a bit then try a start, if it started I would be gentle and use only the SD20S to charge on ground for a while at reasonable rate before taking off. If I have a choice I don't fast charge. Another scenario is after shutting down motor and soaring for a while, for some reason the engine will not start. If you shut down motor you always want a landing spot at hand assuming that the motor won't start. Let's say for whatever reason the battery can't offer up enough electron flow to do an inflight restart, could be too much a load was left on, bad cell in battery or plain bad luck, true if had enough altitude could go course pitch and do an airstart, kinda hard on a cold engine and you need a reasonable amount of altitude, anyway if I was able to do an inflight jump start, just an added bonus. The under 2 pounds lets me sleep a little better knowing I could keep the motor running if all else fails. Unquestionably belt and suspenders, hope I never need to use it. Ron P. Read this topic online here: http://forums.matronics.com/viewtopic.php?p=498342#498342 ________________________________________________________________________________
Date: Sep 13, 2020
From: "Robert L. Nuckolls, III" <nuckolls.bob(at)aeroelectric.com>
Subject: Re: Alternator/Strobe whine
At 11:03 PM 9/12/2020, you wrote: >So it is alternator whine. It's present when the strobes are off. >When they're on, it does warble a bit. There is a bit of strobe >noise with both alternators off and just the battery on but it's not >obnoxious at all. There's no noise at when running on the backup alternator. > >--Rick This about has to be a grounding issue. Refresh my memory. What kind of airplane? Can you describe your ground system? Conforms to recommendations in Z15/Chaper 5 in the 'Connection? Bob . . . ________________________________________________________________________________
Subject: Re: How do you figure C-Rating of a battery?
From: "user9253" <fransew(at)gmail.com>
Date: Sep 13, 2020
Can a dead battery be recharged by a fully charged battery? -------- Joe Gores Read this topic online here: http://forums.matronics.com/viewtopic.php?p=498346#498346 ________________________________________________________________________________
Date: Sep 13, 2020
From: "Robert L. Nuckolls, III" <nuckolls.bob(at)aeroelectric.com>
Subject: Re: How do you figure C-Rating of a battery?
You've stacked a bucket load of suppositions and hypotheticals for which I perceive no foundation. Most of the things you're worrying about are the same things we've been working to mitigate for the last 100 years or so. Worried about crimped AND soldered connections? Why? No, LR3s have no failure modes that would cook a battery. LiFEPO4 has an excellent design history and track record when packaged with a BMS. AeroVolz, EarthX, TruBlue and others have done their homework. AeroVolts has a BMS fitted battery right at 2 pounds and proofs out at 4.5AH . . . it will even crank an engine. What kind of charge/maintenance protocol are you proposing for your home brew array? > Unquestionably belt and suspenders, hope I never need to use it. Ron P. Ron, of all the electrically involved accidents I've worked, the vast majority had root cause in inattention to mechanical or architectural design details. I.e. poor FMEA and/or craftsmanship. When component failure was part of the script, it was almost never due to manufacturing defect . . . in fact I cannot now recall a single instance. Consider sharing your architecture drawing with the List . . . let's run it under the FMEA microscope before you pile on Band-Aids more likely to produce complexity induced risk. Better to prevent the injury than to carry tourniquets and field dressings. Bob . . . ________________________________________________________________________________
Date: Sep 14, 2020
From: "Robert L. Nuckolls, III" <nuckolls.bob(at)aeroelectric.com>
Subject: Re: How do you figure C-Rating of a battery?
At 09:38 PM 9/13/2020, you wrote: > >Can a dead battery be recharged by a fully charged battery? Think about it. What voltage does the alternator put out to charge the battery? What voltage does the battery put out while being discharged? Exception: Those glove box 'chargers' for dead battery recovery are much higher than 14v. Further, they obviously don't have the energy to fully charge their rescue battery . . . just put in enough snort to crank the engine. Bob . . . ________________________________________________________________________________
From: William Daniell <wdaniell.longport(at)gmail.com>
Date: Sep 14, 2020
Subject: Sealed lead Acid battery
Is there any reason why I shouldn't use a SLA battery like this? Ill fess up. I always used these sort of batteries - for the last 15 years to no ill effect but that doesn't not mean that this is a recommended practice. They normally last about 5 years. thanks will PS I am not electrically dependant and have two back up batteries for the avionics [image: image.png] William Daniell LONGPORT +1 786 878 0246 ________________________________________________________________________________
From: Charlie England <ceengland7(at)gmail.com>
Date: Sep 14, 2020
Subject: Re: Sealed lead Acid battery
On Mon, Sep 14, 2020 at 7:03 AM William Daniell wrote: > Is there any reason why I shouldn't use a SLA battery like this? > Ill fess up. I always used these sort of batteries - for the last 15 > years to no ill effect but that doesn't not mean that this is a recommended > practice. They normally last about 5 years. > thanks > will > PS I am not electrically dependant and have two back up batteries for the > avionics > > William Daniell > LONGPORT > +1 786 878 0246 > Hi William, I'll see your 15 & raise you at least 5. If we shouldn't be using them, please don't tell. :-) I've been using similar products, with much less well known brand names, with no issues outside what could happen to any product. (ex: shipping damage) I used the 18 AH size for years on O320 engines, until I discovered 20 AH & 22 AH versions that are almost the same size. If you have a bigger engine, and your battery mount isn't a 'glove fit' arrangement, you can go to the larger batteries for a bit more money but still about 1/2 what an Odyssey costs. And there have been more and more reports over the last couple of years about very short lived Odysseys; unlike the 'no name' stuff I've been using. Also related to the 'big engine' starting current issue: Some (unfortunately, not all) of these batteries will spec their source (or internal) impedance. The one you image, like the ones I've used in the past, has relatively low mass terminals on top. While similar batteries have plenty of 'grunt' to start all but the really big, high compression engines, they are typically designed for use in mobility carts, UPSs, etc that have lower max current than a starter motor. Their internal impedance is typically a bit higher than a purpose-built SLA starting battery with the same AH rating on the label. Most obvious clue, absent detailed specs, will be the somewhat heavier terminals on the starting battery. Charlie ________________________________________________________________________________
From: Carlos Trigo <trigo(at)mail.telepac.pt>
Subject: Re: Sealed lead Acid battery
Date: Sep 14, 2020
William I have to disagree with your PS. You are indeed electrically dependant, otherwise your heart wouldn=99t work, and you will have to use a pacemaker.... Sorry, couldn=99t resist Regards Carlos Enviado do meu iPhone > No dia 14/09/2020, =C3-s 14:28, Charlie England e screveu: > > =EF=BB > > >> On Mon, Sep 14, 2020 at 7:03 AM William Daniell <wdaniell.longport@gmail. com> wrote: >> Is there any reason why I shouldn't use a SLA battery like this? >> Ill fess up. I always used these sort of batteries - for the last 15 yea rs to no ill effect but that doesn't not mean that this is a recommended pra ctice. They normally last about 5 years. >> thanks >> will >> PS I am not electrically dependant and have two back up batteries for the avionics >> >> William Daniell >> LONGPORT >> +1 786 878 0246 ________________________________________________________________________________
Subject: Re: Alternator/Strobe whine
From: Rick Beebe <rick(at)beebe.org>
Date: Sep 14, 2020
On 9/13/2020 12:10 PM, Robert L. Nuckolls, III wrote: > At 11:03 PM 9/12/2020, you wrote: > >> So it is alternator whine. It's present when the strobes are off. >> When they're on, it does warble a bit. There is a bit of strobe noise >> with both alternators off and just the battery on but it's not >> obnoxious at all. There's no noise at when running on the backup >> alternator. >> >> --Rick > > This about has to be a grounding issue. > Refresh my memory. What kind of airplane? > Can you describe your ground system? Conforms > to recommendations in Z15/Chaper 5 in the > 'Connection? It's a GlaStar. Battery is in the back. The negative cable runs to a terminal strip mounted on the sidewall on the copilot side. Most non-avionics things such as lights, boost pumps, etc, are grounded there. When I re-did the panel I put an avionics ground block on the back of the panel. It's connected via redundant wires to the ground terminal strip. The other end of the terminal strip connects to another thick wire the firewall which connects to the engine. I believe the starter, engine and alternator are gounded there but I have to verify. That alternator is one of the few things I did not touch in the panel re-do. I'll be opening things up for the condition inspection in a couple weeks and I'll take a very close look at how that alternator is wired. --Rick ________________________________________________________________________________
Subject: Re: How do you figure C-Rating of a battery?
From: "rparigoris" <rparigor(at)hotmail.com>
Date: Sep 14, 2020
Hi Bob Thank you for your review. I will post schematic when it's completed. A few points: I'm not too worried about LiFe batteries getting thermal runaway, but I am about LiPo batteries getting to a thermal runaway. You posted a link to: https://tinyurl.com/y4oulfy9 As far as LiFe batteries go, I think mfg has done a reasonable amount of homework but fact is there is potential for problems that Lead Acid does not have and they can still fail opened. Good enough for me to use. I'm not worried about soldered and crimped connection to aluminium wires, but the terminal connections can get a high resistance. Keeping connections clean and tight mitigate majority of problems, but not all. As far as keeping NiMh pack charged, using one of my AstroFlight delta peak chargers (110D or 112D), once or twice a year will give them a C/10 for ~ 15 hours to balance, then delta peak charge the rest of the times. Unless battery is very flat, peaking at 1C takes less time than a pre-flight. 1x a year will capacity test down to 1V per cell and fuel pump equivalent load. BTW have you ever used an AstroFlight Whatt Meter (Whatt is correctly spelled)? Pretty slick meter that shows volts, amps and watt hours. Pricey though, here's an alternative that works pretty good: https://hobbyking.com/en_us/turnigy-180a-watt-meter-and-power-analyzer.html?queryID=7b349d17e0c87dff11877b2ea51342de&objectID=42854&indexName=hbk_live_magento_en_us_products Ron P. Read this topic online here: http://forums.matronics.com/viewtopic.php?p=498356#498356 ________________________________________________________________________________
From: William Daniell <wdaniell.longport(at)gmail.com>
Date: Sep 14, 2020
Subject: Re: Sealed lead Acid battery
nice one! but I do have an independent power source William Daniell LONGPORT +1 786 878 0246 On Mon, Sep 14, 2020 at 10:39 AM Carlos Trigo wrote : > William > > I have to disagree with your PS. > You are indeed electrically dependant, otherwise your heart wouldn =99t work, > and you will have to use a pacemaker.... > > Sorry, couldn=99t resist > Regards > Carlos > > > Enviado do meu iPhone > > No dia 14/09/2020, =C3-s 14:28, Charlie England > escreveu: > > =EF=BB > > > On Mon, Sep 14, 2020 at 7:03 AM William Daniell < > wdaniell.longport(at)gmail.com> wrote: > >> Is there any reason why I shouldn't use a SLA battery like this? >> Ill fess up. I always used these sort of batteries - for the last 15 >> years to no ill effect but that doesn't not mean that this is a recommen ded >> practice. They normally last about 5 years. >> thanks >> will >> PS I am not electrically dependant and have two back up batteries for th e >> avionics >> >> William Daniell >> LONGPORT >> +1 786 878 0246 >> > ________________________________________________________________________________
From: William Daniell <wdaniell.longport(at)gmail.com>
Date: Sep 14, 2020
Subject: Re: Sealed lead Acid battery
Charlie thanks, I too have been using cheap chinese SLA batteries which is the only small ish battery I could obtain in Colombia. It is also the normally used rotax battery there except where the battery is in the engine compartment because the belief is that SLA dont handle heat well. Starts my 912 turbo fine. But the stupidest question is the one you don't ask. And since I'm now in the US I have a few more alternatives... Will William Daniell LONGPORT +1 786 878 0246 On Mon, Sep 14, 2020 at 9:25 AM Charlie England wrote: > > > On Mon, Sep 14, 2020 at 7:03 AM William Daniell < > wdaniell.longport(at)gmail.com> wrote: > >> Is there any reason why I shouldn't use a SLA battery like this? >> Ill fess up. I always used these sort of batteries - for the last 15 >> years to no ill effect but that doesn't not mean that this is a recommended >> practice. They normally last about 5 years. >> thanks >> will >> PS I am not electrically dependant and have two back up batteries for the >> avionics >> >> William Daniell >> LONGPORT >> +1 786 878 0246 >> > Hi William, > > I'll see your 15 & raise you at least 5. If we shouldn't be using them, > please don't tell. :-) > > I've been using similar products, with much less well known brand names, > with no issues outside what could happen to any product. (ex: shipping > damage) > I used the 18 AH size for years on O320 engines, until I discovered 20 AH > & 22 AH versions that are almost the same size. If you have a bigger > engine, and your battery mount isn't a 'glove fit' arrangement, you can go > to the larger batteries for a bit more money but still about 1/2 what an > Odyssey costs. And there have been more and more reports over the last > couple of years about very short lived Odysseys; unlike the 'no name' stuff > I've been using. > > Also related to the 'big engine' starting current issue: Some > (unfortunately, not all) of these batteries will spec their source (or > internal) impedance. The one you image, like the ones I've used in the > past, has relatively low mass terminals on top. While similar batteries > have plenty of 'grunt' to start all but the really big, high compression > engines, they are typically designed for use in mobility carts, UPSs, etc > that have lower max current than a starter motor. Their internal impedance > is typically a bit higher than a purpose-built SLA starting battery with > the same AH rating on the label. Most obvious clue, absent detailed specs, > will be the somewhat heavier terminals on the starting battery. > > Charlie > > ________________________________________________________________________________
Subject: Re: Sealed lead Acid battery
From: Charlie England <ceengland7(at)gmail.com>
Date: Sep 14, 2020
Oh... Please don't tell the batteries about that. They lived on the hot side of the firewall of my RV4 for about 15 years. On 9/14/2020 1:51 PM, William Daniell wrote: > Charlie thanks, > I too have been using cheap chinese SLA batteries which is the only > small ish battery I could obtain in Colombia. It is also the normally > used rotax battery there exceptwhere the battery is in the engine > compartment because the belief is that SLA dont handle heat well. > Starts my 912 turbo fine. But the stupidestquestion is the one you > don't ask. And since I'm now in the US I have a few more alternatives... > Will > William Daniell > LONGPORT > +1 786 878 0246 > > > On Mon, Sep 14, 2020 at 9:25 AM Charlie England > wrote: > > > On Mon, Sep 14, 2020 at 7:03 AM William Daniell > > > wrote: > > Is there any reason why I shouldn'tuse a SLA battery like this? > Ill fess up. I always used these sort of batteries - for the > last 15 years to no ill effect but that doesn't not mean that > this is a recommended practice. They normally last about 5 years. > thanks > will > PS I am not electricallydependant and have two back up > batteries for the avionics > > William Daniell > LONGPORT > +1 786 878 0246 > > Hi William, > > I'll see your 15 & raise you at least 5. If we shouldn't be using > them, please don't tell. :-) > > I've been using similar products, with much less well known brand > names, with no issues outside what could happen to any product. > (ex: shipping damage) > I used the 18 AH size for years onO320 engines, until I > discovered 20 AH & 22 AH versions that are almost the same size. > If you have a bigger engine, and your battery mount isn't a 'glove > fit' arrangement, you can go to the larger batteries for a bit > more money but still about 1/2 what an Odyssey costs. And there > have been more and more reports over the last couple of years > about very short lived Odysseys; unlike the 'no name' stuff I've > been using. > > Also related to the 'big engine' starting current issue: Some > (unfortunately, not all) of these batteries will spec their source > (or internal) impedance. The one you image, like the ones I've > used in the past, has relatively low mass terminals on top. While > similar batteries have plenty of 'grunt' to start all but the > really big, high compression engines, they are typically designed > for use in mobility carts, UPSs, etc that have lower max current > than a starter motor. Their internal impedance is typically a bit > higher than a purpose-built SLA starting battery with the same AH > rating on the label. Most obvious clue, absent detailed specs, > will be the somewhat heavier terminals on the starting battery. > > Charlie > -- This email has been checked for viruses by Avast antivirus software. https://www.avast.com/antivirus ________________________________________________________________________________
From: William Daniell <wdaniell.longport(at)gmail.com>
Date: Sep 14, 2020
Subject: Re: Sealed lead Acid battery
Dang...i was misinformed. William Daniell +1 786 878 0246 On Mon, Sep 14, 2020, 15:27 Charlie England wrote: > Oh... Please don't tell the batteries about that. They lived on the hot > side of the firewall of my RV4 for about 15 years. > > On 9/14/2020 1:51 PM, William Daniell wrote: > > Charlie thanks, > I too have been using cheap chinese SLA batteries which is the only small > ish battery I could obtain in Colombia. It is also the normally used rotax > battery there except where the battery is in the engine compartment because > the belief is that SLA dont handle heat well. Starts my 912 turbo fine. > But the stupidest question is the one you don't ask. And since I'm now in > the US I have a few more alternatives... > Will > William Daniell > LONGPORT > +1 786 878 0246 > > > On Mon, Sep 14, 2020 at 9:25 AM Charlie England > wrote: > >> >> >> On Mon, Sep 14, 2020 at 7:03 AM William Daniell < >> wdaniell.longport(at)gmail.com> wrote: >> >>> Is there any reason why I shouldn't use a SLA battery like this? >>> Ill fess up. I always used these sort of batteries - for the last 15 >>> years to no ill effect but that doesn't not mean that this is a recommended >>> practice. They normally last about 5 years. >>> thanks >>> will >>> PS I am not electrically dependant and have two back up batteries for >>> the avionics >>> >>> William Daniell >>> LONGPORT >>> +1 786 878 0246 >>> >> Hi William, >> >> I'll see your 15 & raise you at least 5. If we shouldn't be using them, >> please don't tell. :-) >> >> I've been using similar products, with much less well known brand names, >> with no issues outside what could happen to any product. (ex: shipping >> damage) >> I used the 18 AH size for years on O320 engines, until I discovered 20 AH >> & 22 AH versions that are almost the same size. If you have a bigger >> engine, and your battery mount isn't a 'glove fit' arrangement, you can go >> to the larger batteries for a bit more money but still about 1/2 what an >> Odyssey costs. And there have been more and more reports over the last >> couple of years about very short lived Odysseys; unlike the 'no name' stuff >> I've been using. >> >> Also related to the 'big engine' starting current issue: Some >> (unfortunately, not all) of these batteries will spec their source (or >> internal) impedance. The one you image, like the ones I've used in the >> past, has relatively low mass terminals on top. While similar batteries >> have plenty of 'grunt' to start all but the really big, high compression >> engines, they are typically designed for use in mobility carts, UPSs, etc >> that have lower max current than a starter motor. Their internal impedance >> is typically a bit higher than a purpose-built SLA starting battery with >> the same AH rating on the label. Most obvious clue, absent detailed specs, >> will be the somewhat heavier terminals on the starting battery. >> >> Charlie >> >> > > > Virus-free. > www.avast.com > > <#m_2636863385803712607_DAB4FAD8-2DD7-40BB-A1B8-4E2AA1F9FDF2> > ________________________________________________________________________________
From: Ron Springer <ron228rj(at)gmail.com>
Date: Sep 14, 2020
Subject: Re: B&C SD-8 Alternator/Regulator
To close this one out ... The source of the noise was the magneto itself. Disconnect the p-lead to the engine monitor, and all of the data was stable. However, the magneto was still radiating RF that was causing other problems. It would stop data recording to the USB stick on my EFIS. It created some radio noise. It possibly took my GPS antenna offline (or it might have just been the location, which I also changed). So, why was the brand new magneto creating noise ... because when I bought the plane it was set up to use auto spark plugs, specifically Autolite 386. Other than some radio noise, the previous owner didn't have any issues because he had few electronics in the plane, mostly steam gauges. My theory is that the magneto reacts poorly to high resistance downstream of it. You are supposed to throw out an aircraft plug when it hits 5000 ohms, but most Autolite 386 plugs are around 10000 ohms out of the box. The manufacturer provides a large range from 3k to 12k ohms. I tested six new ones and measured between 7k and 12k ohms. There is probably an auto plug solution that would work fine on my magento, but I am done experimenting for now. I flew with the aircraft plugs and it fixed all my problems at once. Ron On Wed, Aug 26, 2020 at 8:19 PM Robert L. Nuckolls, III < nuckolls.bob(at)aeroelectric.com> wrote: > At 06:28 PM 8/26/2020, you wrote: > > Just did a test flight and shot some video of the EIS to compare to the > EFIS. I haven=C3=A2=82=AC=84=A2t even looked at it yet because I di d something else for > the first time. I shut off the magneto and everything got perfectly quiet > on the EFIS, along with the noise that seems to be getting louder in my > headset on each flight! > > > Aha! One more flight test: > > Leave the mag wire totally unhooked at > the engine end. Of course this leaves > you an always-hot mag but the p-lead > effects are now out of the diagnostic > loop. > > If the effect goes away, you're looking > at p-lead; if it stays you're looking > at plug wires. > > > Bob . . . > ________________________________________________________________________________
From: mike Pienaar <mikepienaar09(at)gmail.com>
Date: Sep 14, 2020
Subject: Regulator failure
A friend has an aircraft with a wind/propellor driven generator controlled by a "Ford type" regulator. The aircraft made its first flight in 6 years recently and the regulator was completely fried. Everything inside was heat damaged and the insulation on the wires entering the regulator melted off. Where do we start looking? Thanks Mike ________________________________________________________________________________
Date: Sep 15, 2020
From: Jeff Luckey <jluckey(at)pacbell.net>
Subject: Re: Regulator failure
Mike, 1. Do you know that the wiring was good before the flight?2. Part numbers / nameplate data for generator & regulator?3. How many amps can this generator generate?4. It's a generator and not an alternator?5. Any pictures? -Jeff wrote: A friend has an aircraft with a wind/propellor driven generator controlled by a "Ford type" regulator. The aircraft made its first flight in 6 years recently and the regulator was completely fried. Everything inside was heat damaged and the insulation on the wires entering the regulator melted off. Where do we start looking? Thanks Mike ________________________________________________________________________________
Subject: Re: B&C SD-8 Alternator/Regulator
From: Charlie England <ceengland7(at)gmail.com>
Date: Sep 14, 2020
Yeah...That would have been a useful data point to supply for troubleshooting. ;-) Some have gotten away with using specific plug wires & automotive plugs with mags, but the whole reason mags have shielded wire and shielded plugs is to suppress the intense RF ignition noise from the mag. Here's one way to do it successfully: http://www.g3ignition.com/magmod.html Note that the 386s are being used by some of us with electronic ignition, but internet lore says they have a higher heat range than typical a/c plugs, or the metric plugs that require an adapter. Charlie On 9/14/2020 6:54 PM, Ron Springer wrote: > To close this one out ... > > The source of the noise was the magneto itself. Disconnect the p-lead > to the engine monitor, and all of the data was stable. However, the > magneto was still radiating RF that was causing other problems. It > would stop data recording to the USB stick on my EFIS. It created some > radio noise. It possibly took my GPS antenna offline (or it might have > just been the location, which I also changed). > > So, why was the brand new magneto creating noise ... because when I > bought the plane it was set up to use auto spark plugs, specifically > Autolite 386. Other than some radio noise, the previous owner didn't > have any issues because he had few electronics in the plane, mostly > steam gauges. My theory is that the magneto reacts poorly to high > resistance downstream of it. You are supposed to throw out an aircraft > plug when it hits 5000 ohms, but most Autolite 386 plugs are around > 10000 ohms out of the box. The manufacturerprovides a large range > from 3k to 12k ohms. I tested six new ones and measured between 7k and > 12k ohms. There is probably an auto plug solution that would work fine > on my magento, but I am done experimenting for now. I flew with the > aircraft plugs and it fixed all my problems at once. > > Ron > > On Wed, Aug 26, 2020 at 8:19 PM Robert L. Nuckolls, III > > > wrote: > > At 06:28 PM 8/26/2020, you wrote: >> Just did a test flight and shot some video of the EIS to compare >> to the EFIS. I havent even looked at it yet because I did >> something else for the first time. I shut off the magneto and >> everything got perfectly quiet on the EFIS, along with the noise >> that seems to be getting louder in my headset on each flight! > > > Aha! One more flight test: > > Leave the mag wire totally unhooked at > the engine end. Of course this leaves > you an always-hot mag but the p-lead > effects are now out of the diagnostic > loop. > > If the effect goes away, you're looking > at p-lead; if it stays you're looking > at plug wires. > > > Bob . . . > -- This email has been checked for viruses by Avast antivirus software. https://www.avast.com/antivirus ________________________________________________________________________________
From: Ron Springer <ron228rj(at)gmail.com>
Subject: Re: B&C SD-8 Alternator/Regulator
Date: Sep 14, 2020
Remember, I only discovered that this had something to do with the ignition s ystem in the very last post. This whole thread has been focused on the elect rical system, not the ignition, because the problem could only be produced i n flight shortly after the charging system kicked in at takeoff. It did not o ccur on the ground at any power setting from idle to full throttle. I didn=99t omit any information about the spark plugs because we haven =99t been talking about the ignition system in this thread. I dealt wi th that on my own. Ignition noise is usually suppressed by adding resistance to the plug, wires , or both to reduce current at the expense of performance. In this case, it a ppears that there may be too much resistance causing the mag to behave badly . The Autolite plugs on my electronic ignition work just fine. It was just a problem with the mag. I had two sets of auto harnesses to choose from. One was a modified aircraft harness with auto plug terminals. That is low resistance wire with a metal s hield. I also had a harness utilizing 8.8 mm Accel wires with a resistance o f about 700 ohms, advertised to be RFI suppression wires. Neither set of wir es made a difference. It was all due to the plugs. That website you mentione d advertises low resistance MSD wires on their auto plug harness but they do n=99t mention the plugs that they recommend for a magneto. Ron Sent from my iPhone > On Sep 14, 2020, at 9:58 PM, Charlie England wrote: > > =EF=BB > Yeah...That would have been a useful data point to supply for troubleshoot ing. ;-) > > Some have gotten away with using specific plug wires & automotive plugs wi th mags, but the whole reason mags have shielded wire and shielded plugs is t o suppress the intense RF ignition noise from the mag. > > Here's one way to do it successfully: > http://www.g3ignition.com/magmod.html > Note that the 386s are being used by some of us with electronic ignition, b ut internet lore says they have a higher heat range than typical a/c plugs, o r the metric plugs that require an adapter. > > Charlie > >> On 9/14/2020 6:54 PM, Ron Springer wrote: >> To close this one out ... >> >> The source of the noise was the magneto itself. Disconnect the p-lead to t he engine monitor, and all of the data was stable. However, the magneto was s till radiating RF that was causing other problems. It would stop data record ing to the USB stick on my EFIS. It created some radio noise. It possibly to ok my GPS antenna offline (or it might have just been the location, which I a lso changed). >> >> So, why was the brand new magneto creating noise ... because when I bough t the plane it was set up to use auto spark plugs, specifically Autolite 386 . Other than some radio noise, the previous owner didn't have any issues bec ause he had few electronics in the plane, mostly steam gauges. My theory is t hat the magneto reacts poorly to high resistance downstream of it. You are s upposed to throw out an aircraft plug when it hits 5000 ohms, but most Autol ite 386 plugs are around 10000 ohms out of the box. The manufacturer provide s a large range from 3k to 12k ohms. I tested six new ones and measured betw een 7k and 12k ohms. There is probably an auto plug solution that would work fine on my magento, but I am done experimenting for now. I flew with the ai rcraft plugs and it fixed all my problems at once. >> >> Ron >> >> On Wed, Aug 26, 2020 at 8:19 PM Robert L. Nuckolls, III <nuckolls.bob@aer oelectric.com> wrote: >>> At 06:28 PM 8/26/2020, you wrote: >>>> Just did a test flight and shot some video of the EIS to compare to the EFIS. I haven=C3=A2=82=AC=84=A2t even looked at it yet because I did s omething else for the first time. I shut off the magneto and everything got p erfectly quiet on the EFIS, along with the noise that seems to be getting lo uder in my headset on each flight! >>> >>> >>> Aha! One more flight test: >>> >>> Leave the mag wire totally unhooked at >>> the engine end. Of course this leaves >>> you an always-hot mag but the p-lead >>> effects are now out of the diagnostic >>> loop. >>> >>> If the effect goes away, you're looking >>> at p-lead; if it stays you're looking >>> at plug wires. >>> >>> >>> >>> Bob . . . >>> > > > Virus-free. www.avast.com ________________________________________________________________________________
From: mike Pienaar <mikepienaar09(at)gmail.com>
Date: Sep 14, 2020
Subject: Re: Regulator failure
1. Ran when parked 2. Generator is believed to be a converted Evenrude generator 3 Not sure about the amps 4 Is a generator 5 Pictures will follow Thanks Mike On Mon, Sep 14, 2020 at 6:43 PM Jeff Luckey wrote: > Mike, > > 1. Do you know that the wiring was good before the flight? > 2. Part numbers / nameplate data for generator & regulator? > 3. How many amps can this generator generate? > 4. It's a generator and not an alternator? > 5. Any pictures? > > > -Jeff > > > mikepienaar09(at)gmail.com> wrote: > > > A friend has an aircraft with a wind/propellor driven generator controlled > by a "Ford type" regulator. The aircraft made its first flight in 6 years > recently and the regulator was completely fried. Everything inside was heat > damaged and the insulation on the wires entering the regulator melted off. > > Where do we start looking? > > Thanks > > Mike > ________________________________________________________________________________
Date: Sep 15, 2020
From: "Robert L. Nuckolls, III" <nuckolls.bob(at)aeroelectric.com>
Subject: Re: Sealed lead Acid battery
At 01:51 PM 9/14/2020, you wrote: >Charlie thanks, >I too have been using cheap chinese SLA >batteries which is the only small ish battery I >could obtain in Colombia.=C2 It is also the >normally used rotax battery there except=C2 where >the battery is in the engine compartment because >the belief is that SLA dont handle heat >well.=C2 =C2 Starts my 912 turbo fine.=C2 But the >stupidest=C2 question is the one you don't >ask.=C2 And since I'm now in the US I have a few more alternatives... Rule #1, it's dumb to argue with success. Rule #2, when initiating a new experiment, you need data based on observation. When selecting a battery for an electrically dependent airplane, one needs a battery-only, endurance goal. Then be ready/willing to test the battery against that requirement. Swap it out for a new one when it fails the endurance test even if it still starts the engine, We've had many discussions here on the List over the years about suitability to task for a host of batteries. Some folks swear by the premium name brands, others have cited years of satisfactory service from generic products. But WITHOUT NUMBERS demonstrating compliance with numerically defined design goals, those conversations were little more than musings over beer and pretzels. For the most part, any and all SVLA products are fair game for use in your airplane. The deciding factor is $ownership$. How much $testing$ and $procurement$ expenses are incurred to keep your design goals covered? Years ago, some of my readers bought the cheapest, 18AH SLVA they could find (about $40 at the time) and simply replaced it every year. If they flew 50 hours a year, then operating expense was under a $1/hr and testing expense was zero. A really expensive battery may prove to the the lowest cost per flight hour when demonstrated performance negates a lot of maintenance testing. If you have no battery-only endurance requirements, then you can run any battery like they do in most GA light aircraft . . . run 'em until they don't crank the engine any more. But keep in mind that most of those dark-n-stormy-night stories involving soggy batteries were penned by pilots who didn't recognize that they had battery-only endurance goals until it was too late to do anything about it. So, pick a battery. Set goals. Monitor performance over time and after you've gone through two or three batteries, you'll KNOW if it was a good value. We'd all appreciate it if you would share your discoveries here on the List. Bob . . . ________________________________________________________________________________
Subject: Re: Regulator failure
From: "user9253" <fransew(at)gmail.com>
Date: Sep 15, 2020
How many wires are coming out of the generator, 2 or 3 or ? Do you know if the output is AC or DC? If not, can you measure it? I suggest that the generator be disconnected from the aircraft electrical system. Then connect the generator output to a high wattage load such as an automotive headlight and go for a test flight. Measure the AC and DC voltage and current through the load. If you suspect the unregulated generator voltage and current might be too high for a single headlight, connect two headlights in series. -------- Joe Gores Read this topic online here: http://forums.matronics.com/viewtopic.php?p=498372#498372 ________________________________________________________________________________
From: Charlie England <ceengland7(at)gmail.com>
Date: Sep 15, 2020
Subject: Re: Brownout Step Up Converter
Situation: I have only one device (EFIS) that's susceptible to brownout, and don't need a relay for my endurance bus. Is there any reason to *not* leave the brownout booster in the circuit continuously? Ex: boost/buck inverter between main bus & EFIS input. In case of inverter death, either a diode (simpler), or a simple switch, wired 'around' the inverter. Only unprotected failure mode I can see is the inverter going overvoltage; that could be handled with an OV module if the risk is high enough. Thoughts? Thanks, Charlie > ________________________________________________________________________________
From: Bill Watson <Mauledriver(at)nc.rr.com>
Subject: Installed new battery, Bus voltage went up over 15volts
Date: Sep 15, 2020
I am running a Z-14 electrical system with dual batts/alts/buses. I recently changed out an old Odyssey 680 for a new one after 4+ years of service because it's fully charged, no load voltage fell to 12.5 volts. I swapped the battery positions as I put in the new one per my normal procedure. On the 2nd leg of the first flight after the swap I noticed that the bus with the new battery was running slightly over 15 volts. Normally it runs between 14.5 and 14.7. After some switching of avionics and other components, I took it off line. Is there any obvious reason a new battery would cause a bus to suddenly run at a higher voltage when at cruise and presumably the power is being supplied by an alternator and the voltage controlled by an B&C regulator? 15 volts is too high a charging voltage for an Odyssey per their user doc. My inclination is to adjust the regulator but I don't recall having to do this with past battery changes or swaps (the regulator is really hard to access so there's that too). Thoughts and opinions welcome. Bill "everytime I get comfortable with a system something new pops up" Watson -- This email has been checked for viruses by Avast antivirus software. https://www.avast.com/antivirus ________________________________________________________________________________
Subject: Re: Installed new battery, Bus voltage went up over
15volts
From: Charlie England <ceengland7(at)gmail.com>
Date: Sep 15, 2020
On 9/15/2020 9:22 AM, Bill Watson wrote: > > > I am running a Z-14 electrical system with dual batts/alts/buses. I > recently changed out an old Odyssey 680 for a new one after 4+ years > of service because it's fully charged, no load voltage fell to 12.5 > volts. I swapped the battery positions as I put in the new one per my > normal procedure. > > On the 2nd leg of the first flight after the swap I noticed that the > bus with the new battery was running slightly over 15 volts. Normally > it runs between 14.5 and 14.7. After some switching of avionics and > other components, I took it off line. > > Is there any obvious reason a new battery would cause a bus to > suddenly run at a higher voltage when at cruise and presumably the > power is being supplied by an alternator and the voltage controlled by > an B&C regulator? 15 volts is too high a charging voltage for an > Odyssey per their user doc. > > My inclination is to adjust the regulator but I don't recall having to > do this with past battery changes or swaps (the regulator is really > hard to access so there's that too). > > Thoughts and opinions welcome. > > Bill "everytime I get comfortable with a system something new pops up" > Watson > > Seems unlikely that the battery change would cause it, but easy enough to verify, if you still have the old battery. Just return everything to pre-replacement configuration & re-check. -- This email has been checked for viruses by Avast antivirus software. https://www.avast.com/antivirus ________________________________________________________________________________
From: Charlie England <ceengland7(at)gmail.com>
Date: Sep 15, 2020
Subject: Re: B&C SD-8 Alternator/Regulator
Understand. Apologies if that was offensive; it was intended to be light hearted. I would have bet that it didn't show up at low throttle because the ignition isn't working hard until MAP goes up at higher power settings, but that doesn't square with your symptoms. One thing that plug wire mfgers say in their docs is that it's important to use compatible wire & plugs. IIRC, solid conductors with resistor plugs or resistor wire with non-resistor plugs. That site I linked will probably tell you which plugs to use when you pay them their $30. :-) Just out of curiosity, did you try a new set of 386s? Any chance the original setup had a different plug; perhaps a 14mm plug with thread adapter? My plane had 14mm plugs on the Lightspeed ignition when I bought it, but I changed to 386s out of paranoia when I found one of the adapters backed out about half way about a month after purchasing the plane. Charlie On Mon, Sep 14, 2020 at 11:06 PM Ron Springer wrote: > Remember, I only discovered that this had something to do with the > ignition system in the very last post. This whole thread has been focused > on the electrical system, not the ignition, because the problem could onl y > be produced in flight shortly after the charging system kicked in at > takeoff. It did not occur on the ground at any power setting from idle to > full throttle. > > I didn=99t omit any information about the spark plugs because we ha ven=99t > been talking about the ignition system in this thread. I dealt with that on > my own. > > Ignition noise is usually suppressed by adding resistance to the plug, > wires, or both to reduce current at the expense of performance. In this > case, it appears that there may be too much resistance causing the mag to > behave badly. The Autolite plugs on my electronic ignition work just fine . > It was just a problem with the mag. > > I had two sets of auto harnesses to choose from. One was a modified > aircraft harness with auto plug terminals. That is low resistance wire wi th > a metal shield. I also had a harness utilizing 8.8 mm Accel wires with a > resistance of about 700 ohms, advertised to be RFI suppression wires. > Neither set of wires made a difference. It was all due to the plugs. That > website you mentioned advertises low resistance MSD wires on their auto > plug harness but they don=99t mention the plugs that they recommend for a > magneto. > > Ron > > Sent from my iPhone > > On Sep 14, 2020, at 9:58 PM, Charlie England wrote : > > =EF=BB > Yeah...That would have been a useful data point to supply for > troubleshooting. ;-) > > Some have gotten away with using specific plug wires & automotive plugs > with mags, but the whole reason mags have shielded wire and shielded plug s > is to suppress the intense RF ignition noise from the mag. > > Here's one way to do it successfully: > http://www.g3ignition.com/magmod.html > Note that the 386s are being used by some of us with electronic ignition, > but internet lore says they have a higher heat range than typical a/c > plugs, or the metric plugs that require an adapter. > > Charlie > > On 9/14/2020 6:54 PM, Ron Springer wrote: > > To close this one out ... > > The source of the noise was the magneto itself. Disconnect the p-lead to > the engine monitor, and all of the data was stable. However, the magneto > was still radiating RF that was causing other problems. It would stop dat a > recording to the USB stick on my EFIS. It created some radio noise. It > possibly took my GPS antenna offline (or it might have just been the > location, which I also changed). > > So, why was the brand new magneto creating noise ... because when I bough t > the plane it was set up to use auto spark plugs, specifically Autolite 38 6. > Other than some radio noise, the previous owner didn't have any issues > because he had few electronics in the plane, mostly steam gauges. My theo ry > is that the magneto reacts poorly to high resistance downstream of it. Yo u > are supposed to throw out an aircraft plug when it hits 5000 ohms, but mo st > Autolite 386 plugs are around 10000 ohms out of the box. The > manufacturer provides a large range from 3k to 12k ohms. I tested six new > ones and measured between 7k and 12k ohms. There is probably an auto plug > solution that would work fine on my magento, but I am done experimenting > for now. I flew with the aircraft plugs and it fixed all my problems at > once. > > Ron > > On Wed, Aug 26, 2020 at 8:19 PM Robert L. Nuckolls, III < > nuckolls.bob(at)aeroelectric.com> wrote: > >> At 06:28 PM 8/26/2020, you wrote: >> >> Just did a test flight and shot some video of the EIS to compare to the >> EFIS. I haven=C3=A2=82=AC=84=A2t even looked at it yet because I d id something else for >> the first time. I shut off the magneto and everything got perfectly quie t >> on the EFIS, along with the noise that seems to be getting louder in my >> headset on each flight! >> >> >> >> Aha! One more flight test: >> >> Leave the mag wire totally unhooked at >> the engine end. Of course this leaves >> you an always-hot mag but the p-lead >> effects are now out of the diagnostic >> loop. >> >> If the effect goes away, you're looking >> at p-lead; if it stays you're looking >> at plug wires. >> >> >> >> Bob . . . >> > > > Virus-free . > www.avast.com > > <#m_8962622669845041058_DAB4FAD8-2DD7-40BB-A1B8-4E2AA1F9FDF2> > > ________________________________________________________________________________
Date: Sep 15, 2020
From: Jeff Luckey <jluckey(at)pacbell.net>
Subject: Re: Installed new battery, Bus voltage went up over
15volts Bill, Thinking out loud here: The only way to get voltage that high is from the alternator (kinda obvious ) and the thing that controls the alternator is the regulator (again, obvio us) therefore we might concentrate on the regulator.=C2- (I know, brillia nt deductive reasoning ;) I'm not familiar with the B&C regulator but I think it has a voltage-sense terminal.=C2- Any chance something weird happened with the sense lead?=C2 - If the regulator is not getting accurate voltage signal it might over-c ontrol the field?? Anecdote:A few years ago a friend has a similar but not identical problem. =C2- The master had been left on and drained the battery.=C2- The batte ry was removed and charged for an hour or two outside the airplane.=C2- H e re-installed the battery and flew the plane and the voltage went crazy an d fried his avionics stack ($$ouch!$$).=C2- There was no over-voltage pro tection module. We have never come up with a scenario that explains the failure.=C2- (Thi s is probably due to incomplete information regarding the details of the ev ent.=C2- I don't have first-hand knowledge of the details, so when we dis cuss it over beers, the details get a little "fuzzy".) I'm interested in some/any hypothesis that might explain that behavior.=C2 - It might be relevant to the situation at hand. -JL=C2- er(at)nc.rr.com> wrote: m> I am running a Z-14 electrical system with dual batts/alts/buses.=C2- I recently changed out an old Odyssey 680 for a new one after 4+ years of service because=C2- it's fully charged, no load voltage fell to 12.5 volts.=C2- I swapped the battery positions as I put in the new one per my normal procedure. On the 2nd leg of the first flight after the swap I noticed that the bus with the new battery was running slightly over 15 volts. Normally it=C2- runs between 14.5 and 14.7.=C2- After some switching of avionics and othe r components, I took it off line. Is there any obvious reason a new battery would cause a bus to suddenly run at a higher voltage when at cruise and presumably the power is being supplied by an alternator and the voltage controlled by an B&C regulator?=C2- 15 volts is too high a charging voltage for an Odyssey per their user doc. My inclination is to adjust the regulator but I don't recall having to do this with past battery changes or swaps (the regulator is really hard to access so there's that too). Thoughts and opinions welcome. Bill "everytime I get comfortable with a system something new pops up" Watson -- This email has been checked for viruses by Avast antivirus software. https://www.avast.com/antivirus - S - WIKI - - =C2- =C2- =C2- =C2- =C2- -Matt Dralle, List Admin. ________________________________________________________________________________
From: Ron Springer <ron228rj(at)gmail.com>
Date: Sep 15, 2020
Subject: Re: B&C SD-8 Alternator/Regulator
Well, when I first tore out all the wiring and installed an EFIS in my plane, I refreshed all my Autolite 386 plugs with new ones. I did not have any problem with noise in the data like I had recently, but on my third flight my EFIS did not record data to the USB stick. I suspected a problem with the EFIS when that happened. I certainly wasn't thinking it was caused by the spark plugs at the time. In some of the following flights it would periodically not record portions of the flight. However, as soon as I installed a brand new mag, I got extreme noise on the engine data plus failure to record data on every single flight starting shortly after takeoff. So, I have not tried a set of new plugs recently. Maybe it was not due to the high resistance but one of the plugs was defective instead? Blaming the problem on high resistance is only a theory at this point. However, I have done nine test flights in the pattern sorting this out over the last couple months, and now it is time to go places instead of experimenting any further. Summer is almost over. I will gladly pay for expensive aircraft plugs to put these problems behind me. Ron On Tue, Sep 15, 2020 at 1:11 PM Charlie England wrote: > > Just out of curiosity, did you try a new set of 386s? Any chance the > original setup had a different plug; perhaps a 14mm plug with thread > adapter? My plane had 14mm plugs on the Lightspeed ignition when I bought > it, but I changed to 386s out of paranoia when I found one of the adapters > backed out about half way about a month after purchasing the plane. > > Charlie > ________________________________________________________________________________
Subject: Re: Brownout Step Up Converter
From: "user9253" <fransew(at)gmail.com>
Date: Sep 15, 2020
Charlie, how about connecting a diode in parallel with the DC-DC converter like you mentioned, but instead of running the DC-DC converter continuously, set its output at just below 12 volts. Then the EFIS will normally be powered via the parallel diode and not by the DC-DC converter. Another option is to use a backup battery. Dual diodes in one package like 863-MBRF20L45CTG can power the EFIS from the source with the highest voltage. -------- Joe Gores Read this topic online here: http://forums.matronics.com/viewtopic.php?p=498380#498380 ________________________________________________________________________________
Subject: Re: Installed new battery, Bus voltage went up over
15volts
From: "user9253" <fransew(at)gmail.com>
Date: Sep 15, 2020
Make sure the voltage regulator has a good ground connection and that the voltage sense terminal has the exact same voltage as the battery. -------- Joe Gores Read this topic online here: http://forums.matronics.com/viewtopic.php?p=498381#498381 ________________________________________________________________________________
Subject: Re: Brownout Step Up Converter
From: Charlie England <ceengland7(at)gmail.com>
Date: Sep 15, 2020
On 9/15/2020 1:37 PM, user9253 wrote: > > Charlie, how about connecting a diode in parallel with the DC-DC converter > like you mentioned, but instead of running the DC-DC converter continuously, > set its output at just below 12 volts. Then the EFIS will normally be powered > via the parallel diode and not by the DC-DC converter. > Another option is to use a backup battery. > Dual diodes in one package like 863-MBRF20L45CTG can power the > EFIS from the source with the highest voltage. > > -------- > Joe Gores Good thought on the voltage setting; that's basically what we'd have with the AEC circuit, too. I've long thought about using the boost/buck converter to both isolate and keep fully charged a backup battery, but I do like the simplicity of Bob's 'one battery to rule them all' philosophy. Also, having a backup battery outside the EFIS (an old GRT HX) would mean that it wouldn't turn off with the master, meaning that I'd soon forget to power it off, meaning.... ;-) Charlie -- This email has been checked for viruses by Avast antivirus software. https://www.avast.com/antivirus ________________________________________________________________________________
Subject: Re: Brownout Step Up Converter
From: "user9253" <fransew(at)gmail.com>
Date: Sep 15, 2020
How about using the DC-DC converter (no backup battery), but only energize it during engine start? A dual diode will allow either the DC-DC converter or the power bus to supply current to the EFIS. Wired that way, there is no concern about the DC-DC converter malfunctioning during flight. -------- Joe Gores Read this topic online here: http://forums.matronics.com/viewtopic.php?p=498385#498385 ________________________________________________________________________________
Subject: Re: Brownout Step Up Converter
From: Charlie England <ceengland7(at)gmail.com>
Date: Sep 15, 2020
On 9/15/2020 6:54 PM, user9253 wrote: > > How about using the DC-DC converter (no backup battery), but only energize > it during engine start? A dual diode will allow either the DC-DC > converter or the power bus to supply current to the EFIS. Wired that way, > there is no concern about the DC-DC converter malfunctioning during flight. > > -------- > Joe Gores > > That's basically what is shown in Z101. I suppose it's worth giving a random choice a try configured to the start button, and if it doesn't work I could move it to the bus. Thanks, Charlie -- This email has been checked for viruses by Avast antivirus software. https://www.avast.com/antivirus ________________________________________________________________________________
From: William Daniell <wdaniell.longport(at)gmail.com>
Date: Sep 15, 2020
Subject: Re: Sealed lead Acid battery
Roger that. William Daniell +1 786 878 0246 On Tue, Sep 15, 2020, 07:50 Robert L. Nuckolls, III < nuckolls.bob(at)aeroelectric.com> wrote: > At 01:51 PM 9/14/2020, you wrote: > > Charlie thanks, > I too have been using cheap chinese SLA batteries which is the only small > ish battery I could obtain in Colombia.=C3=82 It is also the normally us ed > rotax battery there except=C3=82 where the battery is in the engine compa rtment > because the belief is that SLA dont handle heat well.=C3=82 =C3=82 Start s my 912 > turbo fine.=C3=82 But the stupidest=C3=82 question is the one you don't ask.=C3=82 And > since I'm now in the US I have a few more alternatives... > > > Rule #1, it's dumb to argue with success. > > Rule #2, when initiating a new experiment, you > need data based on observation. > > When selecting a battery for an electrically dependent > airplane, one needs a battery-only, endurance > goal. Then be ready/willing to test the battery > against that requirement. Swap it out for a new one when it > fails the endurance test even if it still starts the engine, > > We've had many discussions here on the List over > the years about suitability to task for a host > of batteries. Some folks swear by the premium > name brands, others have cited years of satisfactory > service from generic products. > > But WITHOUT NUMBERS demonstrating compliance with > numerically defined design goals, those > conversations were little more than musings > over beer and pretzels. > > For the most part, any and all SVLA products > are fair game for use in your airplane. The > deciding factor is $ownership$. How much > $testing$ and $procurement$ expenses are > incurred to keep your design goals covered? > > Years ago, some of my readers bought the cheapest, > 18AH SLVA they could find (about $40 at the time) and > simply replaced it every year. If they flew 50 > hours a year, then operating expense was under a > $1/hr and testing expense was zero. > > A really expensive battery may prove to the > the lowest cost per flight hour when > demonstrated performance negates a lot of > maintenance testing. > > If you have no battery-only endurance requirements, > then you can run any battery like they do in most > GA light aircraft . . . run 'em until they don't > crank the engine any more. But keep in mind that > most of those dark-n-stormy-night stories involving > soggy batteries were penned by pilots who didn't > recognize that they had battery-only endurance > goals until it was too late to do anything about it. > > So, pick a battery. Set goals. Monitor performance > over time and after you've gone through two or > three batteries, you'll KNOW if it was a good > value. We'd all appreciate it if you would share > your discoveries here on the List. > > > Bob . . . > ________________________________________________________________________________
Subject: Rotax 912 Eating Ignition Modules
From: "Guy Buchanan" <gebuchanan(at)cox.net>
Date: Sep 16, 2020
A friend has a 912uls in a Kitfox that seems to be eating ignition modules. He's gone through three. Yes they were all old. Yes new ones work, but we're worried that maybe his plane is producing electrical noise detrimental to the modules. We did some testing using an oscilloscope looking for voltage spikes in the ground wire during start up and shutoff. For various reasons we weren't able to capture any voltage spikes during start and stop, but did capture some very messy noise signatures while running. Picture 1 shows one module's ground wire while "open". Note +/- 30V peaks and a regular 30hz sawtooth. Picture 2 shows the same ground "grounded". We did some further testing of the input and output to the voltage regulator and those voltages appeared relatively clean. Question: is it possible the ignition module is being damaged by the voltage noise, regardless of source? Should we apply some caps to the ground wires to kill the noise? Thanks, Guy Buchanan gebuchanan(at)cox.net -------- Guy Buchanan K-IV 1200 / 912Uls / Warp / 500 hours and flying again. Now a glider pilot too. Read this topic online here: http://forums.matronics.com/viewtopic.php?p=498392#498392 Attachments: http://forums.matronics.com//files/labnation_screenshot2_759.png http://forums.matronics.com//files/labnation_screenshot0_837.png ________________________________________________________________________________
Date: Sep 17, 2020
From: "Robert L. Nuckolls, III" <nuckolls.bob(at)aeroelectric.com>
Subject: Re: Brownout Step Up Converter
At 06:54 PM 9/15/2020, you wrote: > >How about using the DC-DC converter (no backup battery), but only energize >it during engine start? A dual diode will allow either the DC-DC >converter or the power bus to supply current to the EFIS. Wired that way, >there is no concern about the DC-DC converter malfunctioning during flight. > >-------- >Joe Gores Yup . . . see Z101 Bob . . . ________________________________________________________________________________
From: David Carter <david(at)carter.net>
Date: Sep 17, 2020
Subject: Difference between B&C Starter Contactors?
Anyone know if there is any practical difference between the S702-1 & the S811-1, besides $10.00? I'm redoing the electrical system in my RV-7A (I'm not the builder) per Z101, and want to replace the existing contactors, which I believe are the ones Vans sells. The form factor on the B&C starter contactors is different, which will make it slightly more complicated to retrofit. Is it worth the effort? Thanks, David --- David Carter david(at)carter.net ________________________________________________________________________________
Subject: Re: Difference between B&C Starter Contactors?
From: "user9253" <fransew(at)gmail.com>
Date: Sep 17, 2020
The $10 might pay for the PMA. -------- Joe Gores Read this topic online here: http://forums.matronics.com/viewtopic.php?p=498396#498396 ________________________________________________________________________________
Date: Sep 17, 2020
From: Jeff Luckey <jluckey(at)pacbell.net>
Subject: Re: Difference between B&C Starter Contactors?
Hi David, It is my understanding that the S702-1 (sometimes called a "pancake" soleno id) is a solenoid that has been optimized for cranking duty.=C2- I believ e that Ford created the design some time in the 70s specifically to start e ngines in Ford vehicles.=C2- Things like contact size and contact pressur e were optimized for the specific task of short-duration high-current loads encountered when starting engines.=C2- I believe that this makes the des ign less likely to be a victim of the "weld shut" scenario that we occasion ally hear about with older tin-can types.=C2- =C2-Based on this info I have decided to use it in my RV-7 and in a friend's RV-14. The other "tin can" configuration is a more general-purpose solenoid.=C2- It seems to work fine as it is in thousands of little GA aircraft. Some advantages of the S702-1:1. Optimized for the task of starting2. Ubiqu itous - is in millions of Ford cars & trucks - you can get one at almost an y auto parts store (even in PoDunk North Dakota)3. Cost effective Jeff Luckey arter.net> wrote: Anyone know if there is any practical difference=C2-between the S702-1 & the S811-1, besides $10.00?=C2- I'm redoing the electrical=C2-system i n my RV-7A (I'm not the builder) per Z101, and want to replace the existing contactors, which I believe=C2-are the ones Vans sells. The form factor on the B&C starter contactors is different, which will make it slightly mor e complicated to retrofit. Is it worth the effort?=C2- Thanks,David --- David Carter david(at)carter.net ________________________________________________________________________________
Date: Sep 17, 2020
From: "Robert L. Nuckolls, III" <nuckolls.bob(at)aeroelectric.com>
Subject: Re: Difference between B&C Starter Contactors?
At 11:21 AM 9/17/2020, you wrote: >Anyone know if there is any practical >difference=C2 between the S702-1 & the S811-1, >besides $10.00?=C2 I'm redoing the electrical=C2 >system in my RV-7A (I'm not the builder) per >Z101, and want to replace the existing >contactors, which I believe=C2 are the ones Vans >sells. The form factor on the B&C starter >contactors is different, which will make it >slightly more complicated to retrofit. Is it worth the effort?=C2 Go here and get one of these https://tinyurl.com/y5yzynuq or get the same part from your local Smiley Jack's Car Parts emporium: Standard SS598T Works good, lasts a long time. Bob . . . ________________________________________________________________________________
Date: Sep 17, 2020
From: "Robert L. Nuckolls, III" <nuckolls.bob(at)aeroelectric.com>
Subject: Re: Difference between B&C Starter Contactors?
At 12:17 PM 9/17/2020, you wrote: >Hi David, > >It is my understanding that the S702-1 (sometimes called a "pancake" >solenoid) is a solenoid that has been optimized for cranking >duty. I believe that Ford created the design some time in the 70s >specifically to start engines in Ford vehicles. Things like contact >size and contact pressure were optimized for the specific task of >short-duration high-current loads encountered when starting >engines. I believe that this makes the design less likely to be a >victim of the "weld shut" scenario that we occasionally hear about >with older tin-can types. Based on this info I have decided to use >it in my RV-7 and in a friend's RV-14. > >The other "tin can" configuration is a more general-purpose >solenoid. It seems to work fine as it is in thousands of little GA aircraft. The all metal, contactors are available in an intermittent duty style but they're hard to find . . . and yes, they feature a large-area, low-pressure contact not ideally suited to hi-inrush loads. But they were used by the thousands in all manner of vehicle including aircraft for decades. The more nimble footed, small-area, high-pressure contact devices are more suited to this application. They've become the gold-standard for resistance to sticking . . . ESPECIALLY with permanent magnet starters. The Standard SS598T is but one example of dozens in this class of device. Inexpensive and very reliable in aircraft. Keep in mind that aircraft engines get started what . . . maybe 50 times a year? My car gets started 50 times a month! Bob . . . ________________________________________________________________________________
From: Sebastien <cluros(at)gmail.com>
Date: Sep 17, 2020
Subject: Re: Brownout Step Up Converter
A couple weeks ago I ordered one of these off eBay: https://www.ebay.ca/itm/192243452240 DC-DC Converter 5 Amps Input 3-35 V Output 5-40 V I'm trying to support ~1 A EFIS during start so I thought 5 A would be plenty but a closer look at the description says it's good for 3 A with a maximum of 5 A. Turns out the 3 A is on the input side, not the output side. With a voltage output set point of 10V and a ~1 A lamp connected, the lamp browns out (literally) as soon as the input voltage sags below 9V. A long way from the 7V minimum I was hoping for. I plugged it into a Battery Analyzer at 1.0 and 0.5 A and momentarily dropped the input voltage to 8V. Even when voltage is restored it takes several seconds to recover back to the set voltage with a 1 A load. A 0.5 A load is fine. [image: Brownout Booster.jpg] So I think these things work fine but are undersized for the application. Unfortunately as soon as I look for 10A or 15A ones the minimum input voltage rises to 8.5 or even 10V. I'm ready to give up on this little project unless someone has found a cheap step up converter that will output at least 1 A @ 10 V with the bus voltage available during engine start. On Thu, Sep 17, 2020 at 7:08 AM Robert L. Nuckolls, III < nuckolls.bob(at)aeroelectric.com> wrote: > At 06:54 PM 9/15/2020, you wrote: > > > How about using the DC-DC converter (no backup battery), but only energize > it during engine start? A dual diode will allow either the DC-DC > converter or the power bus to supply current to the EFIS. Wired that way, > there is no concern about the DC-DC converter malfunctioning during flight. > > -------- > Joe Gores > > > Yup . . . see Z101 > > > Bob . . . > ________________________________________________________________________________
Subject: Need small physical size fuse 1/2 amp
From: "rparigoris" <rparigor(at)hotmail.com>
Date: Sep 18, 2020
Hi Group I need to wire several LEDs and small relays. I want to use small wire to do so and would like to steal some electron flow from some near by sources which are fused well above rating of small wire. Thus I want to install fuses. What are some options for small physical size fuses? At moment I'm thinking something I can solder and shrink wrap onto the wire? Thx. Ron P. Read this topic online here: http://forums.matronics.com/viewtopic.php?p=498418#498418 ________________________________________________________________________________
Subject: Re: Need small physical size fuse 1/2 amp
From: "user9253" <fransew(at)gmail.com>
Date: Sep 18, 2020
https://www.mouser.com/ProductDetail/Littelfuse/0217400MXEP?qs=LpXnx4WbYtDEKhG6ZVugOw%3D%3D -------- Joe Gores Read this topic online here: http://forums.matronics.com/viewtopic.php?p=498420#498420 ________________________________________________________________________________
Subject: Re: Need small physical size fuse 1/2 amp
From: Charlie England <ceengland7(at)gmail.com>
Date: Sep 18, 2020
On 9/18/2020 7:09 PM, user9253 wrote: > > https://www.mouser.com/ProductDetail/Littelfuse/0217400MXEP?qs=LpXnx4WbYtDEKhG6ZVugOw%3D%3D > > -------- > Joe Gores And if you're good with a soldering iron, you can solder directly to a glass fuse yourself. But glass fuses are, in my experience, less than reliable devices, especially in the lower amp values. Here are some options that don't 'package' as well, but might be more reliable. https://en.wikipedia.org/wiki/Fuse_(automotive) Lots of other options, depending on how much you're willing to pay. https://www.mouser.com/Circuit-Protection/Fuses/Fuses-with-Leads-Through-Hole/_/N-ba8hh?P=1yocc0pZ1yzshdi Charlie -- This email has been checked for viruses by Avast antivirus software. https://www.avast.com/antivirus ________________________________________________________________________________
Subject: Alternator sawtooth current generation
From: "rv8ch" <mick-matronics(at)rv8.ch>
Date: Sep 19, 2020
Planepower/Harzell 60Amp, Earthx battery, GRT EIS 4000 w/hall effect sensor on alternator B-lead, running all lights, etc. Steady state getting these kinds of current fluctuations (see attachment). Got some good suggestions from VAF and tried them all, even replaced the alternator, same behavior. https://vansairforce.net/community/showthread.php?t=186043 The current shows a consistent pattern, and regular period of a bit over 11 seconds. I don't know of anything in my aircraft with that period. It's all new, and this has been happening since the start. Happy for any hints! -------- Mickey Coggins http://www.rv8.ch/ Read this topic online here: http://forums.matronics.com/viewtopic.php?p=498424#498424 Attachments: http://forums.matronics.com//files/hb_ymm_alternator_output_20200918_839.png ________________________________________________________________________________
Date: Sep 19, 2020
From: "Robert L. Nuckolls, III" <nuckolls.bob(at)aeroelectric.com>
Subject: Re: Need small physical size fuse 1/2 amp
At 02:01 PM 9/18/2020, you wrote: > >Hi Group I need to wire several LEDs and small relays. I want to use >small wire to do so and would like to steal some electron flow from >some near by sources which are fused well above rating of small >wire. Thus I want to install fuses. What are some options for small >physical size fuses? At moment I'm thinking something I can solder >and shrink wrap onto the wire? Thx. Ron P. > How many 'feeders' at what current levels? Bob . . . ________________________________________________________________________________
From: Charlie England <ceengland7(at)gmail.com>
Date: Sep 19, 2020
Subject: Re: Alternator sawtooth current generation
On Sat, Sep 19, 2020 at 3:59 AM rv8ch wrote: > > Planepower/Harzell 60Amp, Earthx battery, GRT EIS 4000 w/hall effect > sensor on alternator B-lead, running all lights, etc. Steady state getting > these kinds of current fluctuations (see attachment). > > Got some good suggestions from VAF and tried them all, even replaced the > alternator, same behavior. > > https://vansairforce.net/community/showthread.php?t=186043 > > The current shows a consistent pattern, and regular period of a bit over > 11 seconds. > > I don't know of anything in my aircraft with that period. It's all new, > and this has been happening since the start. > > Happy for any hints! > > -------- > Mickey Coggins > http://www.rv8.ch/ > > > Hi Mickey, Questions spawn questions. :-) It doesn't look like there's any issue with the voltage. If you can access a stand-alone high current DC amp meter, that would be my 1st step; wire it in series with the alternator B lead to verify your EIS current readings as vailid. Assuming the current variations are real.... Where's the current sensor installed? On the alt B lead it will show total current. On the Battery 'fat' lead, it will show charge/discharge to/from the battery. On the bus feeder, it will show current consumed by airframe devices. Since it's a Hall sensor, it should be easy to move it around in the electrical system. Move it to the battery's 'fat' pos. lead, to see if the current fluctuates going into the battery. If you see the variations there, repeat the test with an SLA battery.... Try it on the bus feeder. If you didn't see variations on the battery, you should see them going to the airframe's devices. Start with everything turned on, and one by one, turn off devices, until nothing is left powered in the plane. If it's a device causing the variations, it should be obvious which one is doing it. Having said that, a 10A 'bump' in current, going to zero current, on an 11 second cycle rate, does not sound like any typical current consuming device in our planes (you don't have anything like electric seat heaters, do you?). My money is 1st on measurement error, and my hedge is the battery. Charlie ________________________________________________________________________________
Date: Sep 19, 2020
From: "Robert L. Nuckolls, III" <nuckolls.bob(at)aeroelectric.com>
Subject: Re: Alternator sawtooth current generation
At 03:55 AM 9/19/2020, you wrote: > >Planepower/Harzell 60Amp, Earthx battery, GRT EIS 4000 w/hall effect >sensor on alternator B-lead, running all lights, etc. Steady state >getting these kinds of current fluctuations (see attachment). how big are the variations? Is vertical scale on the plot zero to some max amps? >Got some good suggestions from VAF and tried them all, even replaced >the alternator, same behavior. > >https://vansairforce.net/community/showthread.php?t=186043 > >The current shows a consistent pattern, and regular period of a bit >over 11 seconds. > >I don't know of anything in my aircraft with that period. It's all >new, and this has been happening since the start. What's the bus voltage doing during this time? As long as the bus voltage is reasonably constant and the alternator is not overloaded, I'm not sure current flow from the alternator has much significance. The alternator has no control over current . . . only voltage and that by way of commands from the regulator. So it was safe bet that replacing the alternator wouldn't 'fix' it. But it's not really clear that anything needs fixing . . . need to know the scale factors for that current plot. Bob . . . ________________________________________________________________________________
Date: Sep 19, 2020
From: "Robert L. Nuckolls, III" <nuckolls.bob(at)aeroelectric.com>
Subject: Re: Alternator sawtooth current generation
>Hi Mickey, > >Questions spawn questions. :-) >It doesn't look like there's any issue with the voltage. > >If you can access a stand-alone high current DC >amp meter, that would be my 1st step; wire it in >series with the alternator B lead to verify your >EIS current readings as vailid.=C2 Right on >My money is 1st on measurement error, and my hedge is the battery. > >Charlie Agreed. I'd conduct some experiments with the sensor and compared with another instrument and battery power only. See if that puppy's calibration is good. The 11-second period is interesting . . . wondering if this might be a 'beat note' between two regular features of noise on the bus irritating the a/d conversion process in the instrumentation. But if the bus voltage is stable then some sort of measurement error is high on the list of probabilities. Bob . . . ________________________________________________________________________________
Date: Sep 19, 2020
From: "Robert L. Nuckolls, III" <nuckolls.bob(at)aeroelectric.com>
Subject: Re: Brownout Step Up Converter
At 11:02 PM 9/17/2020, you wrote: >A couple weeks ago I ordered one of these off eBay: > >https://www.ebay.ca/itm/192243452240 > >DC-DC Converter >5 Amps >Input 3-35 V >Output 5-40 V > >I'm trying to support ~1 A EFIS during start so I thought 5 A would >be plenty but a closer look at the description says it's good for 3 >A with a maximum of 5 A. Turns out the 3 A is on the input side, not >the output side. With a voltage output set point of 10V and a ~1 A >lamp connected, the lamp browns out (literally) as soon as the input >voltage sags below 9V. A long way from the 7V minimum I was hoping >for. I plugged it into a Battery Analyzer at 1.0 and 0.5 A and >momentarily dropped the input voltage to 8V. Even when voltage is >restored it takes several seconds to recover back to the set voltage >with a 1 A load. A 0.5 A load is fine. Good investigative work sir! You're on the right track. The up-converter you have is a bit on the light side. I'd go for beefy . . . and the up-converter needs to be characterized in Watts. A converter that would support a 5A load at 14V would be rated at 70 W or more. The test articles I have on hand are, as I recall, 150 W devices. I'm hoping to put power to them to see if I can characterize their dynamic qualities before I send you one. I'm a bit out of pocket right now. Getting ready for some roto-rooter work on my back . . . got a leg that occasionally says, "I quit". It would not be a good thing to be loading a patient and suddenly find yourself on the ground asking the patient to help you up! Doc says it's a quick fix with excellent prospects for success but in the interim, my forays about the shop are limited. The up converters I have are, I believe, this one: https://tinyurl.com/yxndk24a Price is right, size is convenient . . . and there is little risk for having one that is too big. Thanks for sharing your observations. Bob . . . ________________________________________________________________________________
Subject: Re: Alternator sawtooth current generation
From: Charlie England <ceengland7(at)gmail.com>
Date: Sep 19, 2020
On 9/19/2020 9:11 AM, Robert L. Nuckolls, III wrote: >> Hi Mickey, >> >> Questions spawn questions. :-) >> It doesn't look like there's any issue with the voltage. >> >> If you can access a stand-alone high current DC amp meter, that would >> be my 1st step; wire it in series with the alternator B lead to >> verify your EIS current readings as vailid. > > Right on > > >> My money is 1st on measurement error, and my hedge is the battery. >> >> Charlie > > Agreed. I'd conduct some experiments with the sensor > and compared with another instrument and battery > power only. See if that puppy's calibration is good. > The 11-second period is interesting . . . wondering > if this might be a 'beat note' between two regular > features of noise on the bus irritating the a/d > conversion process in the instrumentation. > > But if the bus voltage is stable then some sort > of measurement error is high on the list of > probabilities. > > > Bob . . . > Bob, The image in his VAF thread shows V & I ranges; V variation is ~.3V; I is ~0--~10A. The only 'device' I can think of that might cycle like that, at that wattage level, would be a heater circuit that had a controller capable of ramping power up/down in response to temp changes. Oh...Pitot heat! Dynon and probably others now have temp controllers for their pitot heat. And the current is about right. Charlie -- This email has been checked for viruses by Avast antivirus software. https://www.avast.com/antivirus ________________________________________________________________________________
Subject: Re: Brownout Step Up Converter
From: Charlie England <ceengland7(at)gmail.com>
Date: Sep 19, 2020
On 9/19/2020 9:26 AM, Robert L. Nuckolls, III wrote: > At 11:02 PM 9/17/2020, you wrote: >> A couple weeks ago I ordered one of these off eBay: >> >> https://www.ebay.ca/itm/192243452240 >> >> >> DC-DC Converter >> 5 Amps >> Input 3-35 V >> Output 5-40 V >> >> I'm trying to support ~1 A EFIS during start so I thought 5 A would >> be plenty but a closer look at the description says it's good for 3 A >> with a maximum of 5 A. Turns out the 3 A is on the input side, not >> the output side. With a voltage output set point of 10V and a ~1 A >> lamp connected, the lamp browns out (literally) as soon as the input >> voltage sags below 9V. A long way from the 7V minimum I was hoping >> for. I plugged it into a Battery Analyzer at 1.0 and 0.5 A and >> momentarily dropped the input voltage to 8V. Even when voltage is >> restored it takes several seconds to recover back to the set voltage >> with a 1 A load. A 0.5 A load is fine. > > Good investigative work sir! You're > on the right track. The up-converter > you have is a bit on the light side. > > I'd go for beefy . . . and the up-converter > needs to be characterized in Watts. A converter > that would support a 5A load at 14V would be > rated at 70 W or more. > > The test articles I have on hand are, as I recall, > 150 W devices. I'm hoping to put power to them > to see if I can characterize their dynamic qualities > before I send you one. > > I'm a bit out of pocket right now. Getting ready for > some roto-rooter work on my back . . . got a leg > that occasionally says, "I quit". It would not be > a good thing to be loading a patient and suddenly > find yourself on the ground asking the patient > to help you up! > > Doc says it's a quick fix with excellent > prospects for success but in the interim, > my forays about the shop are limited. The > up converters I have are, I believe, this > one: > > https://tinyurl.com/yxndk24a > > Price is right, size is convenient . . . > and there is little risk for having > one that is too big. Thanks for sharing > your observations. > > > Bob . . . > These: https://www.ebay.com/itm/DC-DC-Converter-10-12-15-20A-150-250-300-400-1200W-Step-up-Step-down-Buck-Boost/382219150784?hash=item58fe0965c0:g:6OwAAOSwisVZrgue Claim to be good down to 8.5V input. Various wattage versions available. Charlie -- This email has been checked for viruses by Avast antivirus software. https://www.avast.com/antivirus ________________________________________________________________________________
Subject: Landing and Taxi lights wig wag
From: "meat_ball" <arjayefem(at)fastmail.net>
Date: Sep 19, 2020
Hello all again. I want to install AeroLeds Aerosun landing and taxi lights. Two lights total, one will be oriented for taxi attitude and one slightly higher orientation for the landing light attitude on my tail dragger. That said, my thoughts, since these lights have a built in wig-wag function, are to do just that. However, I cannot figure out how to wire it. According to the install guide for the lights, it says to have 2 switches. One for the landing lights, and one for the wig wag function, which is fine if you are using both lights as landing lights. Since I want one light as a landing light and one as a taxi light, but use both for the wig-wag function, how does that work switch-wise? One switch as OFF-Taxi-Both, and the other wig-wag? There isn't enough information to know if when the wig-wag is "ON" the other switch is bypassed? Am I thinking along the right lines? Better way to do this? Any help? I've attached the install manual wiring diagram... Thanks so much! Rob Read this topic online here: http://forums.matronics.com/viewtopic.php?p=498434#498434 Attachments: http://forums.matronics.com//files/screenshot_2020_09_19_0100_0003_01_2120_aerosun_installation_revb_pdf_166.png ________________________________________________________________________________
From: Sebastien <cluros(at)gmail.com>
Date: Sep 19, 2020
Subject: Re: Landing and Taxi lights wig wag
Hello Rob, I think you are overthinking it. By all means aim them differently so that you have the best lighting for both landing and taxi, and using the wig wag function is better than not using the wig wag function regardless of which way they are aiming, but why not just wire it exactly as per that drawing? Are you ever going to need to turn off just one light? On Sat, Sep 19, 2020 at 8:52 AM meat_ball wrote: > arjayefem(at)fastmail.net> > > Hello all again. > > I want to install AeroLeds Aerosun landing and taxi lights. Two lights > total, one will be oriented for taxi attitude and one slightly higher > orientation for the landing light attitude on my tail dragger. That said, > my thoughts, since these lights have a built in wig-wag function, are to do > just that. However, I cannot figure out how to wire it. > > According to the install guide for the lights, it says to have 2 > switches. One for the landing lights, and one for the wig wag function, > which is fine if you are using both lights as landing lights. Since I want > one light as a landing light and one as a taxi light, but use both for the > wig-wag function, how does that work switch-wise? One switch as > OFF-Taxi-Both, and the other wig-wag? There isn't enough information to > know if when the wig-wag is "ON" the other switch is bypassed? Am I > thinking along the right lines? Better way to do this? > > Any help? I've attached the install manual wiring diagram... > > Thanks so much! > > Rob > > > Read this topic online here: > > http://forums.matronics.com/viewtopic.php?p=498434#498434 > > > Attachments: > > > http://forums.matronics.com//files/screenshot_2020_09_19_0100_0003_01_2120_aerosun_installation_revb_pdf_166.png > > ________________________________________________________________________________
From: Sebastien <cluros(at)gmail.com>
Date: Sep 19, 2020
Subject: Re: Brownout Step Up Converter
Now I'm getting confused. The converter Bob linked to has a minimum of 10 volts input. Charlie's is 8.5 volts. The only time we want these units working is below 10 volts, and they need to work much lower than that. The last start on my aircraft recorded a 7.6 from a fully charged battery and that's just what the EFIS managed to capture at 1 sample per second. Looking again I have found a couple units that claim to work at very low voltages: https://www.ebay.ca/itm/DC-DC-10A-Buck-Boost-Converter-Step-Up-Down-regulator-Module-for-LED-Driver-US/254169431168 https://www.ebay.ca/itm/DC-DC-Buck-Boost-Converter-Adjustable-CC-CV-Step-Up-Down-Power-Supply-Module/124260980594 I'm considering ordering one for further testing. Bob best wishes for a quick recovery but take your time. Get yourself fully in fettle before worrying about anyone else! On Sat, Sep 19, 2020 at 8:17 AM Charlie England wrote: > On 9/19/2020 9:26 AM, Robert L. Nuckolls, III wrote: > > At 11:02 PM 9/17/2020, you wrote: > > A couple weeks ago I ordered one of these off eBay: > > https://www.ebay.ca/itm/192243452240 > > DC-DC Converter > 5 Amps > Input 3-35 V > Output 5-40 V > > I'm trying to support ~1 A EFIS during start so I thought 5 A would be > plenty but a closer look at the description says it's good for 3 A with a > maximum of 5 A. Turns out the 3 A is on the input side, not the output > side. With a voltage output set point of 10V and a ~1 A lamp connected, the > lamp browns out (literally) as soon as the input voltage sags below 9V. A > long way from the 7V minimum I was hoping for. I plugged it into a Battery > Analyzer at 1.0 and 0.5 A and momentarily dropped the input voltage to 8V. > Even when voltage is restored it takes several seconds to recover back to > the set voltage with a 1 A load. A 0.5 A load is fine. > > > Good investigative work sir! You're > on the right track. The up-converter > you have is a bit on the light side. > > I'd go for beefy . . . and the up-converter > needs to be characterized in Watts. A converter > that would support a 5A load at 14V would be > rated at 70 W or more. > > The test articles I have on hand are, as I recall, > 150 W devices. I'm hoping to put power to them > to see if I can characterize their dynamic qualities > before I send you one. > > I'm a bit out of pocket right now. Getting ready for > some roto-rooter work on my back . . . got a leg > that occasionally says, "I quit". It would not be > a good thing to be loading a patient and suddenly > find yourself on the ground asking the patient > to help you up! > > Doc says it's a quick fix with excellent > prospects for success but in the interim, > my forays about the shop are limited. The > up converters I have are, I believe, this > one: > > https://tinyurl.com/yxndk24a > > Price is right, size is convenient . . . > and there is little risk for having > one that is too big. Thanks for sharing > your observations. > > > Bob . . . > > These: > > https://www.ebay.com/itm/DC-DC-Converter-10-12-15-20A-150-250-300-400-1200W-Step-up-Step-down-Buck-Boost/382219150784?hash=item58fe0965c0:g:6OwAAOSwisVZrgue > Claim to be good down to 8.5V input. Various wattage versions available. > > Charlie > > > Virus-free. > www.avast.com > > <#m_4819214867954981025_DAB4FAD8-2DD7-40BB-A1B8-4E2AA1F9FDF2> > ________________________________________________________________________________
Subject: Re: Landing and Taxi lights wig wag
From: "meat_ball" <arjayefem(at)fastmail.net>
Date: Sep 19, 2020
cluros(at)gmail.com wrote: > Hello Rob, I think you are overthinking it. By all means aim them differently so that you have the best lighting for both landing and taxi, and using the wig wag function is better than not using the wig wag function regardless of which way they are aiming, but why not just wire it exactly as per that drawing? Are you ever going to need to turn off just one light? > Well, I thought of that as well, and thanks for the response! I am quite sensitive, because of my real job, to people blinding each other during night taxi ops. I feel its quite unneighborly and dont want to be in that position. Great point though...Ill have to think on it a bit more. Perhaps Im still overthinking it, but in either case, Id like to know if it would be possible to do what Im asking. Read this topic online here: http://forums.matronics.com/viewtopic.php?p=498436#498436 ________________________________________________________________________________
From: Mickey Coggins <mick-matronics(at)rv8.ch>
Date: Sep 19, 2020
Subject: Re: Alternator sawtooth current generation
Thanks for the hints. The voltage stays very constant. No pitot heat or other big consumers, except lights (landing/wigwag, position, strobes). I did some experiments today without the lights for a longer period, and with the fuel pump, and the current is smooth. I have the hall effect sensor between the alternator and the main bus on the B lead, so it should only be measuring the output of the alternator. It really looks like the "culprit" is the lights. I will talk to Paul the Flyleds guy and see if he's seen this. All my lights are Flyleds, but I use Eric's wig-wag controller. It could be totally normal that he grabs a lot of power in a cyclical way which is causing this. Starting to wish I had installed multiple hall sensors, but my system is so simple, I didn't really see a huge advantage. One other thing I see is that when I remove the consumers, and leave just the EFIS, radio, transponder, and pmags, the current from the alternator drops very low, and then the ALT warning light I have wired in there as shown by PlanePower flickers. It's a 12v LED from Fry's. I feel kind of bad that PP sent me a new alternator. I described the problem and they were certain it was a faulty alternator. I guess I should have done more investigation on my own. I can only say good things about their quick response to a problem! Got some rainy days coming up so I might play around with it a bit more before the next flight and report back. Thanks again for the suggestions! ________________________________________________________________________________
From: Sebastien <cluros(at)gmail.com>
Date: Sep 19, 2020
Subject: Re: Landing and Taxi lights wig wag
Install a separate switch for each red wire. On Sat, Sep 19, 2020, 09:36 meat_ball wrote: > arjayefem(at)fastmail.net> > > > cluros(at)gmail.com wrote: > > Hello Rob, I think you are overthinking it. By all means aim them > differently so that you have the best lighting for both landing and taxi, > and using the wig wag function is better than not using the wig wag > function regardless of which way they are aiming, but why not just wire i t > exactly as per that drawing? Are you ever going to need to turn off just > one light? > > > > > Well, I thought of that as well, and thanks for the response! I am quite > sensitive, because of my real job, to people blinding each other during > night taxi ops. I feel it=99s quite unneighborly and don=99t want to be in that > position. Great point though...I=99ll have to think on it a bit mor e. > Perhaps I=99m still overthinking it, but in either case, I=99 d like to know if > it would be possible to do what I=99m asking. > > > Read this topic online here: > > http://forums.matronics.com/viewtopic.php?p=498436#498436 > > =========== =========== =========== =========== =========== > > ________________________________________________________________________________
From: David Saylor <saylor.dave(at)gmail.com>
Date: Sep 19, 2020
Subject: Re: Landing and Taxi lights wig wag
Here's figure 5.3, but with the center lights and optional diode removed. I think that's what you're after. 5.2 would give you two landing lights that wig-wag. Not quite the same. [image: image.png] On Sat, Sep 19, 2020 at 8:52 AM meat_ball wrote: > arjayefem(at)fastmail.net> > > Hello all again. > > I want to install AeroLeds Aerosun landing and taxi lights. Two lights > total, one will be oriented for taxi attitude and one slightly higher > orientation for the landing light attitude on my tail dragger. That said, > my thoughts, since these lights have a built in wig-wag function, are to do > just that. However, I cannot figure out how to wire it. > > According to the install guide for the lights, it says to have 2 > switches. One for the landing lights, and one for the wig wag function, > which is fine if you are using both lights as landing lights. Since I want > one light as a landing light and one as a taxi light, but use both for the > wig-wag function, how does that work switch-wise? One switch as > OFF-Taxi-Both, and the other wig-wag? There isn't enough information to > know if when the wig-wag is "ON" the other switch is bypassed? Am I > thinking along the right lines? Better way to do this? > > Any help? I've attached the install manual wiring diagram... > > Thanks so much! > > Rob > > > Read this topic online here: > > http://forums.matronics.com/viewtopic.php?p=498434#498434 > > > Attachments: > > > http://forums.matronics.com//files/screenshot_2020_09_19_0100_0003_01_2120_aerosun_installation_revb_pdf_166.png > > ________________________________________________________________________________
Date: Sep 19, 2020
From: "Robert L. Nuckolls, III" <nuckolls.bob(at)aeroelectric.com>
Subject: Getting the numbers . . .
> >The image in his VAF thread shows V & I ranges; V variation is ~.3V; >I is ~0--~10A. > >The only 'device' I can think of that might cycle like that, at that >wattage level, would be a heater circuit that had a controller >capable of ramping power up/down in response to temp changes. > >Oh...Pitot heat! Dynon and probably others now have temp controllers >for their pitot heat. And the current is about right. > >Charlie Aha! yeah . . . I'm betting it's pretty simple. It's unfortunate that the condition created so much work . . . especially doing 'swap-tronics'. I've always taught that one should KNOW what needs to be removed before picking up any wrenches. Sometimes getting the numbers can be an exercise . . . worked an intermittent in the tail de-ice system on a Beechjet that had grounded a revenue generating airplane for over a month. Seems the problem only manifested after 15 or 20 minutes at altitude. A LOT of fuel and hours were consumed before I was asked to join the search. We needed to take measurements back in the 'hell hole' just above the baggage compartment and outside the pressure vessel. Tried to capture the glitch with my laptop DAS strapped down to the floor of the baggage. compartment. It takes quite awhile and lots of gas to get to 41,000 feet . . . but about 12 minutes after leveling off, the system glitched and we hoped to 'catch it' on the DAS. No joy . . . the heads on the lap-top's hard- drive wouldn't 'fly' at 41K feet! That was long before solid state hard drives. http://aeroelectric.com/Pictures/Misc/RK371_Deice_A.JPG http://aeroelectric.com/Pictures/Misc/RK371_Deice_B.JPG http://aeroelectric.com/Pictures/Misc/RK371_Deice_C.JPG http://aeroelectric.com/Pictures/Misc/RK371_Deice_D.JPG Plan B: I built a breakout box that would let me bring the sample leads out to a ribbon cable routed through the baggage compartment door gasket, taped to the fuselage and into the cabin through the cabin door gasket. Had to put an x-ticket on the airplane. I could sit in the cabin and watch the system parameters. On flight #2, the system 'glitched' again and the DAS was in my lap. Knew exactly which system wire to scope out. Found a pushed back pin in a pressure bulkhead connector that would de-mate as the wire bundle contracted in the cold air. Sometimes the simplest things can be exceedingly expensive to find . . . in this instance it was tens of killobux! But in this case, as in virtually all others, known good numbers are the key solving the most intractable problems. Bob . . . ________________________________________________________________________________
Subject: Re: Need small physical size fuse 1/2 amp
From: "rparigoris" <rparigor(at)hotmail.com>
Date: Sep 19, 2020
Hi Bob 2 LEDs and 1 mini relay 1 LED needs power from always hot bus about 4 feet away from LED, there's another wire within inches 1 LED about 4 feet away from main bus, other power sources within inches mini relay needs power from a specific wire that's passing right by the relay The LEDs and mini relay are very low power, for robustness of wire will use 20 or 22, 1/2 or 1 amp should be fine. Ron P. Read this topic online here: http://forums.matronics.com/viewtopic.php?p=498444#498444 ________________________________________________________________________________
Subject: Re: Landing and Taxi lights wig wag
From: "rparigoris" <rparigor(at)hotmail.com>
Date: Sep 19, 2020
Hi Rob I just did exactly what you want. I used 3 switches. I checked with AeroLED and you can't hurt anything no matter how many switches you have on at once. Just Wig Wag will Wig Wag. Landing on and Wig Wag will Wig Wag. Taxi on and Wig Wag on will Wig Wag. Both Landing and Taxi on and Wig Wag on will Wig Wag. Note when turning on Wig Wag both LEDs will light together for a short time at the same time. I have a an Aerosun Landing and Microsun Taxi. I'm using 7 1/2 amp fuse for Wig Wag, 4 amp Aerosun and 3 amp for Microsun. I have mine located within a few inches of each other. Ron P. Read this topic online here: http://forums.matronics.com/viewtopic.php?p=498445#498445 ________________________________________________________________________________
From: David Carter <david(at)carter.net>
Date: Sep 19, 2020
Subject: S701-1 master relay wiring?
The attached picture is as received from B&C. According to the enclosed wiring instructions / picture, it appears to be wired backwards. The jumper wire is on the wrong side, and the diode appears to be backwards. Anyone else seen this? I'll put it on the test bench & verify the connections before I install it. -- --- David Carter david(at)carter.net ________________________________________________________________________________
Subject: Re: Landing and Taxi lights wig wag
From: "meat_ball" <arjayefem(at)fastmail.net>
Date: Sep 19, 2020
Dave Saylor wrote: > Here's figure 5.3, but with the center lights and optional diode removed. I think that's what you're after. > > > 5.2 would give you two landing lights that wig-wag. Not quite the same. > > Much obliged, Dave! I didnt think to use that diagram because it had too many lights on it...haha. I need to think out of the box I guess. Read this topic online here: http://forums.matronics.com/viewtopic.php?p=498448#498448 ________________________________________________________________________________
Subject: Re: Landing and Taxi lights wig wag
From: "meat_ball" <arjayefem(at)fastmail.net>
Date: Sep 19, 2020
rparigoris wrote: > Hi Rob I just did exactly what you want. I used 3 switches. I checked with AeroLED and you can't hurt anything no matter how many switches you have on at once. Just Wig Wag will Wig Wag. Landing on and Wig Wag will Wig Wag. Taxi on and Wig Wag on will Wig Wag. Both Landing and Taxi on and Wig Wag on will Wig Wag. Note when turning on Wig Wag both LEDs will light together for a short time at the same time. I have a an Aerosun Landing and Microsun Taxi. I'm using 7 1/2 amp fuse for Wig Wag, 4 amp Aerosun and 3 amp for Microsun. I have mine located within a few inches of each other. Ron P. Sound good Ron! Thanks so much. Thats the info I was looking for. Since thats the circuitry for the wig-wag, I suppose it wouldnt be difficult to combine the landing and taxi circuits onto one double throw switch...you agree? Im trying to minimize my panel layout... Read this topic online here: http://forums.matronics.com/viewtopic.php?p=498449#498449 ________________________________________________________________________________
From: Kent or Jackie Ashton <kjashton(at)vnet.net>
Subject: Re: S701-1 master relay wiring?
Date: Sep 19, 2020
The contactor can work in either direction so each picture is wired correctly. For the second pic, from the battery would be on the right and master switch on the left. I will be embarrassed if Bob says otherwise. :-) The diode works so that when the the master switch is turned off, the coil current collapses creating a surge but the surge cannot return to arc the master switch contacts. It can only go towards the battery which does not care. -Kent > On Sep 19, 2020, at 3:36 PM, David Carter wrote: > > The attached picture is as received from B&C. According to the enclosed wiring instructions / picture, it appears to be wired backwards. The jumper wire is on the wrong side, and the diode appears to be backwards. Anyone else seen this? I'll put it on the test bench & verify the connections before I install it. > > > -- > --- > David Carter > david(at)carter.net ________________________________________________________________________________
From: Charlie England <ceengland7(at)gmail.com>
Date: Sep 19, 2020
Subject: Re: Need small physical size fuse 1/2 amp
On Sat, Sep 19, 2020 at 2:01 PM rparigoris wrote: > rparigor(at)hotmail.com> > > Hi Bob > > 2 LEDs and 1 mini relay > > 1 LED needs power from always hot bus about 4 feet away from LED, there's > another wire within inches > > 1 LED about 4 feet away from main bus, other power sources within inches > > mini relay needs power from a specific wire that's passing right by the > relay > > The LEDs and mini relay are very low power, for robustness of wire will > use 20 or 22, 1/2 or 1 amp should be fine. > > Ron P. *With a few inches of feeder, **I'd probably just hook them up to the closest power feeder with same-gauge wire, if available sources are on when the LED or relay needs to be on. With inches of wire in each case, the weight would be barely measurable. Only thing that might give me pause is if the selected power source fed a flight-critical component, and with reasonable wire routing, there'd be little risk, regardless. If an indicator LED shorts, it should 'self fuse' (burn open) almost instantly, if it's current limit resistor allows it. The current limit resistor could be at the power tap, if it uses a separate resistor.I suspect that a mini relay coil would self fuse, as well, but that's a little less certain. You might well be able to use a series resistor at the source as a current limiter for the relay coil, as well.* *Charlie* ________________________________________________________________________________
Date: Sep 19, 2020
From: "Robert L. Nuckolls, III" <nuckolls.bob(at)aeroelectric.com>
Subject: Re: S701-1 master relay wiring?
At 02:36 PM 9/19/2020, you wrote: >The attached picture is as received from >B&C.=C2 According to the enclosed wiring >instructions / picture, it appears to be wired >backwards.=C2 The jumper wire is on the wrong >side, and the diode appears to be >backwards.=C2 Anyone else seen this? I'll put it >on the test bench & verify the connections before I install=C2 it. The contactor IS a symmetrical device. If for mechanical reasons one wished to reverse the input/output functons of the main terminals, you could flip the diode and move the jumper to the other side. But if installation mechanics are no issue then the device is correctly configured as supplied and yes, the illustration describes the ALTERNATE but equally functional condition. Bob . . . ________________________________________________________________________________
From: A RICHARD GOLDMAN <argoldman(at)aol.com>
Date: Sep 19, 2020
Subject: Re: S701-1 master relay wiring?
Dave, Perhaps the picture reflects that the master relay is activated by connectin g the pole to ground not 12v If that were not the case, you would need to turn on the master switch to t hen turn on the master switch. This also prevents the necessity to always have a constant hot wire to the p anel. What could possibly go wrong with that? Rich Sent from my iPhone > On Sep 19, 2020, at 2:36 PM, David Carter wrote: > > The attached picture is as received from B&C. According to the enclosed w iring instructions / picture, it appears to be wired backwards. The jumper w ire is on the wrong side, and the diode appears to be backwards. Anyone els e seen this? I'll put it on the test bench & verify the connections before I install it. > > > -- > --- > David Carter > david(at)carter.net ________________________________________________________________________________
Subject: Re: S701-1 master relay wiring?
From: "user9253" <fransew(at)gmail.com>
Date: Sep 19, 2020
Arc suppression diodes are always wired with the banded end connected to positive. The diagram is correct. -------- Joe Gores Read this topic online here: http://forums.matronics.com/viewtopic.php?p=498454#498454 ________________________________________________________________________________
From: Art Zemon <art(at)zemon.name>
Date: Sep 19, 2020
Subject: Re: Landing and Taxi lights wig wag
Rob, I did essentially the same thing you describe with a pair of AeroSun lamps. The only difference is that I just turned them both on all of the time. I want to be seen. I really don't care whether my taxi light is on when I am landing. I want *all* of the lights on. Here is my wiring diagram. (The AeroLEDs diagram is included on the lower, left corner of the page.) landing taxi lights.pdf BTW, I think that you will be delighted with how bright these are. I have had multiple people come up to me on the ramp and say stuff like, "Whoa! You are super bright and easy to see!" I never got around to aiming the lights very carefully and they still illuminate the ramp quite well. Cheers, -- Art Z. On Sat, Sep 19, 2020 at 11:02 AM meat_ball wrote: > arjayefem(at)fastmail.net> > > Hello all again. > > I want to install AeroLeds Aerosun landing and taxi lights. Two lights > total, one will be oriented for taxi attitude and one slightly higher > orientation for the landing light attitude on my tail dragger. That said, > my thoughts, since these lights have a built in wig-wag function, are to do > just that. However, I cannot figure out how to wire it. > > According to the install guide for the lights, it says to have 2 > switches. One for the landing lights, and one for the wig wag function, > which is fine if you are using both lights as landing lights. Since I want > one light as a landing light and one as a taxi light, but use both for the > wig-wag function, how does that work switch-wise? One switch as > OFF-Taxi-Both, and the other wig-wag? There isn't enough information to > know if when the wig-wag is "ON" the other switch is bypassed? Am I > thinking along the right lines? Better way to do this? > > Any help? I've attached the install manual wiring diagram... -- https://CheerfulCurmudgeon.com/ *Each of us is worth only what we are willing to give away to others. -- Lynn Schusterman* ________________________________________________________________________________
From: Art Zemon <art(at)zemon.name>
Date: Sep 19, 2020
Subject: Re: Landing and Taxi lights wig wag
Rob, That makes perfect sense. For that reason, I have my strobes on a separate switch and when I am taxiing toward someone at night, I turn off my landing/taxi lights (unless I really need them). 99% of my flying is during the day, though, so I just leave them on wig-wag all the time. -- Art Z. On Sat, Sep 19, 2020 at 11:50 AM meat_ball wrote: > arjayefem(at)fastmail.net> > > > cluros(at)gmail.com wrote: > > Hello Rob, I think you are overthinking it. By all means aim them > differently so that you have the best lighting for both landing and taxi, > and using the wig wag function is better than not using the wig wag > function regardless of which way they are aiming, but why not just wire i t > exactly as per that drawing? Are you ever going to need to turn off just > one light? > > > > > Well, I thought of that as well, and thanks for the response! I am quite > sensitive, because of my real job, to people blinding each other during > night taxi ops. I feel it=99s quite unneighborly and don=99t want to be in that > position. Great point though...I=99ll have to think on it a bit mor e. > Perhaps I=99m still overthinking it, but in either case, I=99 d like to know if > it would be possible to do what I=99m asking. > > > Read this topic online here: > > http://forums.matronics.com/viewtopic.php?p=498436#498436 > > =========== =========== =========== =========== =========== > > -- https://CheerfulCurmudgeon.com/ *Each of us is worth only what we are willing to give away to others. -- Lynn Schusterman* ________________________________________________________________________________
Subject: Re: Landing and Taxi lights wig wag
From: "meat_ball" <arjayefem(at)fastmail.net>
Date: Sep 20, 2020
art(at)zemon.name wrote: > Rob, > > > I did essentially the same thing youdescribe with a pair of AeroSun lamps. The only difference is that I just turned them both on all of the time. I want to be seen. I reallydon't care whether my taxi light is on when I am landing. I want allof the lights on. > > > Here is my wiring diagram. (The AeroLEDs diagram is included on the lower, left corner of the page.) > > [img]https://drive-thirdparty.googleusercontent.com/16/type/application/pdf[/img]landing taxi lights.pdf (https://drive.google.com/file/d/0BzOP2gb9_3RQQkt6aWw3aTU2NVk/view?usp=drive_web) > > > BTW, I think that you will be delighted with how bright these are. I have had multiple people come up to me on the ramp and say stuff like, "Whoa! You are super bright and easy to see!" I never got around to aiming the lights very carefully and they still illuminate the ramp quite well. > > > Cheers, > -- Art Z. > Art, thanks for the response. All makes perfect sense! I have a question about your wiring diagram. You have a green circle around Positions 4, 5, 6 in 2 different places on the upper right. Will you explain what those are depicting as I am not familiar with that symbology? Read this topic online here: http://forums.matronics.com/viewtopic.php?p=498461#498461 ________________________________________________________________________________
From: Charlie England <ceengland7(at)gmail.com>
Date: Sep 20, 2020
Subject: Re: S701-1 master relay wiring?
Just to be sure all bases are covered... If the diode & jumper are reversed, the battery connection and switch connection will be reversed, as well. The master switch is the only switch that's in the 'ground side' of a circuit. Instead of supplying power to the device (relay coil, in this case), it completes the circuit to ground. That avoids having an extra 'hot' wire making the long run to the switch, and avoids the need to protect the wire with a fuse/breaker. Charlie On Sun, Sep 20, 2020 at 7:18 AM Kent or Jackie Ashton wrote: > kjashton(at)vnet.net> > > The contactor can work in either direction so each picture is wired > correctly. For the second pic, =9Cfrom the battery=9D would be on the right=9D > and =9Cmaster switch=9D on the left. I will be embarrassed if Bob says > otherwise. :-) > > The diode works so that when the the master switch is turned off, the coi l > current collapses creating a surge but the surge cannot return to arc the > master switch contacts. It can only go towards the battery which does no t > care. > -Kent > > > On Sep 19, 2020, at 3:36 PM, David Carter wrote: > > > > The attached picture is as received from B&C. According to the enclose d > wiring instructions / picture, it appears to be wired backwards. The > jumper wire is on the wrong side, and the diode appears to be backwards. > Anyone else seen this? I'll put it on the test bench & verify the > connections before I install it. > > > > > > -- > > --- > > David Carter > > david(at)carter.net > > =========== =========== =========== =========== =========== > > ________________________________________________________________________________
Date: Sep 20, 2020
From: "Robert L. Nuckolls, III" <nuckolls.bob(at)aeroelectric.com>
Subject: evolution of lithium technologies
I've been studying current options for lithium cells on the market. Discovered some interesting things. For example . . . this excerpt from the engineering data sheet on an 18650 cell cited as suitable for battery powered tools: ---------------------------- 9.0. Safety 9.1 Overcharge test Test method: To charge with 20A-20V at 25=81=8E for 3hr. Criteria: No fire, and no explosion. 9.2 External short-circuit test Test method: To short-circuit the standard charged cell (or 50% discharged cell) by connecting positive and negative terminal by 80mOhm wire for 10min. Criteria: No fire, and no explosion. 9.3 Reverse charge test Test method: To charge the standard charged cell with charge current 10A By 20V for 2.5 hours. Criteria: No fire, and no explosion. 9.4 Heating test Test method: To heat up the standard charged cell at heating rate 5=81=8E per minute up to 130=81=8E and keep the cell in oven for 10 minutes. Criteria: No fire, and no explosion. ------------------------------- What insults might these cells endure if incorporated into a flight system? Any worse than those cited above? Just thinking . . . Bob . . . ________________________________________________________________________________
Subject: Re: Landing and Taxi lights wig wag
From: "rparigoris" <rparigor(at)hotmail.com>
Date: Sep 20, 2020
Hi Rob I think using 1 switch for both Taxi and Landing LED should be fine. I suggest you try it, could do a temporary on bench or in situation test. Because you are not wanting to control LEDs individual, you could get away with 1 power fuse. Measure amperage draw during your test. Ron Read this topic online here: http://forums.matronics.com/viewtopic.php?p=498465#498465 ________________________________________________________________________________
Date: Sep 20, 2020
From: "Robert L. Nuckolls, III" <nuckolls.bob(at)aeroelectric.com>
Subject: who wudda thunk it?
I'm replacing an in-dash radio in the family chariot . . . seems the venerable ol' Kenwood CD player radio just doesn't cut it any more. Nobody carries CDs in the car and everyone wants to link their mobile phone to the radio via blue tooth. Ahhh fair Kenwood, tho has served me well . . . RIP. The vehicle to radio adapter harness needs to transition from Kenwood to JVC. Its a harness I built about 6 years ago. I was surprised to find that about some of the shrink over splices between radio and vehicle connectors had MIGRATED to locations that no longer cover the solder joint. I know that the shrink had a good grip on the irregularities of the solder joint when the harness was built. But after years of temperature cycles under the dash, the heat shrink kinda un-shrunk! It became more round and released it's grip on the joint. This allowed the tubing to vibrate clear of the solder joint! Only a few actually moved but all were loose and could be easily moved. We now know a lot more than we understand. Funky brand of HS? Normal behavior for a polyolfin shrink? I'm getting ready to mod the harness with a new radio connector . . . I think I'll use something different for joints. Perhaps crimp-shrink butt splices, solder sleeves or maybe double wall heat shrink with the internal adhesive. So many choices but no doubt all of them better than the choice I made last time. Just a heads-up guys.


August 25, 2020 - September 20, 2020

AeroElectric-Archive.digest.vol-po